Sunteți pe pagina 1din 96

Anul XIV, Nr.

Iulie Decembrie 2012

RECREAII
MATEMATICE
REVIST DE MATEMATIC PENTRU ELEVI I PROFESORI

e i = 1

Asociaia Recreaii Matematice


IAI - 2012

Semnificaia formulei de pe copert:


i
ntr-o form concis, formula e = 1 leag cele patru ramuri fundamentale

ale matematicii:

ARITMETICA
GEOMETRIA
ALGEBRA
ANALIZA MATEMATIC

reprezentat
reprezentat
reprezentat
reprezentat

de
de
de
de

i
e

Redacia revistei :
Petru ASAFTEI, Temistocle BRSAN, Dan BRNZEI , Alexandru CRUU, Constantin
CHIRIL, Eugenia COHAL, Adrian CORDUNEANU, Mihai CRCIUN (Pacani),
Paraschiva GALIA, Paul GEORGESCU, Gheorghe ILIE, Gheorghe IUREA, Gabriel
MRANU, Gabriel POPA, Dan POPESCU (Suceava), Maria RACU, Neculai ROMAN
(Mirceti), Ioan ERDEAN (Ortie), Dan TIBA (Bucureti), Marian TETIVA (Brlad),
Lucian TUESCU (Craiova), Adrian ZANOSCHI, Titu ZVONARU (Comneti)
Materialele se trimit la una dintre adresele: t_birsan@yahoo.com, profgpopa@yahoo.co.uk
Pagina web a revistei RecreaiiMatematice: http://www.recreatiimatematice.ro
COPYRIGHT 2012, ASOCIAIA RECREAII MATEMATICE
Toate drepturile aparin Asociaiei Recreaii Matematice. Reproducerea integral sau
parial a textului sau a ilustraiilor din aceast revist este posibil numai cu acordul prealabil
scris al acesteia. Se consider c autorii materialelor trimise redaciei revistei sunt, n mod
implicit, de acord cu publicarea lor, i asum responsabilitatea coninutului lor i cedeaz
Asociaiei Recreaii Matematice dreptul de proprietate intelectual asupra acestora.
TIPRIT LA BLUE SIM&Co IAI
Bd. Carol I, nr. 3-5
Tel. 0332 111021, 0721 571705
E-mail: simonaslf@yahoo.com
ISSN 1582 - 1765

Anul XIV, Nr. 2

Iulie Decembrie 2012

RECREAII
MATEMATICE
REVIST DE MATEMATIC PENTRU ELEVI I PROFESORI

e i = 1
Revist cu apariie semestrial

EDITURA RECREAII MATEMATICE

IAI - 2012


DAN BRANZEI
(1942 - 2012)
Avea mpliniti de curand, n luna martie, 70 de ani cand rul vietii sale s-a
rupt. O viat
a trait
a intens, devotata matematicii, daruita nvatamantului matematic romanesc, o viat
a de generoasa ndrumare a multor generatii de adolescenti
si tineri porniti pe urcusul anevoios al performantei n matematica. Dan Br
anzei
a fost o personalitate puternica, deopotriva fascinant
a si atipica. Cu o inteligenta
sclipitoare si o fantezie nestavilit
a, capta imediat atentia si interesul interlocutorilor.
A iubit oamenii si a fost iubit si pretuit de acestia. Pe ultimul sau drum din cimitirul
Eternitatea, n luna mai, au venit sa-l conduca si sa-si exprime durerea si regretele
o multime de apropiati, colaboratori si cunoscuti. Mormantul sau a fost acoperit de
coroane de ori din partea multor prieteni si institutii.
Nascut la Iasi, n 1942, n plina desfasurare a razboiului, a ramas pentru ntreaga
viata legat de orasul Iasi pe care l-a ndragit. Parintii sai au fost doctori; ntr-o
sa autobiograca si aminteste mai tarziu: ,,Mama a reusit s
a aduc
a din Basarabia
un geamandan n care abundau c
arti [...] Tata pornise din T
ib
anestii Iasului [...]
Ca sugar i-am urmat pe ei, care nsoteau (ca medici) c
and spre miaz
azi, c
and spre
miaz
anoapte, linia frontului [...] Cu vremea, tata a devenit un psihiatru de renume
mondial si eu m-am simtit mereu dator s
a nu-i ntinez amintirea.
A absolvit Liceul Internat ,,Costache Negruzzi n anul 1959. Aici - spune n
aceeasi sa autobiograca -, ,,am beneciat de un profesor de matematic
a cu neasemuite calit
ati, Nicolae Colibaba, care a fost - spune n alt loc - o ,,sans
a a devenirii
mele. Colaboreaza la Gazeta Matematic
a, n numerele din anii 1957-1960 numele lui
aandu-se printre cele ale rezolvitorilor de probleme evidentiati. Are rezultate remarcabile la olimpiadele scolare de matematica. Este coleg cu Viorel Barbu si Octavian
Nanu, alti doi eminenti elevi cu nclinatii spre matematica.
In perioada 1959-1964 cei trei colegi de liceu devin colegi de facultate, Facultatea
de Matematic
a-Fizic
a a Universit
atii ,,Al. I. Cuza, si vor ramane colegi ca tineri
asistenti la aceeasi facultate. Sub conducerea prof. dr. Gheorghe Gheorghiev si
sustine teza de doctorat cu tema Structuri de variet
ati diferentiabile generalizate n
anul 1976. In anul 1992 a fost titularizat ca profesor universitar la facultatea ieseana.
Dan Br
anzei a mbinat n mod armonios cercetarea stiintica cu activitatea
didactic
a. In lunga sa cariera universitara a predat diverse cursuri de geometrie (sintetica, analitica, proiectiva, diferentiala, neeuclidiana), dar si fundamentele matematicii, matematici pentru economisti, metodica predarii matematicii s.a. Dan Br
anzei
este prin excelent
a un geometru, dovada teza sa de doctorat, articolele publicate,
precum si cartile si manualele scrise. Lucrul temeinic documentat si riguros prezentat, ideile scoase bine n evident
a si puse n valoare, fraza dinamica, stilul accesibil
si placut, nelipsind nici umorul si nici accentele critice, fac scrierile lui deopotriva
utile/fundamentale si agreabile.
Cu o pasiune constant
a se intereseaza de cele mai diverse domenii ale cunoasterii
sau se implica n variate actiuni concrete. Un roman bun, o teorie noua, un joc
85

de sah, o petrecere cu prietenii, si gasesc cuprindere n suetul lui mare, necesare


pentru nepotolita lui sete intelectuala si de viata. A fost un pasionat participant
la concursurile de orientare turistica n munti si apoi a organizat/jurizat el nsusi
astfel de concursuri. Pasiunea pentru orientarea turistica s-a metamorfozat mai apoi
n aceea de generos si competent ndrumator al multor generatii de elevi olimpici n
directia matematicii.
Vechea colaborare cu Gazeta matematic
a si are acum continuare ntr-o sustinuta
si rodnica activitate n cadrul SSMR si a lialei sale din Iasi n folosul nvatamantului
preuniversitar romanesc. Ar sucient sa enumeram doar multele responsabilitati pe
care si le-a asumat si onorat cu competenta si har, cu detasare si daruire; amintim
cateva: membru n biroul comitetului de conducere al SSM, liala Iasi a SSM l-a avut
succesiv , ncep
and cu anul 1968 si pana la sfarsitul vietii, ca secretar, vicepresedinte
si presedinte, membru al comisiei de acreditare a manualelor scolare din MECT,
consilier stiintic la editura Paralela 45, presedintele multor concursuri scolare de
prestigiu s.a. A fost membru n redactiile celor mai importante reviste de matematici
elementare din tar
a, n primul rand Gazeta Matematic
a. A intrat n redactia revistei
iesene Recreatii Matematice ncepand cu al doilea an de existenta a ei si a contribuit
la promovarea acesteia cu articole si prestigiul sau.
Dan Br
anzei a dedicat mult timp si multa energie modernizarii nvatamantului
scolar romanesc, n care scop a desfasurat o bogata activitate publicistica. Pentru ridicarea nivelului de cunostinte ale elevilor si profesorilor a scris, cel mai adesea n colaborare, monograi: Planul si spatiul euclidian (Ed. Academiei, 1986),
Analogii triunghi-tetraedru (Paralela 45, 2000), Probleme de pivotare (Ed. Recreatii
Matematice, 2011) sau carti de sinteza n care sunt expuse bazele matematicii: Bazele
rationamentului geometric (Ed. Academiei, 1983) sau, n colaborare cu acad. R.
Miron, Fundamentele aritmeticii si geometriei (Ed. Academiei, 1983, care a cunoscut o editie n engleza, Backgrounds of Aritmetic and Geometry. An Introduction,
World Scientic,1995). Este coautor al unor manuale de geometrie de succes, pentru
toate clasele la care programele prevad acest obiect. Un numar nsemnat de culegeri
de probleme sau carti ce contin subiectele, nsotite de solutii, date la balcaniada de
matematica pentru juniori, olimpiada nationala si diverse concursuri scolare vin n
sprijinul elevilor care au ales calea performantei n matematica.
Una dintre activitatile la care tinea, poate cel mai mult, a fost pregatirea lotului de
juniori al Romaniei pentru balcaniadele matematice, nsotirea si conducerea lotului
reprezentativ de juniori al tarii la aceste balcaniade. S-a atasat cu tot suetul de
acesti copii n care vedea un ,,z
ac
am
ant de pietre pretioase si despre care opina ca
,,eu nu stiu s
a aib
a tara noastr
a avere mai de seam
a dec
at acesti copii grozavi!. De
altfel, rasplata cea mare pentru munca si toata daruirea lui i-a venit tot de la acesti
copii si tineri inteligenti si minunati, care, de-a lungul timpului, l-au ntampinat cu
bucurie si ndelungi aplauze.
S
a ne bucuram ca l-am cunoscut, sa-i multumim pentru tot ce a facut pentru progresul nv
at
am
antului romanesc, sa folosim tot ce am deprins de la dansul, purtandu-l
astfel n amintirea noastra.

Redactia revistei ,,Recreatii Matematice


86

Caius IACOB
100 de ani de la nastere

Caius Iacob face parte din pleiada de


matematicieni romani care au contribuit la
recunoasterea si prestigiul scolii matematice
romanesti n lumea ntreag
a.
S-a nascut la 29 martie 1912, la Arad.
Tatal sau, Lazar Iacob, a fost profesor de drept
canonic la Arad, Oradea, Cluj si n nal la
Facultatea de Teologie din Bucuresti. A fost
printre delegatii ociali la Marea Adunare de
la Alba Iulia, 1 decembrie 1918. Mama sa,
Camelia, nascut
a Moldovan, era originara din
Lugoj.
A urmat primele trei clase la Liceul ,,Moise
Nicoar
a din Arad, a continuat la Liceul
,,Emanuil Gojdu
din Oradea, unde si-a
sustinut bacalaureatul, n 1928, cu media cea
mai mare pe tar
a. A fost un colaborator activ
al Gazetei Matematice si Revistei Matematice din Timisoara.
Intre anii 1928 si 1931 a urmat cursurile Facultatii de Stiinte a Universit
atii din
Bucuresti, unde a avut profesori renumiti ca Gh. T
iteica, D. Pompeiu,V. V
alcovici
s.a. Este licentiat n matematici la 19 ani, de asemenea fruntas al seriei.
In perioada 1931-1935 este la Paris, beneciind de o bursa de specializare n
mecanica uidelor la Institutul de mecanica de la Sorbona. Urmeaza cursurile celor
mai renumiti matematicieni si mecanicieni ai Frantei din vremea aceea: E. Cartan,
E. Borel, H. Lebesgue, H. Villat, M. Frechet s.a. si obtine certicate pentru analiza
superioara, aerodinamica si hidrodinamica. Cea mai puternica inuenta asupra sa a
avut-o H. Villat, sub conducerea caruia si sustine la 25 iunie 1935 teza de doctorat
Sur la determination des fonctions harmoniques conjuguees par certaines conditions
aux limites. Applications `
a hydrodynamique. A publicat aceasta teza n revista clujeana Mathematica, vol.11 (1935).
Rentors imediat n tar
a, a fost numit asistent mai ntai la Scoala Politehnica
din Timisoara (1935) si apoi la Universitatea din Cluj (1938) si la Universitatea din
Bucuresti (1939). In anul 1943 este numit conferentiar si apoi profesor la Universitatea
din Cluj (care functiona n acel moment la Timisoara), iar n 1950 devine profesor, sef
de catedra, la Universitatea din Bucuresti n specialitatea mecanica uidelor, calitate
n care a functionat pan
a la pensionarea sa n 1982. In perioada 1952 -1953 a avut
functia administrativa de prorector al Universitatii din Bucuresti.
Caius Iacob are contributii importante n domeniile mecanicii uidelor, analizei
matematice (ecuatii cu derivate partiale) si mecanicii generale. In jurul anilor 30
87

n aerodinamica se utiliza nca ipoteza uidului incompresibil, cu rezultate valabile


pentru viteze mici. Pentru uidele compresibile, studiile lui Caius Iacob sunt printre
primele cercetari n domeniul vitezelor mari. Unele dintre contributiile sale sunt legate
de importante rezultate ale matematicienilor rusi S.A. Ciaplghin si N.E. Jucovschi.
Matematicianul si mecanicianul italian F.G.Tricomi, parintele aerodinamicii transsonice, l considera pe Caius Iacob precursor al aerodinamicii transsonice moderne.
In 1940, Academia de S
tiinte din Paris i-a decernat Premiul ,,Henri de Parville,
la propunerea lui H. Villat, pentru rezultatele obtinute n dinamica uidelor compresibile. Un amplu si documentat tratat, Introducere matematic
a n mecanica uidelor,
Ed. Acad. Romane, 1952 (840 p.), care reprezinta o sinteza a studiilor sale, a primit
Premiul de Stat. In 1959 a aparut traducerea acestuia n limba franceza, mult amplicata, Introduction mathematique des uides, Gauthier-Villars, Paris (1288 p.), cu o
prefat
a semnata de H. Villat.
A fost preocupat de ridicarea calitatii si ecientei nvatamantului matematic romanesc, universitar si general, prin actul didactic direct, seminariile stiintice, ndrumarile generoase, conferinte etc., dar si prin publicistica bogata menita acestui scop.
Pentru studenti a publicat cursuri si tratate: Matematici superioare, Matematici clasice si moderne s.a. Pentru elevi si profesori a publicat carti si manuale, articole de
popularizare a matematicii sau de dezbatere a unor teme actuale ale nvatamantului:
Asupra unor conditii necesare transform
arii n sateliti ai P
am
antului a corpurilor
lansate de pe P
am
ant (G.M.,1948-49), Jucovschi, Ciaplghin, fondatori ai hidrodinamicii (G.M.-A, 1952) sau cartea Matematic
a aplicat
a si mecanic
a (Colectia profesorului de matematica), Ed. Acad. Romane, 1989.
Caius Iacob a participat la multe congrese stiintice n tara si strainatate:
Franta, Belgia, Polonia, Marea Britanie, Italia etc. si a conferentiat la multe universitati din lume. Contributiile sale au primit recunoasterea comunitatii internationale
a matematicienilor.
atii de S
tiinte Matematice (si Fizice) de la constiA fost vicepresedinte al Societ
tuirea sa (1949), pan
a la sfarsitul vietii. De asemenea, a fost n conducerea lialei
Bucuresti a S.S.M. ca vicepresedinte (1950-1981) si apoi ca presedinte.
In 1955 Caius Iacob a fost ales membru corespondent al Academiei Romane,
iar n 1963 a devenit membru titular. Din 1980 si pana la sfarsitul vietii sale a fost
presedintele Sectiei Matematice a acestui for academic. Academia Romana a hotarat
sa tipareasc
a n ntregime opera sa stiintica n doua volume; primul volum a aparut
deja n anul 2000.
A facut parte din primul Senat postdecembrist, ca reprezentant al PNT
CD-ului,
partid ce tocmai fusese renintat n mod legal.
Caius Iacob a ncetat din viata la 6 februarie 1992.
Activitatea sa stiintic
a este continuata de S
coala de mecanic
a romaneasca al
carei mentor a fost si de care este strans legat, rezultatele sale stiintice pastrandu-si
si astazi actualitatea si constituind surse de inspiratie pentru tinerele generatii de
mecanicieni si matematicieni.

Prof. dr. Adrian CORDUNEANU


88


Henri POINCARE
100 de ani de la disparitia sa

Jules Henri Poincar


e a fost unul dintre marii matematicieni francezi. Este considerat, totodata, unul dintre cei mai mari
matematicieni ai lumii. A fost un mare
zician (teoretician) si un mare losof al
stiintei. Este descris adesea ca ultimul
matematician universal, av
and n vedere
ca a excelat n aproape toate domeniile
matematicii cu adevarat importante, existente n perioada vietii sale relativ scurte (a
trait 58 ani!). Dupa moarte, a fost caracterizat ca: matematician, geometru, losof
si om de litere, poet al innitului, bard al
stiintei.
S-a nascut la 29 aprilie 1854 la Nancy
(Lorraine) n Franta, din Leon Poincare (profesor de medicina la Universitate) si
Eugenie (nascut
a Launois). Din familia Poincare au provenit si alti oameni importanti
pentru Franta. Raymond Poincare, var al lui Henri, a fost de mai multe ori prim
ministru, presedinte al Frantei n timpul primului razboi mondial si membru la
Academiei Franceze. Un alt var al lui Henri, frate cu Raymond, a ajuns nalt
functionar n administratia Universitatii.
Dupa o copilarie dicila, marcata de mbolnavirea de difterie, dar beneciind de o
ngrijire atent
a, precum si de o instructie speciala din partea mamei care s-a ocupat
de el cu devotament, a urmat scoala generala. In timpul licelului (din Nancy, acum
Liceul ,,Henri Poincare), frecventat timp de sapte ani, Poincare a fost unul dintre
cei mai buni elevi, castig
and n numeroase ocazii premiile ntai la concursuri generale
ntre copiii din liceele din ntreaga Franta. Profesorul sau de matematica l descria ca
un ,,monstru al matematicilor. Intre 1873-1875 a urmat Scoala Politehnica. In ciuda
unor dicultati de coordonare a miscarilor si a vederii slabe, Poincare citea enorm si
memora cu mare usurinta. Dupa absolvirea Scolii Politehnice, Poincare a continuat sa
studieze la Scoala de Mine si a profesat, pentru o perioda scurta, ca inginer de mine.
In acelasi timp si-a nceput studiile de doctorat si a obtinut doctoratul n matematici
la Universitatea din Paris n 1879. Subiectul tezei de doctorat era din teoria ecuatiilor
diferentiale. A predat, timp de doi ani, analiza matematica la Universitatea din Caen,
apoi, n 1881, a obtinut un post la Facultatea de stiinte din Paris. In 1886, Poincare
a fost numit ca profesor la catedra de Fizic
a matematic
a si probabilit
ati de la Sorbona
si, concomitent, profesor la S
coala Politehnic
a. In aceste doua locuri a profesat pana
la sfarsitul vietii, 17 iulie 1912. A avut printre studentii doctorali pe Louis Bachelier
(1900) si Dimitrie Pompeiu (1905).
89

Henri Poincar
e a lucrat si a obtinut rezultate profunde n mai multe ramuri
ale matematicilor: mecanica cereasc
a, mecanica uidelor, teoria relativitatii speciale
si losoa stiintei. Multe din cercetarile sale au implicat interactiuni substantiale
ntre diverse aspecte ale stiintelor naturii si au necesitat aplicatii multiple ale matematicii. Intelegerea profunda a matematicii si a stiintei timpului sau i-au permis sa
atace multe probleme din mai multe puncte de vedere. Domeniile matematicii n
care Poincare a avut contributii notabile sunt: topologia algebric
a (n principal n
ce priveste introducerea conceptului de grup fundamental ), teoria functiilor analitice
de mai multe variabile complexe, teoria functiilor abeliene, geometria algebric
a, formularea faimoasei conjecturi ce i poarta numele privitoare la structura homotopica
a sferei 3-dimensionale (conjectura rezolvata de G. Perelman n 2003), teorema de
recurent
a a lui Poincare, geometria hiperbolic
a, teoria numerelor, problema celor trei
corpuri, teoria ecuatiilor diofantice, teoria electromagnestismului, teoria relativitatii
speciale, teoria ecuatiilor diferentiale, unde a introdus conceptele de sfera Poincare si
de aplicatie Poincare, contributii la explicitarea legii normale a erorilor, argumentarea
matematic
a a mecanicii cuantice.
O descriere amanuntit
a a rezultatelor din domeniile enumerate mai sus ar necesita
mai mult spatiu si mai mult timp de citire. Vom enumera notiunile si rezultatele de
care este legat numele lui Poincare. Pe langa evocarea numelui lui Poincare n teoria
relativitatii speciale, mai putem sa mention
am: teorema PoincareBendixson (n teoria ecuatiilor diferentiale), teorema Poincare-Birkho-Witt, conjectura lui Poincare,
dualitatea Poincare, grupul lui Poincare (n topologia algebrica), modelul lui Poincare
al planului, metrica Poincare (n geometriile neeuclidiene). Alte notiuni legate de numele lui Poincare: teorema Poincare-Hopf, ecuatia Poincare-Lelong, meoda PoincareLindstedt, aplicatia Poincare, schita lui Poincare (Poincare plot - folosita azi, mai ales
n medicina, n dinamica simbolica a functionarii inimii), operatorul Poincare-Steklov,
simetria Poincare.
Vom puncta cateva din contributiile lui Poincare n domeniile enumerate mai
sus. In primul rand, avea preocupari speciale asupra modului de g
andire. Intr-o
lectie prezentat
a n 1908 la Institutul General de Psihologie din Paris, intitulata Descoperirea matematic
a si-a investigat propriile progese ale gandirii care au contribuit la
descoperirile sale n domeniul matematicii. Subiectul a fost aprofundat de E. Toulouse
n lucrarea [3] despre Poincare. O exemplicare a modului de gandire al lui Poincare
se gaseste n descrierea felului n care au fost descoperite functiile fuchsiene n 1908.
In nal, functiile respective au fost numite functii automorfe.
In cartea Analysis situs, publicata n 1895, Poincare prezinta o tratare sistematica
a topologiei combinatorii (algebrice). La preocuparile din acest domeniu, justica el,
a fost condus de cercetarile sale n domeniile ecuatiilor diferentiale si integralelor multiple. Poincare a introdus grupul fundamental n 1894 n legatura cu cercetarile unor
clase de suprafete (varietati 2-dimensionale n spatiu). Imediat a ajuns la conjectura
n legatur
a cu tipurile de homotopie ale varietatilor 3-dimensionale numita azi conjectura lui Poincare. Mai tarziu, problematica a fost abordata si n cazul varietatilor cu
mai multe dimensiuni si, n mod surprinzator, demonstratiile au ,,iesit mai repede
n cazul dimensiunilor mai mari decat trei.
A avut contributii majore n domeniul teoriei functiilor de mai multe variabile
90

complexe si a aplicat rezultatele n geometria algebrica, obtinand demonstratii simple si elegante pentru unele rezultate datorate matematicienilor E. Picard, F. Severi,
G. Castelnuovo si F. Enriques. In matematicile aplicate, a contribuit cu studii n
optic
a, electricitate, telegrae, capilaritate, elasticitate, termodinamic
a, teoria potentialului, teoria cuantic
a, teoria relativitatii si cosmologie. A studiat problema celor
trei corpuri si teoriile luminii si undelor electromagnetice. Poate considerat codescoperitor, mpreun
a cu H. Lorentz si A. Einstein, al relativitatii speciale. A mai avut
descoperiri majore n domeniul mecanicii ceresti prezentate n tratatul Metodele noi
ale mecanicii ceresti, aparut n trei volume ntre 1892 si 1899.
Mai trebuie mentionate si lucrarile de popularizare S
tiint
a si ipotez
a (1902), Valoarea stiintei (1905) si S
tiint
a si metod
a (1908). A mai avut contributii la losoa
matematicii si stiintei exprim
andu-si opiniile n legatura cu intuitionismul si apreciind
unele rezultate ale lui D. Hilbert si exprimand unele critici fata de cele ale lui B.
Russell.
A castigat numeroase premii, medalii si alte distinctii: Competitia Oscar II, regele
Suediei pentru rezolvarea problemei celor trei corpuri, premiul Societatii Filosoce
Americane, Medalia de aur a Societatii Regale Astronomice din Londra, premiul
Bolyai, medalia Mateucci, premiul Academiei Franceze de S
tiinte, premiul Academiei
Franceze, medalia Bruce etc. Numele sau este purtat de premiul Poincare (premiu
international n zica matematica), de Annales de l Institut Henri Poincare, Seminarul Poincare, craterul Poincare de pe Luna, asteroidul Poincare (nr. 2021).
In timpul vietii sale, Henri Poincar
e a primit numeroase recunoasteri ale geniului
sau. A fost ales n Academia de S
tiinte din Paris n 1887, iar n 1906 a fost ales ca
presedinte al acesteia. A fost singurul membru ales n ecare din cele cinci sectii ale
Academiei: Geometrie, Mecanica, Fizica, Geograe si Stiinte ale navigatiei. A mai
fost ales si membru n Academia Francez
a si a ajuns director al acestei Academii n
1912. A fost Cavaler al Legiunii de onoare si membru de onoare al multor societati
si academii de stiinte din ntreaga lume. A fost membru de onoare al Academiei
Romane.
Bibliografie
1. Henri Poincare, wikipedia http://en.wikipedia/wiki/Henri Poincar
e.
2. Jules Henri Poincare, Indexes of biographies, www.history.mcs.st-and.ac.uk
3. E. Toulouse Henri Poincare, Paris, 1910.
4. V. Oproiu Conjectura lui Poincare, Recreatii matematice, an IX , 2, (2007), 79-80.
5. L. Mihalcea Henri Poincare, Referat 27.03.2005, www.referat.ro

Prof.dr. Vasile OPROIU


Directorul Seminarului ,,A. Myller din Iasi

91

Multimi de puterea continuului n spatii euclidiene

Temistocle BIRSAN

Abstract. The aim of this Note consists in estabilishing, in an elementary and constructive way,
that certain usual sets in R or Rn (n 2) have the power of continuum; the bijections f : A R
that are employed belong to the mathematical folklore.
Keywords: bijection, cardinal number, power of continuum.
MSC 2010: 97E60.

1. Introducere. Creatorul teoriei multimilor este Georg Cantor (1845-1918),


care a fundamentat si dezvoltat aceasta teorie ntr-un numar de memorii publicate la
sfarsitul sec. al XIX-lea. El utilizeaza conceptul de functie bijectiv
a (Corespondent
a
biunivoc
a sau unu-unu) pentru a opera o ierarhizare a multimilor innite. Introduce
notiunile de num
ar cardinal si num
ar ordinal si construieste o aritmetica a acestora.
Importanta lucrarilor lui G. Cantor a fost recunoscuta n secolul urmator, iar teoria
multimilor patrunde n toate ramurile matematicii si n nvatamantul general ncepand
cu treptele sale inferioare.
Se spune ca o multime A din Rn este num
arabil
a daca exista o bijectie ntre A si
N si ca este de puterea continuului daca exista o bijectie ntre A si R. In al doilea caz,
se mai spune ca multimea A are cardinalul c si se noteaza card A = c.
In cele ce urmeaza, ne vom limita la cateva considerente asupra multimilor de puterea continuului, care au tangenta cu preocuparile elevilor si profesorilor. Mai precis,
vom arata ca anumite multimi ,,uzuale din R si Rn au puterea continuului indicand
cate o functie reala bijectiva denita pe ecare dintre ele (si evitand modalitatile
neconstructiviste).
2. Cardinalul intervalelor lui R. Amintim ca intervalele proprii ale lui R sunt:
1) m
arginite: (a, b), (a, b], [a, b) si [a, b]; 2) nem
arginite: (a, +), (, a), [a, +) si
(, a], unde a, b R, a < b. Vom arata ca au cardinalul c construind o bijectie
ntre un astfel de interval (indiferent de ce tip ar !) si R .
S
a observam mai nt
ai ca functia liniara : R R denita prin
(1)

(t) =

1
(t a), t R,
ba

transforma intervalele (a, b), (a, b], [a, b) si [a, b] n (0, 1), (0, 1], [0, 1) si, respectiv,
[0, 1], iar translatia : R R :
(t) = t a, t R,

(2)

transforma semidreptele (a, +), (, a), [a, +) si (, a] n (0, +), (, 0),


[0, +) si, respectiv, (, 0].
Propozitia 1. Urm
atoarele armatii sunt adev
arate:
1 Prof.

dr., Univ. Tehnic


a ,,Gh. Asachi, Iasi

92

1) intervalele (0, 1), (0, 1], [0, 1) si [0, 1] au cardinalul c;


2) pentru orice a, bR, a<b, intervalele (a, b), (a, b], [a, b) si [a, b] au cardinalul c.
Demonstratie. 1) Solutia I. Evident, functia f1 : (0, 1) R denita prin
(3)

f1 (x) = tg

(2x 1), x (0, 1)


2



(obtinuta ,,aranjand tg : ,
R) este bijectiva; deci card (0, 1) = c.
2 2

1 1 1
Introducand multimea A = (0, 1)
, , , . . . , putem scrie egalitatile:
2 3 4

1 1
1 1
1 1
(0, 1] = A 1, , , . . . , [0, 1) = A 0, , , . . . , [0, 1] = A 0, 1, , , . . . .
2 3
2 3
2 3
Se constata imediat ca functiile f2 : (0, 1] (0, 1), f3 : [0, 1) (0, 1), f4 : [0, 1]
(0, 1) denite, respectiv, prin
8
<x,

(4)

f2 (x) =

xA
1
1
:
, x= ,
k+1
k

8
>x,
>
<1

(5)

f3 (x) =

xA

x=0
1
, x= ,
k+1
k

>2
> 1
:
8
>x,
>
>1
> ,
<

(6)

f4 (x) =

k N ,

k {2, 3, . . .},

xA

x = 0,
2
1
> ,
x=1
>3
>
> 1
1
:
, x= ,
k+2
k

k {2, 3, . . .}

sunt bijective; ca urmare, card(0, 1] = card[0, 1) = card [0, 1] = c.

am sirul
Solut
ia II. Vom construi n alt mod o bijectie f2 : (0, 1] (0, 1). Consider

1
2k

care tinde la 0 (sau oricare alt sir strict descrescator la 0). Avem:
kN

(0, 1] =


[
1
k=1

1
, k1
k
2 2

Se veric
a usor ca functia fk :

si (0, 1) =


[
1

1
, k1
k
2 2

k=1

1
1
1
1
, k1 k , k1
k
2 2
2 2

denita prin fk (x) =

3
x,
2k

1
1
,
, este o bijectie. Ca urmare, aceeasi proprietate o are si functia
2k 2k1
93

f2 : (0, 1] (0, 1) data de


(7)

f2 (x) = fk (x), daca x

1
1
,
.
2k 2k1

In aceeasi masur
a se poate proceda pentru a indica o bijectie f3 : [0, 1) (0, 1)
(se porneste cu un sir de numere pozitive strict crescator la 1).
2) Obisnuit, notam cu X restrictia functiei data de (1) la multimea X R.
Din consideratiile precedente rezulta ca functiile F1 : (a, b) R, F2 : (a, b] R,
F3 : [a, b) R si F4 : [a, b] R denite prin F1 = f1 (a,b) , F2 = f2 (a,b] ,
F3 = f3 [a,b) si, respectiv, F4 = f4 [a,b] sunt bijective (compuneri de bijectii).
Deci card(a, b) = card (a, b] = card [a, b] = c. Propozitia este complet demonstrata.
Propozitia 2. Pentru orice a R, semidreptele (a, +), (, a), [a, +) si
(, a] au puterea continuului.
Demonstratie. Apeland la translatia (2), am putea reduce semidreptele din
enunt la cazul n care a = 0; nu vom obtine avantaje esentiale, motiv pentru care
renunt
am.
Este evident ca functiile G1 : (a, +) R si G2 : (, a) R denite prin
G1 (x) = ln(x a), x > a si G2 (x) = ln(a x), x < a, sunt bijectii.
In privinta semidreptelor nchise [a, +) si (, a], pentru constructia de bijectii
ntre acestea si R se poate urma ecare dintre cele doua cai indicate n solutiile de mai
sus. Astfel, daca avem n vedere semidreapta [a, +), notam A = (a, +){a+1, a+
2, . . .} si scriem [a, +) = A {a, a + 1, a + 2, . . .} si (a, +) = A {a + 1, a + 2, . . .}.
Constructia unei bijectii g : [a, +) (a, +) este evidenta si obtinem n cele din
urma o bijectie G3 : [a, +) R prin compunerea G3 = G1 g.
Sau, procedand pe a doua cale, scriem [a, +) = [a, a + 1) [a + 1, a + 2) . . .
si (a, +) = (a, a + 1] (a + 1, a + 2] . . ., apoi realizam pentru ecare k N cate
o bijectie gk : [a + k, a + k + 1) (a + k, a + k + 1] prin gk (t) = (2a + 2k + 1) t,
t [a + k, a + k + 1), obtinem o bijectie g : [a, +) (a, +). In nal, functia
G3 : [a, +) R, data de G3 = G1 g , este bijectie. Cu semidreapta (, a] se
procedeaza similar. Demonstratia este ncheiata.
Corolar. Intervalele dreptei reale, ca si R ns
asi, au cardinalul c.
3. Cardinalul unor intervale, discuri si coroane din R2 . Sa trecem de la
spatiul unidimensional R la Rn , n 2. Consideratiile urmatoare se vor referi numai
la R2 , trecerea la Rn , n 3, facandu-se far
a dicultate.
In planul R2 , corespunzatorul unui interval (a, b) = {x; a < x < b} din R este
atat intervalul
at si coroana circulara
p bidimensional {(x, y); a < x < b, c < y < d}, c
{(x, y); a < x2 + y 2 < b} (care, n particular, poate un disc). Vom stabili mai
jos ca intervalele (bidimensionale), discurile si coroanele circulare - deschise, nchise,
seminchise - ca si spatiul R2 au cardinalul c.
Asadar, n spiritul acestei note, vom arata cum se poate construi o bijectie ntre
oricare dintre aceste multimi plane si R .
Date numerele a, b, c, d R cu a < b si c < d, avem sase tipuri de intervale
bidimensionale {(x, y); a < x < b, c < y < d}, {(x, y); a x b, c y d},
94

{(x, y); a x < b, c y < d}, {(x, y); a < x b, c < y d}, {(x, y); a x < b, c <
y d}, {(x, y); a < x b, c y < d}.
Propozitia 3. 1) Dac
a I 2 noteaz
a un interval bidimensional, atunci F : I 2 R
bijectiv
a.
2) G : {(x, y); 0 x < 1, 0 y < 1} [0, 1) bijectiv
a.
3) Intervalele bidimensionale au cardinalul c.
Demonstratie. S
a luam, de exemplu, I 2 = {(x, y); a < x < b, c < y < d}. Avem
I = (a, b) (c, d). Conform Propozitiei 1, f : (a, b) R si g : (c, d) R bijective.
Atunci, functia F : I 2 R2 denit
a prin F (x, y) = (f (x), g(y)), (x, y) I 2 este
bijectiva.
2) Fie (x, y) un punct oarecare al patratului P = {(x, y); 0 x < 1, 0 y < 1}.
Coordonatele x si y se pot scrie sub forma de fractii zecimale innite (convenim
ca de la un loc ncolo sa nu apara numai cifra 9). Grupam zecimalele numerelor
x si y astfel: ecare grupa contine o singura cifra diferita de 9 care ncheie grupa
(de exemplu, x = 0, 94709913958 . . . are grupele: 94, 7, 0, 991, 3, 95, 8, . . .). Pentru x
obtinem sirul de grupe g1 , g2 , . . ., iar pentru y sirul g1 , g2 , . . . Fie t numarul zecimal
avand sirul de grupe g1 , g1 , g2 , g2 , . . . Functia G : P [0, 1) denita prin G(x, y) = t
este bijectie.
3) Armatia este o consecint
a a punctelor precedente si a punctului 1) al Propozitiei
1: succesiunea I 2 R2 {(x, y); 0 x < 1, 0 y < 1} [0, 1) R sugereaza
bijectiile ce trebuie compuse pentru a obtine o bijectie de la I 2 la R. Asadar, card
I 2 = c.
2

Corolar. Exist
a o bijectie ntre Rn si R, i.e. card Rn = c.
Demonstratie. Prin inductie, pe baza faptului ca Rn = Rn1 R, n 2.
Observatie. O bijectie ntre R2 si R poate interpretata astfel: pozitia unui
punct n plan este data de o singura coordonata. Vom arata, nsa, ca o bijectie de la
R2 la R nu este functie continu
a; bijectia carteziana atribuie punctului din plan doua
coordonate, dar functiile-proiectie pe axe sunt continue. Intr-adevar, sa presupunem,
prin absurd, ca ar exista o functie : R2 R bijectiva si continua (pe R2 si R
se considera metricele euclidiene). Axa x-lor, notata X, este multime conexa n R2 .
Imaginea sa prin , notata X , este o parte conexa a lui R, deci un interval. Axa
Y -lor, notata Y , este conexa n R2 si imaginea sa, Y , este un interval cu un singur
punct comun cu X . Evident, acest punct nu poate n interiorul intervalului X , ci
extremitate a lui. Alte doua drepte Y1 si Y2 paralele cu Y , distincte de Y si ntre ele,
au ca imagine prin intervalele Y1 si Y2 , ecare avand n comun cu X o extremitate
a acestuia. Absurd, ntruc
at Y , Y1 si Y2 sunt disjuncte.
Vom considera numai discuri si coroane circulare cu centrul n origine, celelalte
putand aduse n aceasta situatie printr-o translatie. Pentru coroane utilizam notatiile:
C(a, b) = {(x, y); a2 < x2 + y 2 < b2 },

C[a, b) = {(x, y); a2 x2 + y 2 < b2 },

C(a, b] = {(x, y); a2 < x2 + y 2 b2 } si C[a, b] = {(x, y); a2 x2 + y 2 b2 },


95

unde a, b R, 0 a < b, sunt razele coroanei. Pentru a = 0, C[0, b) este discul


deschis D[0, b) = {(x, y); x2 + y 2 < b2 }, iar C(0, b) este discul deschis punctat (i.e,
far
a centru) D (0, b) = {(x, y); 0 < x2 + y 2 < b2 }; similar, D[0, b] si D (0, b] noteaza
discurile nchis si punctat nchis de raza b.
Lema 1. Prin transformarea
8
>x =
<

(8)

1
ad bc
(d c) p 2
x,
ba
x + y2 
ad bc
1
>
:y =
(d c) p 2
y,
ba
x + y2

coroana de raze a, b trece n coroana de raze c, d p


atrunz
and n acelasi timp tipul (de
ex., C(a, b] trece n C(c, d] etc.).
Demonstratie. Punctului P (x, y) facem sa-i corespunda punctul P (x , y ) ncat:
1) P OP , unde O noteaza originea (0, 0); 2) segmentul C(a, b] OP trece n
segmentul C(c, d]OP printr-o transfomare liniara L : = + ce verica conditiile
L(a) = c p
si L(b) = d (evident, coordonata pe OP este distanta la originea O,
adica = x2 + y 2 . Conditia 2) conduce imediat la urmatoarea expresie pentru L:
dc
ad bc
y
y
=

, iar conditia 1) revine la = . Rezolvand sistemul acestor


ba
ba
x
x
doua ecuatii n x , y obtinem (8).
Lema 2. Prin transformarea

(9)



8
>x = pa + b
1 x,
<
2
2

 x +y
a+b
>
:y = p
1 y,

x2 + y 2

imaginea coroanei C[a, b) este coroana C(a, b].


Demonstratie. Urm
am demonstratia Lemei 1, cu modicarea urmatoare: coecientii
si din conditia 2) se vor determina din conditiile L(a) = b si L(b) = a. In nal se
obtine transformarea (9).
Propozitia 4. 1) Exist
a o bijectie ntre orice tip de disc sau coroan
a circular
a si
R2 .
2) Cardinalul oric
arui disc sau coroane circulare este c.
Demonstratie. 1) Conform Lemei 1, putem construi o bijectie ntre doua coroane
de acelasi tip, indiferent de razele lor. Lema 2 indica o bijectie ntre coroanele de tip
diferit C[a, b) si C(a, b] av
and aceleasi raze.
Cu procedeul din Propozitia 1, Solutia II, vom indica o bijectie ntre C(a, b] si
C(a, b). Intr-adev
ar, e (ak )kN sirul strict descrescator la a denit prin ak = a +

S
S
ba
,
k

N.
Avem
C(a,
b]
=
C(a
,
a
]

s
i
C(a,
b)
=
C[ak , ak1 ). Conform
k
k1
2k
k=1
k=1
Lemei 2, pentru orice k N putem indica o bijectie Fk : C(ak , ak1 ] C[ak , ak1 )
96

si apoi denim bijectia F : C(a, b] C(a, b) prin F (x, y) = Fk (x, y) daca (x, y)
C(ak , ak1 ].
Cu acest ultim rezultat si tin
and seama de relatiile C[a, b] = C(a, b] {(x, y)x2 +
2
y = a} si C[a, b) = C(a, b) {(x, y); x2 + y 2 = a2 } stabilim o bijectie ntre coroanele
C[a, b] si C[a, b).
Stabilim cum se poate construi o bijectie ntre discul D(0, a) si discul punctat
D (0,a) 
C(0, a). Procedam ca n Propozitia 1, Solutia I. Fie (ak )k2 sirul dat de
a a
,
, k 2, sir inclus n D (0, a). Notam A = D (0, a) {a2 , a2 , . . .}. Avem
ak =
k k
D(0, a) = A {0, a2 , a3 , . . .} si D (0, a) = A {a2 , a3 , . . .} si constructia bijectiei
dorite este evident
a.
Se constata usor ca o bijectie ntre D(0, 1) si R2 este realizata prin transformarea
8
x

,
<x = p
1

x2 y 2
(10)
y
:y = p
,
1 x2 y 2
ceea ce ncheie demosntratia punctului 1).
2) Armatia rezulta pe baza precedentelor si relatiei card R = c.
4. Comentariu final. In Sectiunile 2 si 3 au fost considerate multimi cu o forma
geometric
a simpla. Nu nt
ampl
ator, n constructia bijectiilor cu care s-a aratat ca ele
au puterea continuului, s-au folosit cunostinte de geometrie. Se ntelege ca, pentru
o multime M complicat
a, constructia unei bijectii f : M R este anevoioasa sau
imposibila. Teoria multimilor ofera alte instrumente puternice de lucru ce permit
stabilirea cardinalului unei multimi ([1], [2]). De exemplu, dac
a multimile A, B, C
sunt astfel nc
at A B C si card A = card C, atunci avem si card B = card A =
card C. Aceste armatii sunt ilustrate de
Propozitia 5. Multimea numerelor irationale din intervalul (0, 1) are puterea
continuului.
Demonstratie. Not
am cu Q1 si I1 multimile numerelor rationale si, respectiv,
irationale din intervalul (0, 1]. Stim ca Q1 este numarabila.

Avem I1 = (I1 Q1 ) {x; x Q1 , x } {x; x Q1 , < x } si


2
2
(0, 1] = (I1 Q1 ) Q1 Q1 , unde I1 este un numar irational luat n mod
arbitrar, iar Q1 = {r; r Q1 }. Se verica usor ca functia f : I1 (0, 1] denita

prin f (x) = x dac


a x I1 Q1 , f (x) = 2x daca x Q1 si x , f (x) = 2x 1
2

daca x Q1 si
< x este o bijectie. Cum card (0, 1] = c (Propozitia 1),
2
rezulta ca avem si card I1 = c.
Bibliografie
1.

O. Constinescu, C. Amih
aesei, T. Brsan Topologie general
a. Probleme,
E.D.P., Bucuresti, 1974.
2. K. Kuratowski Introducere n teoria multimilor si n topologie, Ed. Tehnica,
Bucuresti, 1969.
97

Cum putem folosi ntregii algebrici n matematica


elementar
a
Marian TETIVA1
Abstract. The paper brings some tools from advanced algebra (namely algebraic integers) in
attention of those interested in elementary, but harder problems, like those involved in mathematical
competitions. Yet, the author would like to try and open their eyes, so that they can see, beyond
their immediate goals, the higher and greater mathematics which is expecting them to be explored,
known, and understood.
Keywords: rational number, complex number, algebraic integer, symmetric polynomial.
MSC 2010: 97H99.

Este vorba, mai precis, de probleme care au un enunt elementar, dar care se
pot rezolva (uneori) numai recurgand la o notiune pe care n-o prevad programele
matematicii scolare; sau pentru care aceasta notiune (de ntreg algebric) simplifca
mult solutia. V-ati nt
alnit, pesemne, cu
Problema 1. S
a se arate c
a, dac
a r este un num
ar rational si cos(r) este, de
asemenea, num
ar rational, atunci cos(r) poate doar 1, 1/2, 0, 1/2 sau 1.
Solutie (a se vedea [3] sau [4]). Fie r = m/n, cu m si n numere ntregi si
= cos(r) + i sin(r). Avem 2 cos(r) = + si 2n = 1. Inseamna ca si sunt
solutii ale ecuatiei x2n 1 = 0, deci sunt ntregi algebrici; odata cu ele va ntreg
algebric si suma lor, adica 2 cos(r). Pe de alta parte, ipoteza ne spune ca 2 cos(r)
este num
ar rational, iar un numar rational care este si ntreg algebric este n mod
necesar un ntreg. Deci 2 cos(r) Z [2, 2], ceea ce conduce direct la concluzie.
Pentru cei interesati de mai mult n privinta aceasta avem si
Problema 2. Pentru r num
ar rational singurele valori posibile ale lui cos(r)
care sunt si irationale p
atratice (adic
asolutii
irationale ale unei
ecuatii de gradul al
doilea cu coecienti rationali) sunt 2/2, 3/2 si (1 5)/4.
Puteti gasi un material destul de elaborat la adresa
http://www.uni-math.gwdg.de/jahnel/Preprints/cos.pdf pentru a aa mai multe despre aceste chestiuni. Noi vom dezvolta subiectul n alta directie.
Trebuie, binenteles, sa ncepem prin a clarica solutia de mai sus: ne putem
lamuri destul de repede, cu o denitie si doua teoreme (ce-i drept, una dintre ele mai
avansat
a, pentru care demonstratia, eventual, se ocoleste; asa cum vom face si noi)
ca nu e vorba de lucruri foarte grele. (Chiar daca vom sari peste unele demonstratii,
o facem pentru a nu aglomera expunerea, iar nu pentru ca ele ar inaccesibile.)
Definitie. Un num
ar complex se numeste ntreg algebric daca este solutie a
unei ecuatii algebrice cu coecienti ntregi si coecientul termenului de grad maxim
egal cu 1; altfel spus, daca exista numerele ntregi a0 , a1 , . . . , an1 astfel ncat
n + an1 n1 + + a1 + a0 = 0.
1 Profesor,

Colegiul National ,,Gheorghe Rosca Codreanu, B


arlad

98

Teoremele sunt urmatoarele:


Teorema 1. Dac
a si sunt ntregi algebrici, atunci + , si sunt,
de asemenea, ntregi algebrici.
Asadar, multimea ntregilor algebrici are o structura de inel (mai precis este subinel
al inelului numerelor complexe). Nu prezentam aici demonstratia acestei teoreme,
neind spatiu. Altminteri ea nu este asa complicata (dar nici evidenta), poate
nteleas
a de catre un elev de liceu (din an terminal). Cei interesati pot consulta
demonstratia n [3].
Teorema 2. Un num
ar rational care este si ntreg algebric este ntreg.
Demonstratie. Fie = p/q, cu p si q numere ntregi prime ntre ele, q > 0.
Avem pentru o egalitate ca n denitia de mai sus, care ne conduce la
pn + an1 pn1 q + + a1 pq n1 + a0 q n = 0
(cu a0 , . . . , an1 numere ntregi); de aici vedem ca q divide pe pn . Dar q si pn sunt
prime ntre ele (q si p ind astfel), deci q = 1 si = p Z.
Rezultatul acesta are sigur un aer cunoscut; chiar daca acum e cuprins abia n
programa clasei a XII-a, l stiti, probabil, n urmatoarea forma mai generala:
Teorema 2 . Fie x = p/q, cu p si q numere ntregi prime ntre ele, o solutie
rational
a a ecuatiei algebrice cu coecienti ntregi an xn +an1 xn1 + +a1 x+a0 = 0.
particular, dac
Atunci p divide pe a0 si q divide pe an . In
a an = 1 si a0 {1, 1},
singurele (eventuale) solutii rationale ale ecuatiei sunt 1 sau 1.
Cred ca puteti demonstra Teorema 2 si cred ca acum poate deplin nteleasa
rezolvarea Problemei 1!?
Si, pentru ca am pornit pe acest drum, dati-mi voie sa va port spre o alta solutie
a Problemei 1, trecand iar printr-o teorema neelementara (oare?). Ea se numeste
teorema fundamental
a a polinoamelor simetrice, deci trebuie sa mai dam o
Definitie. Un polinom n n nedeterminate f (X1 , X2 , . . . , Xn ) se numeste simetric
daca
f (X(1) , X(2) , . . . , X(n) ) = f (X1 , X2 , . . . , Xn )
oricare ar permutarea a multimii {1, 2, . . . , n} (deci daca este invariant la orice
permutare a nedeterminatelor).
Se vorbeste putin despre polinoame simetrice n matematica de liceu (si cred ca e
regretabil acest lucru); ceva n clasa a IX-a (cand se fac sisteme de ecuatii simetrice doar cu doua necunoscute, deci si polinoamele implicate sunt cu doua nedeterminate)
si apoi abia n clasa a XII-a cand, pentru a da relatiile ntre radacinile si coecientii
unei ecuatii algebrice (relatiile lui Vi`ete) trebuie introduse polinoamele simetrice fundamentale n n nedeterminate, adica polinoamele
sj =

Xi1 Xij , 1 j n;

1i1 <<ij n

99

sj este, cu alte cuvinte, suma tuturor produselor de cate j nedeterminate (s1 =


X1 + + Xn este suma nederminatelor, sn = X1 Xn este produsul lor si asa
mai departe). Se mai nvata atunci (consecutiv relatiilor lui Vi`ete) diverse exprimari
cum ar X12 + X22 + + Xn2 = s21 2s2 , sau X12 X22 + X12 X32 + X22 X33 = s22 2s1 s3
(aici e vorba de s1 , s2 , s3 n trei variabile), etc. Enuntul urmator arata ca asemenea
exprimari nu sunt deloc ntamplatoare.
Teorema 3 (Teorema fundamental
a a polinoamelor simetrice). Pentru orice polinom f (X1 , X2 , . . . , Xn ) simetric n nedeterminatele X1 , X2 , . . . , Xn exist
a un polinom
g(X1 , X2 , . . . , Xn ) astfel nc
at
f (X1 , X2 , . . . , Xn ) = g(s1 , s2 , . . . , sn ).
De exemplu, pentru f = X12 + X22 + + Xn2 , avem pe g = X12 2X2 . Nu intram
nici n detaliile acestei demonstratii, o puteti gasi n orice curs de algebra superioara
(de exemplu [2]).
Pentru noi e important doar sa observam ca are loc urmatoarea
Consecint
a. Fie f un polinom cu coecienti ntregi si x1 , x2 , . . . , xk toate r
ad
acinile sale (n general complexe). Fie g un alt polinom cu coecienti ntregi si e h
polinomul denit prin
h(X) = (X g(x1 ))(X g(x2 )) (X g(xk )).
Atunci h are coecienti ntregi (si, evident, coecientul termenului de grad maxim
egal cu 1).
Folositi relatiile lui Vi`ete pentru polinomul f si teorema fundamentala a polinoamelor simetrice pentru a justica aceasta consecinta!
A doua solutie a Problemei 1. Sa ncepem tot prin a observa ca, daca r = m/n
(cu m si n ntregi), atunci 2 cos(r) = 2 cos(m/n) = m +1/ m , unde = cos(/n)+
i sin(/n), deci 2 cos(m/n) este radacina a polinomului
h=

2n1
Y
j=0

X j +

1
j

2n1
Y

X j + ( j )2n1

j=0

care are coecienti ntregi (conform consecintei imediat mai sus mentionate a teoremei
fundamentale a polinoamelor simetrice, nu?) si coecientul termenului de grad maxim
egal cu 1. Termenul sau liber

2n1
Y
1
j + j

j=0
este, daca l presupunem pe n impar, egal cu 4 (vericati acest fapt! a se vedea si
[4]). Cum doi dintre factorii produsului sunt 2 si 2 (pentru j = 0, respectiv j = n),
ram
ane

Y
1
j + j = 1

j{0,...,2n1}\{0,n}

100

deci polinomul

X j +

j{0,...,2n1}\{0,n}

1
j

(obtinut prin mp
artirea lui h la X 2 4; din algoritmul mpartirii vedem ca are
coecienti ntregi si coecientul termenului de grad maxim egal cu 1) poate avea,
pentru n impar, ca rad
acini rationale doar pe 1 sau 1. Altfel spus, daca numitorul
lui r este impar si cos(r) este rational, atunci cos(r) {0, 1/2, 1} (de fapt 0 nu
poate n acest caz).
Cazul cand r are numitorul par se reduce la acesta, folosind formula cos 2x =
2 cos2 x 1 si apeland, eventual, la o mica inductie. Intr-adevar, daca stim cos(r)
Q, rezulta si cos(2r) = 2 cos2 (r) 1 Q. Daca 2r are numitorul impar, conform
celor deja demonstrate, cos(2r)
poate doar 1, 1, 1/2 sau 1/2 si vericam usor ca

rezulta pentru cos(r) = (1 + cos(2r))/2 (si cos(r) Q) doar una din valorile
din multimea {0, 1/2, 1}. Daca nu (daca 2r nca are numitorul par), mergem la
cos(4r), care este, de asemenea, rational si asa mai departe.
Problema 3. Ar
atati c
a, dac
a r si tan(r) sunt numere rationale (astfel nc
at
2r s
a nu e num
ar ntreg impar), atunci tan(r) {1, 0, 1}.
Evident, provocarea aici este sa gasiti o solutie directa, care nu se foloseste de
Problema 1 (dar asta numai pentru a exersa metoda; altminteri solutia bazata pe
rezultatul deja demonstrat este cea care se da de obicei [3,4]). Se poate ncerca
o inductie dupa numitorul lui r (atunci cand acesta e scris ca fractie ireductibila cu
numitor pozitiv) folosind faptul ca tan(nr) = 0 si formula pentru tan(nx). Observati
ca Teorema 2 se aplica usor daca n (numitorul lui r) este prim.
Acum ne vom muta (aparent) n alta zona cu
Problema 4. Fie a, b si c numere ntregi nenule astfel nc
at
u=

a b
c
b
c a
+ +
si v = + +
b
c a
a b
c

sunt, si ele, numere ntregi. S


a se arate c
a |a| = |b| = |c|.
Solutie. Parc
a nu seaman
a cu cele dinainte - dar se rezolva n acelasi cerc de
idei. Am (re)nt
alnit de curand problema pe forumul MathLinks (multumim!) si cred
ca e unul din punctele de pornire ale acestei note, alaturi de dorinta de a prezenta
aceasta metoda de rezolvare a unor probleme, mai ales pentru ca deschide ferestre
catre matematica mai nalt
a (si, daca va place matematica, trebuie sa priviti prin
asemenea ferestre). Probabil ca deja v-ati dat seama cum stam: numerele x1 = a/b,
x2 = b/c si x3 = c/a sunt rationale si radacini ale polinomului (cu coecienti ntregi
si coecientul termenului de grad maxim egal cu 1)
(X x1 )(X x2 )(X x3 ) = X 3 uX 2 + vX 1
(caci x1 + x2 + x3 = u, x1 x2 + x1 x3 + x2 x3 = v si x1 x2 x3 = 1; iar ntalnim relatiile lui
Vi`ete). Orice rad
acin
a rational
a a sa este, deci, 1 sau 1, de unde concluzia decurge
imediat.
101

Iat
a, ca de obicei, nc
a niste probleme pentru dumneavoastra. Prima o sa vi
se para, poate, o gluma - dar ganditi-va ca n-ati stiut Problema 4 (altfel se vad
lucrurile, nu?). La fel putem spune despre cea de-a doua problema propusa, daca
stim Problema 1.
Problema 5. Fie a, b, c si d numere ntregi nenule astfel nc
at
a
b
c
d
+ + +
si
b
a d
c

a
b
+
b
a

c
d
+
d
c

sunt numere ntregi. S


a se arate c
a |a| = |b| si |c| = |d|.
Problema 6. Fie r si q numere rationale astfel nc
at = cos r + cos q este
num
ar rational. S
a se arate c
a {2, 3/2, 1, 1/2, 0, 1/2, 1, 3/2, 2}. Puteti
extinde pentru o sum
a de trei (patru, etc) cosinusuri?
Problema 7. Fie s, m si n numere ntregi, s > 0 si n impar. S
a se arate c
a,
dac
a (cos(m/n))s este num
ar rational, atunci (cos(m/n))s {1, 1/2s }.
Problema 8 (Marius Cavachi [1]). Fie n un num
ar natural mai mare ca 1 si
diferit de 4. Fie p si q numere ntregi pozitive mai mici dec
at n si prime cu n. Fie
cos(2p/n)
a =
. S
a se arate c
a, dac
a ak este rational pentru un ntreg pozitiv k,
cos(2q/n)
atunci ak este egal ori cu 1, ori cu 1.
Bibliografie
1. M. Cavachi Problem 11540, The American Mathematical Monthly, 10/2010, p.
929.
2. I. D. Ion, N. Radu Algebra, Editura Didactica si Pedagogica, Bucuresti, 1991.

3. M. T
ena Intregi
algebrici si aplicatii n Zece lectii alese de matematic
a elementar
a,
Societatea de Stiinte Matematice din Romania, 1998.
4. M. T
ena R
ad
acinile unit
atii, Societatea de Stiinte Matematice din Romania, 2005.

1. Diagonale si triunghiuri. Completati tabelul de mai jos cu cel putin dou


a
coloane:
n 3 4 5 ...
d 0 2 5 ...
t 1 8 31 . . .
unde n = num
arul laturilor unui poligon, d = num
arul diagonalelor sale si t =
num
arul triunghiurilor formate de laturile si diagonalele sale.
2. Ce liter
a urmeaz
a n sirul U, I, I, U, I, E, E, T, . . .?
Lucian Tutescu, Craiova
(R
aspunsuri la pag. 105)
102

Conexiuni cu cercul celor sase puncte


Constantin DRAGOMIR1
Abstract. A problem proposed at the OIM, Madrid 2008, is generalized in this Note by substituting the circumcenter and the orthocenter of a triangle for two arbitrary isogonal conjugate
points.
Keywords: orthocenter, circumcenter, isogonal conjugate points.
MSC 2010: 51M04.

Punctul de plecare al acestei note este urmatoarea problema, propusa de delegatia


Rusiei la a 49-a Olimpiad
a International
a de Matematic
a, Madrid, 2008 [2,p.352]:

Intr-un
triunghi ascutitunghic ABC se noteaz
a cu H ortocentrul s
au. Cercul cu
centrul n mijlocul segmentului BC si care trece prin H intersecteaz
a dreapta BC
n punctele A1 si A2 . Analog, cercul cu centrul n mijlocul segmentului CA si care
trece prin H intersecteaz
a dreapta CA n punctele B1 si B2 , iar cercul cu centrul n
mijlocul segmentului AB si care trece prin H intersecteaz
a dreapta AB n punctele
C1 si C2 . Ar
atati c
a punctele A1 , A2 , B1 , B2 , C1 , C2 sunt conciclice.
Dupa cum se stie, cercul lui Euler (cercul celor nou
a puncte) este cercul circumscris
triunghiurilor podare ale punctelor O si H, centrul sau ind mijlocul segmentului OH.
Se mai stie ca punctele O si H sunt izogonale.
O generalizare, utila n cele ce urmeaza, este data de
Propozitia 1 [1; 5.16, p.57]. Proiectiile a dou
a puncte izogonale P si Q pe laturile
unui triunghi sunt sase puncte conciclice, iar cercul acestor proiectii are centrul n
mijlocul segmentului P Q (cercul celor sase puncte).
Se observa ca n problema de olimpiada enuntata mai sus apar punctele H si O
(prin mijloacele laturilor), dar nu n mod simetric.
Propunem o generalizare a problemei de felul trecerii de la cercul lui Euler la
cercul celor sase puncte si o restabilire a simetriei n
Propozitia 2.
In triunghiul ABC se consider
a dou
a puncte izogonale P si Q si se
noteaz
a cu Pa , Pb , Pc si respectiv Qa , Qb , Qc proiectiile lor pe dreptele BC, CA si AB.
Cercurile C(Pa , Pa Q), C(Pb , Pb Q) si C(Pc , Pc Q) intersecteaz
a BC, CA, respectiv AB
n perechile de puncte A1 si A2 , B1 si B2 , respectiv C1 si C2 , iar cercurile C(Qa , Qa P ),
C(Qb , Qb P ) si C(Qc , Qc P ) intersecteaz
a BC, CA, respectiv AB n perechile de puncte
A3 si A4 , B3 si B4 , respectiv C3 si C4 .
Ar
atati c
a punctele A1 , A2 , B1 , B2 , C1 , C2 sunt conciclice, iar cercul acestor puncte
are centrul n P si c
a punctele A3 , A4 , B3 , B4 , C3 , C4 sunt conciclice si situate pe un
cerc de centrul Q si de aceeasi raz
a.
Demonstratie. Fie q lungimea segmentului P Q si M mijlocul sau. Conform
Propozitiei 1, punctele Pa , Pb , Pc , Qa , Qb , Qc se aa pe cercul cu centrul M si raza rM
data de
(1)

rM = M Pa = M Pb = M Pc = M Qa = M Qb = M Qc .
103

B2

C1

B4

C3
Qc
Pc

Pb
Qb

B1

C4

B3

A2

A1
C2 B A3

Pa

Qa

A4

Vom exprima prin rM si q distantele punctului P la punctele A1 , A2 , B1 , B2 , C1 , C2 .


In acest scop, observam ca P A1 = P A2 (P situat pe mediatoarea segmentului A1 A2 )
si ca P A21 = P Pa2 + Pa A21 (P Pa A1 dreptunghic). Cum Pa A1 = Pa Q, avem
P A21 = P A22 = P Pa2 + Pa Q2 .

(2)

Aplicand teorema medianei n P Pa Q, obtinem


(3)

1
1
2
P Pa2 + Pa Q2 = 2M Pa2 + P Q2 = 2rM
+ q2 .
2
2

Din (2) si (3) rezulta ca


r

(4)

P A1 = P A2 =

2 + 1 q2 .
2rM
2

Analog, avem QA3 = QA4 si QA23 = QQ2a +Qa A23 = QQ2a +Qa P 2 = 2M Q2a + 21 P Q2 =
2
2rM
+ 12 q 2 ; ca urmare,
r

(5)
1 Profesor,

QA3 = QA4 =
Liceul ,,Ion Barbu, Pitesti

104

2 + 1 q2 .
2rM
2

Asadar, combin
and (4) cu (5), pentru punctele de pe BC obtinem relatiile:
r

2 + 1 q2 .
2rM
2
Procedand la fel, pentru punctele de pe CA si cele de pe AB se obtin relatiile
asemanatoare:

(7)

P A1 = P A2 = QA3 = QA4 =

2 + 1 q2 ,
2rM
2
r
2 + 1 q2 .
(9)
P C1 = P C2 = QC3 = QC4 = 2rM
2
Relatiile (7), (8) si
(9) arata ca punctele A1 , A2 , B1 , B2 , C1 , C2 se gasesc pe cercul
2 + 1 q 2 , iar punctele A , A , B , B , C , C se g
asesc pe
de centru P si raza 2rM
3
4
3
4
3
4
2
cercul de centru Q si aceeasi raza. Propozitia este complet demonstrata.

(8)

P B1 = P B2 = QB3 = QB4 =

Observatii. 1) Se considera ca punctele izogonale P, Q nu se aa pe dreptele


AB, BC, CA. Asa cum se observa n [2], restrictia ca ABC sa e ascutitunghic nu
este necesara.
2) Problema data la olimpiad
a revine la cazul n care P, Q sunt punctele H, O si

se are n vedere doar cercul C(O, r2 + 12 HO2 ), unde r este raza cercului lui Euler.
Bibliografie
1. T. Lalescu - Geometria triunghiului, Ed. Tineretului, Bucuresti, 1958.
2. Gazeta Matematic
a (B), nr. 7-8/2008, p. 352.

(R
aspunsuri la ,,recreatiile de la pag. 102)
1. Formulele de calcul pentru d si t sunt:
n(n 3)
, t = 3n3 28n2 + 92n 104.
2
Tabelul completat cu nc
a trei coloane este:
d=

n
d
t

3
0
1

4
2
8

5
5
31

6
9
88

7
14
197

8
20
376

...
...
...

2. Este vorba de sirul ultimelor litere (cele subliniate) ale cuvintelor din sirul
...
UNU, DOI, TREI, PATRU, CINCI, SASE, SAPTE, OPT, NOUA,

Asadar, n sirul dat urmeaza litera A.


Observatie. Aceasta ,,recreatie este o replica la sirul U, D, T, P, C, . . . (publicat
n G.M.-9/2002, Problema 31, p.348) format cu primele litere ale cuvintelor din sirul
de mai sus, sir n care trebuie continuat cu litera S.
105

C
ateva inegalit
ati geometrice n poligoane convexe
si nu numai

Dumitru M. BATINET
U-GIURGIU 1 , Neculai STANCIU

Abstract. In this article we present some algebraic inequalities and some of their applications
for general convex polygons, and tetrahedrons.
Keywords: Bergstr
om s inequality, Jensen s inequality, convex polygon.
MSC 2010: 51Mxx, 26D15.

Scopul acestui articol este de a stabili unele inegalitati algebrice pe baza carora
vom obtine cateva inegalitati geometrice n poligoane convexe, altele decat cele din
[1].
Teorema 1. Dac
a n N {1}, a, b, c, d, xk , yk , uk , vk R+ , k = 1, n, m, q
n
P

R + , Xn =

k=1
mVn

d max uk ,
1kn
n
X
k=1

xk , Yn =
q max

1kn

n
P

y k , Un =

k=1
vk , atunci:

n
P

uk , V n =

k=1

n
P

vj , astfel nc
at cUn >

k=1

aXn2 + bYn2
ax2k + byk2

cUn duk + mVn qvk


(cn d)Un + (mn q)Vn

(1)

1
(aXn + bYn )2
4ab
Xn Y n

.
a + b (cn d)Un + (mn q)Vn
a + b (cn d)Un + (mn q)Vn

Demonstratia 1. Conform inegalit


atii lui Bergstr
om avem:
n
X
k=1

x2k

cUn duk + mVn qvk

(
P
n

n
P

xk )2

k=1

(cUn duk + mVn qvk )

k=1

(2)

Xn2
(cn d)Un + (mn q)Vn

si analog
(3)

n
X
k=1

Yn2
yk2

.
cUn duk + mVn qvk
(cn d)Un + (mn q)Vn

Din (2) si (3) obtinem:


(4)

n
X
k=1

1 Profesor,
2 Profesor,

aXn2 bYn2
ax2k + byk2

.
cUn duk + mVn qvk
(cn d)Un + (mn q)Vn

Colegiul National ,,Matei Basarab, Bucuresti


S
coala General
a ,,George Emil Palade, Buz
au

106

Deoarece functia f : R+ R+ , f (t) = t2 este convexa pe R+ , conform inegalit


atii lui
Jensen obtinem:
b
a
f (t1 ) +
f (t2 )
a+b
a+b

(5)

at1 + bt2
a+b

, t1 , t2 R+ .

Daca n (5) luam t1 = Xn , t2 = Yn obtinem ca


(aXn bYn )2
aXn2 + bYn2
(aXn + bYn )2

.
aXn2 + bYn2
2
a+b
(a + b)
a+b

(6)

Din (4) si (6) deducem ca


n
X
k=1

aXn2 + bYn2
ax2k + yk2

cUn duk + mVn qvk


(cn d)Un + (mn q)Vn

(7)

1
(aXn + bYn )2

.
a + b (cn d)Un + (mn q)Vn

Conform inegalitatii dintre media aritmetica si media geometrica avem:


aXn + bYn 2

(8)

abXn Yn .

Din relatiile (7) si (8) deducem relatiile (1), ceea ce demonstreaza teorema.
Demonstratia 2. Datorita convexitatii functiei f : R+ R+ , f (t) = t2 pe R+ ,
conform inegalitaatii lui Jensen, rezulta inegalitatea (5) si atunci avem:
(9)

a
b
(axk + byk )2
1
x2k +
yk2
ax2k + byk2
(axk + byk )2 , k = 1, n.
a+b
a+b
(a + b)2
a+b

Conform inegalitatilor (9) avem:


n
X
k=1

ax2k + byk2
1 X
(ax k + byk )2

,
cUn duk + mVn qvk
a+b
cUn duk + mVn qvk
n

k=1

unde aplicam inegalitatea lui H. Bergstrom, iar apoi inegalitatea dintre media aritmetica si media geometrica si obtinem inegalitatile (1).
Comentariu 1.1. In prima demonstratie am aplicat mai ntai inegalitatea lui H.
Bergstrom si apoi inegalitatea lui Jensen pe cand n a doua demonstratie am aplicat
ntai inegalitatea lui Jensen si apoi inegalitatea lui H. Bergstrom.
Fie A1 A2 . . . An , n 3 un poligon convex si P Int A1 A2 . . . An . Notam: xk =
xk (P ) = P Ak , dk distanta de la P la dreapta Ak Ak+1 , ak = Ak Ak+1 , Bk piciorul
bisectoarei interioare a unghiului Ak P Ak+1 n triunghiul Ak P Ak+1 , Pk mijlocul
segmentului [Ak Ak+1 ], mk = P Pk si wk = P Bk , unde k = 1, n si An+1 A1 . De
asemenea notam: Xn = Xn (P ) =
n
P
k=1

mk , Wn = Wn (P ) =

n
P

n
P

xk , Dn = Dn (P ) =

k=1

n
P

k=1

wk si cu 2p perimetrul poligonului.

k=1

107

dk , Mn = Mn (P ) =

Teorema 2. Dac
a A1 A2 . . . An , n 3, este un poligon convex, P un punct
interior poligonului, a, b, c, d R+ , m, q R+ astfel nc
at cWn > d max wk , mDn
1kn

q max dk , atunci:
qkn

n
X

(10)

k=1

Dn2 sec2 n
ax2k + bd2k
4ab

.
cWn + mDn dwk qdk
a + b ((c + m)n (d + q))Xn

Demonstratie. Daca n relatiile (1) luam yk = dk , uk = wk , vk = dk , k = 1, n,


atunci obtinem:
n
X
k=1

(11)

ax2k + bd2k
aXn2 + bDn2

cWn dwk + mDn qdk


(cn d)Wn + (mn q)Dn

(aXn + bDn )2
4ab
Xn Dn
1

.
a + b (cn d)Wn + (mn q)Dn
a + b (cn d)Wn + (mn q)Dn

Conform [2], H.C. Lenhard a demonstrat ca


(12)

Wn =

n
X

wk

k=1

n
X

xk

k=1

cos

= Xn cos .
n
n

Datorita faptului ca dk wk n orice triunghi Ak P Ak+1 , k = 1, n rezulta ca


(13)

Dn =

n
X

dk Wn Xn cos

k=1

.
n

Inegalitatea (13) este inegalitatea lui L. Fejes T


oth (a se vedea [3]).
Dac
a tinem seama de relatiile (12) si (13), atunci din (11) rezulta ceea ce trebuia
demonstrat.
Comentariu 2.1. Alegand n Teorema 1 xk , yk , uk , vk {xk , wk , dk }, unde k =
1, n, n diverse moduri se pot obtine inegalitati de tipul (10). De exemplu, daca
xk = xk , yk = wk , uk = dk , vk = xk , k = 1, n, atunci obtinem ca
n
X
k=1

ax2k + bwk2
4ab
Xn W n

cDn ddk + mXn qxk


a + b (cn d)Dn + (mn q)Xn

(14)

Wn2 sec2 n
4ab

.
a + b (cn d) cos + mn q Xn
n

Observatia 2.1. Daca m = q = 0, atunci din relatia (11) obtinem ca


n
X
ax2k + bd2k
k=1

(15)

cWn dwk

aXn2 + bDn2
4ab
Xn Dn

(cn d)Wn
a + b (cn d)Wn

4abDn2 sec2 n
4abDn2 sec2 n
=
.
(a + b)(cn d)Xn cos n
(a + b)(cn d)Xn
108

Observatia 2.2. Dac


a poligonul este un triunghi A1 A2 A3 , atunci relatiile (13) si
(14) devin:
(16)

3
X

W3 =

wk X3 cos

k=1

1
= X3
3
2

si respectiv
D3 = d1 + d2 + d3

(17)

1
X3 2(d1 + d2 + d3 ) x1 + x2 + x3 ,
2

unde relatia (17) este inegalitatea Erd


os-Mordell.
In acest mod obtinem din relatiile (10) si (14):
ax22 + bd22
ax23 + bd23
ax21 +bd21
+
+

cW3 +mD3 dw1 qd1 cW3 + mD3 dw2 qd2 cW3 + mD3 dw3 qd3
(18)
D32 sec2 3
16abD32
4ab

=
,

a + b (3(c + m) (d + q))X3
(3(c + m) (d + q))X3
si respectiv
3
X

W32 sec 3
ax2k + bwk2
4ab

=
cDn + mXn ddk qxk
a + b (3c d) cos + (3m q) X3
k=1
3
(19)

16abW32
.
(3(c + 2m) (d + 2q))X3

Fie un paralelipiped dreptunghic de dimensiuni x, y, z R+ si diagonala R+ ,


iar [ABCD] un tetraedru tridreptunghic n A. Notam cu SA aria fetei opuse varfului
A, iar cu SB , SC si SD analoagele lui SA . Din relatiile (1) obtinem ca
c 2

4
ax4 + bSB
ay 4 + bSc4
+ 2
2
2
2 dy 2 qS 2 +
2
+ mSA dx qSB
c + mSA
C

(20)
+

2
4
az 2 + bSD
a 4 + bSA

2
2
2
c 2 + mSA dz 2 qSD
(3c d) 2 + (3m )SA

2 2
2
1
(a 2 + bSA
)
4ab
2 SA

,
2
a + b (3c d) 2 + (3m q)SA
a + b (3c d) 2 + (3m q)S42

2
2
2
2
unde am tinut seama ca 2 = x2 + y 2 + z 2 si ca SA
= SB
+ SC
+ SD
.

Fie [ABCD] un tetraedru oarecare si ha , hb , hc , hd , ra , rb , rc si rd , lungimile naltimilor tetraedrului si respectiv razele sferelor exnscrise tetraedrului. Daca r este raza
sferei nscrise n tetraedrul considerat, atunci se stie ca
1
1
1
1
1
+
+
+
=
ha
hb
hc
hd
r

si
109

1
1
1
1
2
+
+
+
= .
ra
rb
rc
rd
r

Teorema 3. Dac
a a, b, c, d R+ , m, q R+ , astfel nc
at
m
q max ha , atunci:
r
a r12 + b h12

X
ciclic

(2c +

m) 1r

d r1a

q h1a

1
2c
> d max
si
r
ra

4a + b

(8c + 4m 2d q)r

(21)

1
(2a + b)2
8ab
1

.
a + b (8c + 4m 2d q)r
a + b (8c + 4m 2d q)r

Demonstratie. Pentru n = 4 din Teorema 1 obtinem:


4
X
k=1

ax2k + byk2
aX42 + bY42

cU4 duk + mV4 qvk


(4c d)U4 + (4m q)V4

(aX4 + bY4 )2
4ab
X4 Y 4
1

,
a + b (4c d)U4 + (4m q)V4
a + b (4c d)U4 + (4m q)V4

n care luam X4 =
concluzia.

P 1

ra

P 1
P 1
P 1
2
2
, Y4 =
, U4 =
= si V4 =
si obtinem
r
ha
ra
r
ha

Bibliografie
1. D.M. B
atinetu-Giurgiu, N. Stanciu Inegalit
ati geometrice n poligoane convexe,
de tip Bergstr
om-Mitrinovi
c, Recreatii Matematice, 2/2011, 112-115.
2. H.C. Lenhard Verallgemeinerung und Versch
arfung der Erd
os-Mordellschen Ungleichung f
ur Polygone, Arh. Math. Vol XII (1961), 311-314.
3. N. Minculete, A. Gobej Inegalit
ati geometrice de tipul Erd
os-Mordell ntr-un
poligon convex, Gazeta Matematica - seria A, nr. 1-2/2010, 1-8.

Scrieti ecare dintre numerele 1, 2, 3, 4, 5, 6, 7, 8, 9, 10 utilizand numai dou


a cifre
de 7, operatii aritmetice si diverse functii.

Un model care face apel la functiile x x, x R+ , si x [x] , x R, este:


1 = 7: 7, 
2 = 7, 7 ,

3=
7+7 ,


4=
7+ 7 ,


7 7 ,

6= 7 7 ,

7 = 7 7,

5=

8=

77 ,

9=7+ 7 ,

10 = 7 7 .
Titu ZVONARU

110

O dubl
a inegalitate integral
a si c
ateva aplicatii
1
R
azvan-Dumitru CEUCA
Abstract. The aim of this Note consists in establishing the inequalities (1). A couple of their
consequences and applications are also given.
Keywords: Cauchy-Buniakovski-Schwarz inequality, Bergstr
om s inequality.
MSC 2010: 26D15.

Scopul acestei Note este stabilirea urmatorului rezultat:


Teorem
a. Dac
a functiile f, g : [a, b] R si , : [a, b] R+ sunt continue,
atunci au loc inegalit
atile:


Z b 2
f (t)

(1)

g 2 (t)
+
dt
(t)
(t)

2
b
a f (t)dt
Rb
a (t)dt

2
b
a g(t)dt
Rb
a (t)dt

R
b

a (f (t) + g(t))dt

Rb

a [(t)

+ (t)]dt

Egalit
atile sunt atinse dac
a si numai dac
a f = k1 , g = k2 cu k1 , k2 R.
Demonstratie. Pentru a dovedi prima inegalitate este sucient sa aratam ca
Z b 2
f (t)

(2)

(t)

Rb

dt

2
a f (t)dt]
Rb
a (t)dt

Z b 2
g (t)

si

(t)

Rb

dt Rab
a

g(t)dt]2

(t)dt

Punem prima dintre inegalitatile (2) sub forma


Z b 2
f (t)

(3)

(t)

dt

Z b

(t)dt

2

Z b

f (t)dt
a

si observam ca aceasta din urma este adevarata conform inegalitatii Cauchy-BuniakovskiSchwarz, forma integral
a. La fel procedam pentru a dovedi a doua inegalitate din (2).
Pentru a dovedi a doua inegalitate din (1), vom utliza inegalitatea
x2
y2
(x + y)2
+

(4)

x, y R, , R+

(T. Andreescu, R.M.T., 1979), care este un caz particular al inegalit


atii lui Bergstr
om.
x
y
Egalitatea n (4) se atinge daca si numai daca = .

Intr-adev
ar, luand n (4) x =
Z b

Z b

Z b

f (t)dt, y =
a

Z b

g(t)dt, =
a

(t)dt > 0 obtinem partea din dreapta a inegalitatilor (1).


a

1 Elev,

cl. a XII-a, Colegiul National, Iasi

111

(t)dt > 0 si
a

In sfarsit, armatia relativ la cazul de egalitate n (1) rezulta imediat din conditiile
n care n (3) si (4) avem egalitate. Teorema este complet demonstrata.
Corolar 1. Dac
a functiile f, g : [a, b] R+ sunt continue, atunci
Z b 3
f (t) + g 3 (t)

(5)

f (t) g(t)

dt

b
a

f (t)dt
Rb
a

Rb

f (t)dt

b
a

g(t)dt

g(t)dt

Z b

[f (t) + g(t)]dt,
a

cu egalitate dac
a si numai dac
a f = kg, k R+ .
Demonstratie. In (1) luam = g si = f.
Exemplul 1. Ar
atati c
a

Z 1
arctg3 x + e3x
0

ex

arctg x

dx

+ e ln 2 1.
4

Utilizand (5), avem:


Z 1
arctg3 x e3x
0

ex arctg x

1
Z 1

x dx
dx
(arctg x + e )dx = x arctg x
+
2
0
0 1+x
0
1
1

x
2
+ e = + e 1 ln(1 + x ) = + e ln 2 1.
4
2
4
Z 1

Corolar 2. Dac
a functiile f, g : [a, b]
Z b

f 2 (t) g(t) +

(6)
a

R+

sunt continue, atunci

1
dt
f (t) g 2 (t)

Z b

f (t) +
a

1
dt,
g(t)

cu egalitate dac
a si numai dac
a f g = 1.
Demonstratie. In (1), din care se omite membrul din mijloc, luam ca functii
1 1
f, g, , functiile f, , , respectiv f .
g g
Z e

1
xe2x + 2 x dx ee e + 1.
Exemplul 2. Ar
atati c
a
x e
1 Z
Z e

e
1
1
2x
x
Avem:
xe + 2 x dx
e +
dx = ee e + 1.
x e
x
1
1
In nal, propunem urmatoarele exercitii (f, g : [a, b] R continue, 0 < a < b):
2
Z b
Z b 2
f (x) + g 2 (x)
1
(f
(x)
+
g(x))dx
;
1)
dx 2
x
b a2 a
a
2
Z b
Z b
1
x[f 2 (x) + g 2 (x)]dx
2)
(f
(x)
+
g(x))dx
.
2 ln ab
a
a
Bibliografie
1. M. Becheanu, B. Enescu Inegalit
ati elementare ... si mai putin elementare, Ed.
Gil, Zalau, 2002.
2. D. Dr
acea, L. Niculescu, I. P
atrascu, D. Secl
aman Matematica. Manual
pentru clasa a XII-a, Ed. Cardinal, Craiova, 2007.
112

Cercles et inversion
Adrien REISNER1
Abstract. In this paper we use the inversion for find certain locus concerning circles.
Keywords: orthogonal circles, inversion, locus.
MSC 2010: 97D40.

On donne trois points dans le plan A, B et P . Sur la droite (AB) on consid`ere


un point variable C. Soient (S) et (S ) les cercles circonscrits aux triangles P AC et
P BC respectivement. On trace le cercle (S1 ) passant par P, A et orthogonal au cercle
(S) et le cercle (S1 ) passant par P, B et orthogonal au cercle (S ).
Soient D, E, F les points de rencontre autres que P des couples de cercles
((S1 ), (S1 )), ((S), (S1 )), ((S ), (S1 )) respectivement.
Je designe par Inv l inversion Inv(P, k) de pole P et de puissance k, o`
u k est un
reel arbitraire non nul, i.e. la transformation qui au point M du plan fait correspondre
le point m = Inv(M ) de la droite (P M ) veriant la relation
PM Pm = k

(produit des valeurs a lg ebriques).

On transforme la gure par l inversion Inv en designant par a, b, c, d, ... les inverses
des points A, B, C, D, . . . La droite
(AB) admet pour inverse le cercle ()
qui passe par les points P, a et b. Le
point c est un point variable de ce cercle (). Les cercles (S), (S ), (S1 ),
(S1 ) ont respectivement pour inverses
les droites (ac), (bc), (ad) perpendiculaire `a la droite (ac), et (bd) perpendiculaire `a (bc) voir gure .
Proposition 1. Les deux cercles
(S) et (S ) se coupent sous un angle constant .
D
emonstration. Lorsque le point c decrit le cercle (), l angle des droites (ca, cb)
conserve une valeur constante . On en deduit la proposition 1.
Proposition 2. Les cercles (S1 ) et (S1 ) se coupent sous le meme angle constant .
D
emonstration. Les angles ]cad et ]cbd etant droits, le point d se trouve sur
le cercle () diametralement oppose au point c. Il vient donc:
(da, db) = (ca, cb) =
1 TELECOM

(angle de droites)

ParisTech; e-mail: Adrien.Reisner@telecom-paristech.fr

113

et par consequent les cercles (S1 ) et (S1 ) se coupent aussi sous le meme angle ,
c.q.f.d.
Proposition 3. a) Le lieu geometrique du point D est la droite (AB).
b) Le lieu geometrique des points E et F est le cercle decrit sur (AB) comme
diam`etre.
D
emonstration. Le lieu du point d etant le cercle () on en deduit immediatement
l assertion a) de la proposition 3.
b) Cherchons les lieux geometriques des points e et f . On a (ea, eb) = (ca, db) =

(ca, cb) + (cb, db) ou (ea, eb) = + . L angle des droites (ea, eb) etant constant, le
2
point e decrit un cercle ( ) passant par les points a et b.

On a de meme (f a, f b) = (da, cb) = (da, db) + (db, cb) ou (f a, f b) = + . Le


2
point f decrit aussi le meme cercle ( ).
On se propose de montrer que les deux cercles () et ( ) sont orthogonaux.
Designons par at et at les tangentes `a ces deux cercles au point a. Lorsque le point c
est en a, on a (ca, cb) = (at, ab) = . Quand le point e est en a, nous avons de meme

(ea, eb) = (at , ab) = + .


2
On en deduit:

(at , ab) (at, ab) = ,


2
soit nalement:

(at , at) = ,
2
c.q.f.d. Ainsi le lieu des points e et f est le cercle ( ) orthogonal au cercle () et
passant par les points a et b.
Il en resulte que le lieu des points E et F est un cercle () passant par les points
A et B et orthogonal `a la droite (AB), i.e. le cercle de diam`etre AB. La proposition
3 est ainsi demontree.
Proposition 4. Le centre du cercle circonscrit au triangle P CD se trouve sur la
droite (AB).
D
emonstration. Le cercle (C) circonscrit au triangle P CD admet pour inverse
la droite (cd). Celle-ci est orthogonale au cercle () puisqu elle est un diam`etre de
ce cercle. (C) est donc orthogonal `a la droite (AB): c est le cercle de diam`etre AB,
d o`
u la proposition 4.
Je tiens `
a remercier ici mes coll`egues Mmes Anne-Francoise Chacun, Nadine Diop, MM.
Christian Blin, Mourad Debbabi, Mehdi El-Kadaoui, Gilles Dauphin, Arnaud Galisson, Si
Mohamed Laroussi, Yves Meugniot, Laurent Mondo Enny, Frederic Pauget, Eric Peltier,
Patrice Pietu, Jorge Queixalos, Nicolas Religieux, Jean-Francois Sintes et les autres pour le
soutien qu ils m ont apporte `
a lecriture de mes articles.

Bibliografie

1. Mich`
ele Audin Geometrie, Editions
EDP, Paris, 2006.

2. J. Commeau Cours de Geometrie, Editions Masson, Paris, 1968.


114

Concursul International de Matematic


a
,,Vladimir Andrunachievici
Editia a II-a, Chisin
au, 5 ianuarie 2012

Clasa a VI-a
1. Dac
a la un num
ar natural se adun
a suma cifrelor lui, atunci se obtine num
arul
2012. S
a se determine toate numerele naturale cu aceast
a proprietate.
Solutie. Evident, numerele cautate au patru cifre, iar cifra miilor poate doar 1
sau 2. Daca 1abc + 1 + a + b + c = 2012, obtinem ca 101a + 11b + 2c = 1011; cum
11b+2c 117, rezulta ca a = 9, apoi b = 8, c = 7, deci numarul 1987 verica cerintele
problemei. Daca 2abc + 2 + a + b + c = 2012, atunci 101a + 11b + 2c = 10, de unde
a = b = 0, c = 5, obtin
and astfel num
arul 2005 ca a doua solutie a problemei.
sistemul zecimal, cifrele x si y si num
2. In
arul natural n satisfac egalitatea
xy + yx = 1 + 2 + ... + n.
S
a se determine toate valorile lui n si toate numerele xy care satisfac aceast
a egalitate.
Solutie. Egalitatea din enunt revine la 2 11(x + y) = n(n + 1), unde x + y
{2, 3, . . . , 18}. Din n(n + 1) 2 11 18 obtinem ca n 19. Deoarece numarul prim
11 divide n(n + 1), avem doar doua posibilitati: n = 10 sau n = 11. Daca n = 10,
atunci x + y = 5, prin urmare xy {14, 23, 32, 41}. Daca n = 11, atunci x + y = 6,
deci xy {15, 24, 33, 42, 51}.
3. S
a se determine 15 numere naturale nenule cu proprietatea c
a produsul succesorilor lor este de 2012 ori mai mare dec
at produsul lor.
Solutie. Vom gasi efectiv 15 numere x1 , x2 , . . . , x15 cu proprietatea ca (x1 +
1)(x2 + 1) . . . (x15 + 1) = 2012 x1 x2 . . . x15 . Cum 2012 = 22 503, produsul din stanga
trebuie sa se divida cu 503; pentru aceasta, luam x15 = 502. Atunci produsul din
dreapta se va divide cu 251, ceea ce conduce la alegerea x14 = 250. Egalitatea initiala
revine astfel la
(x1 + 1)(x2 + 1) . . . (x13 + 1) = 24 53 x1 x2 . . . x13 .
Consider
am x13 = x12 = x11 = 4 si suntem condusi la
(x1 + 1)(x2 + 1) . . . (x10 + 1) = 210 x1 x2 . . . x10 .
Numerele naturale nenule x1 = x2 = . . . = x10 = 1 satisfac ultima relatie. Prin
urmare exista 15 numere naturale cu proprietatea dorita, anume 1, 1, 1, 1, 1, 1, 1, 1, 1, 1,
4, 4, 4, 250, 502. Binenteles, sunt posibile si alte exemple.
4. Pe tabl
a sunt scrise numerele 1, 2, 3, ..., 33. Un elev efectueaz
a urm
atoarea
operatie: alege dou
a numere astfel nc
at unul din ele s
a e multiplu al celuilalt si
apoi le nlocuieste cu c
atul lor. Elevul repet
a operatia p
an
a c
and niciun num
ar de pe
115

tabl
a nu este multiplu al altuia. S
a se determine num
arul minim de numere care pot
r
am
ane pe tabl
a.
Solutie. Produsul tuturor numerelor aate initial pe tabla este P = 33! = 231
315 57 74 113 132 17 19 23 29 31. Efectuarea unei operati nu schimba paritatea
exponentilor din descompuneea n factori primi a produsului numerelor aate pe tabla:
alegand numerele x si y, cu y = x a, n locul lor va aparea pe tabla numarul a;
astfel produsul x y = x2 a se nlocuieste cu a, paritatea exponentilor ramanand
aceeasi. Rezulta ca produsul numerelor aate n nal pe tabla va contine factorii
2, 3, 5, 11, 17, 19, 23, 29, 31.
Dorim ca, la sfarsit, pe tabla sa raman
a cat mai putine numere. In acest scop,
factorii de mai sus ar trebui grupati astfel ncat sa nu depasim 33; observam ca nu
vom putea avea mai putin de sapte numere.
Vom demonstra ca num
arul minim cautat este 7. Succesiunea de operatii:
(16, 32) 2; (15, 30) 2; (14, 28) 2; (13, 26) 2; (12, 24) 2; (11, 22) 2;
(9, 27) 3; (7, 21) (3); (6, 18) 3; (5, 25) 5; (4, 20) 5; (2, 8) 4; (5, 5) 1;
(2, 4) 2; (3, 3) 1; (3, 3) 1; (2, 2) 1; (2, 2) 1; (2, 2) 1 lasa pe
tabla numerele 17, 19, 23, 29, 31, 10, 33 si sapte de 1. In urmatorii sapte pasi eliminam
numerele de 1, astfel nc
at n nal vor ramane pe tabla sapte numere.
Clasa a VII-a
1. S
a se determine toate perechile (m, n) de numere ntregi care satisfac ecuatia
(m + n) (m + n + 2) = 23 3|mn| + 1.
Solutie. Relatia din enunt revine la (m + n + 1)2 = 23 3|mn| + 2. Cum nu exista
patrate perfecte de forma M3 + 2, rezulta ca |m n| = 0, deci m = n. Obtinem ca
(2m + 1)2 = 25, de unde m {3, 2} si atunci (m, n) {(3, 3); (2, 2)}.
2. Un p
atrat de 8 8 este divizat n 64 p
atr
atele de 1 1. Care este num
arul
maxim de diagonale care pot duse n p
atr
atelele 1 1 astfel nc
at oricare dou
a
diagonale s
a nu aib
a puncte comune?
Solutie. In gura alaturata sunt trasate 36 de diagonale, oricare doua fara puncte
comune.
Vom demonstra ca 36 este numarul maxim cautat.
Fie 37 de diagonale duse n patratelele 1 1. Impartim
patratul 8 8 n patru dreptunghiuri 8 2; din principiul cutiei,
exista un astfel de dreptunghi care contine macar 10 diagonale.
Fiecare dintre ele are un capat pe axa de simetrie de lungime 8 a
dreptunghiului, care nu contine nsa decat noua noduri. Rezulta
ca doua dintre cele 10 diagonale vor avea un capat comun, ceea
ce ncheie solutia problemei.
3. Fie p un num
ar prim. Printre numerele naturale n de forma
n = (p2 + 32)2 69 (p2 + 32) + 2250,
s
a se determine num
arul cu cea mai mic
a sum
a a cifrelor lui.
116

Solutie. Observ
am ca n = p4 5p2 + 1066 = (p2 4)(p2 1) + 1062 = (p 2)(p
1)(p + 1)(p + 2) + 1062. Daca p = 3, atunci numerele (p 2)(p 1), (p + 1)(p + 2)
si 1062 se divid cu 9, prin urmare n este multiplu de 9. Suma cifrelor lui n va , de
asemenea, multiplu de 9, deci va cel putin egala cu 9. Daca p = 3, atunci n = 1102,
numar care are suma cifrelor 4. In concluzie, numarul n care are suma cifrelor minima
este 1102.
4. Numerele a, b, c sunt elemente ale multimii {1, 2, ..., 2012} . S
a se determine
num
arul tuturor tripletelor ordonate diferite (a, b, c) cu proprietatea c
a suma a + b + c
este un multiplu al ec
arui din numerele a, b, c. (Dou
a triplete formate din aceleasi
trei numere sunt triplete ordonate diferite dac
a ordinea numerelor n aceste triplete
este diferit
a.)
Solutie. Fie, pentru nceput, a b c. Cum c|a + b + c si 0 < a + b 2c, rezulta
ca a + b {c, 2c}.
I. Daca a + b = c, atunci num
arul 2c = a + b + c este multiplu al lui a si al lui
1
1
1
b, prin urmare 2c = am, 2c = an, cu m, n N , m n. Obtinem ca
+ = ,
m
n
2
1
1
1
1
1
2
cu
, deci =
+ , adica n 4. Prin vericari directe, pentru n = 4
m
n
2
m n
n
gasim tripletele (k, k, 2k), 1 k 1006, iar pentru n = 3 gasim tripletele (k, 2k, 3k),
1 k 670.
II. Daca a+b = 2c, atunci a = b = c, deci tripletele (k, k, k), k k {1, 2, . . . , 2012},
verica cerintele problemei.
In total, tin
and seama si de posibilele permutari n cadrul unui triplet, obtinem
3 1006 + 6 670 + 2012 = 9050 tripletele ca n enunt.
Clasa a IX-a
1. S
a se determine cel mai mic num
ar natural nenul n astfel nc
at numerele
14n, 16n, 18n, 20n s
a aib
a acelasi num
ar de divizori.
Solutie. Cel mai mic num
ar n cu proprietatea din enunt va avea n descompunerea
sa n factori primi doar divizori primi ai numerelor 14, 16, 18 sau 20, deci va de
forma n = 2a 3b 5c 7d . Notand cu d(x) numarul divizorilor naturali ai lui x, vom
avea: d(14n) = (a + 2)(b + 1)(c + 1)(d + 2); d(16n) = (a + 5)(b + 1)(c + 1)(d + 1);
d(18n) = (a+2)(b+3)(c+1)(d+1); d(20n) = (a+3)(b+1)(c+2)(d+1). Egaland doua
cate doua aceste numere, obtinem ca a 1 = 3d, 2a + 1 = 3b, respectiv a + 1 = 2c.
Cea mai mica valoare a lui a pentru care a, b, c, d sunt numere naturale este a = 1,
cand b = c = 1, d = 0. In concluzie, n = 21 31 51 70 = 30 este cel mai mic numar
cu proprietatile din enunt.
obtuz. Punctul P interior tri2. Fie paralelogramul ABCD cu unghiul ABC
unghiului BDC este situat pe diagonala (AC) astfel nc
at m (BP D) = m (ABC) .
S
a se demonstreze c
a dreapta CD este tangent
a la cercul circumscris triunghiului
BCP dac
a si numai dac
a AB = BD.
b

b
= 180 , patrulaterul (convex)
Solutie. Intruc
at m(A)+m(
BP
D) = m(A)+m(
B)

117

DAP

ABP D este inscriptibil, prin urmare DBP

BCP . Cum dreapta BP separa punctele C si D,


avem: dreapta CD este tangenta cercului (BCP )

P
CD P
BC m(P
CD) + m(P
CB) = m(P
BC) +
A

m(P BD) DCB DBC DC = DB AB = BD.

3. S
a se determine toate perechile (a, b) de numere naturale care satisfac ecuatia
4a + 4a2 + 4 = b2 .
Solutie. Daca a 7, se arata prin inductie ca 2a+1 > 4a2 + 3 si atunci 4a + 4a2 +
4 < 4a + 2a+1 + 1 = (2a + 1)2 . Evident ca 4a + 4a2 + 4 > (2a )2 , a 7 si, cum ntre
doua patrate perfecte consecutive nu se mai gasesc alte patrate perfecte, rezulta ca
ecuatia din enunt nu are solutii (a, b) cu a 7. Efectuand vericari directe pentru
a {0, 1, . . . , 6}, obtinem solutiile (a, b) {(2, 6); (4, 18)}.
4. Fie numerele reale

A=

10 +

B=

10

1+
1+

10 +

10

2 + ... +
2 + ... +

10 +

10

99,
99.

A
2 este natural.
B
Solutie. Are loc identitatea

S
a se arate c
a num
arul

20 + 2 n = 10 + 100 n + 10 100 n, n {1, 2, . . . , 99}.

Dand lui n valorile 1, 2, . . . , 99, adunand membru cu


tile obtinute si
membru egalita
observand ca suma din membrul stang este chiar A 2, obtinem ca A 2 = A + B

A
A( 2 1) = B
2 = 1 N.
B
Clasa a XII-a
1. Toate elementele multimii
A = {p, 3p + 2, 5p + 4, 7p + 6, 9p + 8, 11p + 10}
sunt numere prime. S
a se arate c
a num
arul 17p + 2012 este compus.
Solutie. Daca p = 5, atunci 11p + 10 = 65, numar compus. Daca p 1(mod 5),
atunci 3p + 2 0 (mod 5), deci 3p + 2 este compus. Daca p 2 (mod 5), atunci
7p + 6 0 (mod 5), deci 7p + 6 este compus. Daca p 3 (mod 5), atunci 9p + 8 0
(mod 5), deci 9p + 8 este compus. Ramane ca p 4 (mod 5) si atunci 17p + 2012 0
(mod 5), asadar 17p + 2012 este numar compus.
Not
a. Pentru p = 2099, cele sase numere din enunt sunt toate prime, deci problema are obiect.
2. Multimea nevid
a A contine m numere naturale pare nenule, iar multimea
nevid
a B contine n numere naturale impare astfel nc
at suma elementelor din ambele
multimi este egal
a cu 2012. S
a se ae cea mai mare valoare posibil
a a sumei 3m + 4n.
118

Solutie. Fie Sm suma elementelor lui A si Sn suma elementelor lui B; atunci Sm


2+4+. . .+2m = m(m+1), iar Sn 1+3+. . .+(2n1) = n2 . Cum Sm +Sn = 2012,
obtinem ca m(m + 1) + n2 2012, de unde (y 3m)2 16(2012 m(m 1)), unde
am notat y = 3m + 4n. Deducem astfel ca 25m2 2(3y 8)m + y 2 32192 0,
unde y N, y 7. Impunem conditia 0 si rezulta ca y 2 + 3y 50304 0,
deci y 222. Pentru a dovedi faptul ca ymax = 222, este sucient sa observam ca,
daca A = {2, 4, . . . , 48, 50, 62} si B = {1, 3, . . . , 67, 69, 75}, atunci m = 26, n = 36,
Sm + Sn = 712 + 1300 = 2012, iar 3m + 4n = 222.
3. S
a se calculeze integrala Riemann a functiei

h
cos 2x + 2 cos x + 4
i
x
f : 0,
R,
f (x) = e
.
2
(1 + sin x) (1 + cos x)



Solutie. Intruc
at [(1+sin x)(1+cos x)] = cos 2x+ 2 cos x +
, notatia g(x) =
4
Z
i
h

(1 + sin x)(1 + cos x), x 0, , transforma integrala de calculat I =


f (x)dx
2
0
n
Z
Z

2
2
I=2
ex g (x)dx = 2 2(e 2 1) 2
ex g(x)dx.
0

1
Din identitatea (1 + cos x + sin x)2 = 2(1 + sin x)(1 + cos x) rezulta ca g(x) = (1 +
2
cos x + sin x), prin urmare
Z


2
1
1
ex (1 + sin x + cos x)dx = ex (1 + sin x) = 2(e 2 ).
2
2
0
0
0

Inlocuind n I, obtinem n nal ca I = 2.


4. S
irul de numere ntregi pozitive (an )nN satisface conditiile: an este un multiplu al lui n si |an an+1 | 5 oricare ar n N . S
a se determine cea mai mare
valoare posibil
a a lui a1 .
Solutie. Din 5 an an+1 5, rezulta ca an+1 an + 5 de unde, din aproape
n aproape, obtinem ca an a1 + 5(n 1), n N . Pentru valori sucient de mari
ale lui n (de exemplu, n > a1 5), gasim ca an < 6n si de aici, deoarece an este
multiplu de n, avem ca an 5n (pentru n sucient de mare).
Astfel, exista un numar nit de valori ale lui n pentru care an > 5n; e k cel mai
mare num
ar cu aceasta proprietate, deci ak > 5k si ak+1 5(k + 1). Cum ak este
multiplu al lui k, avem ca ak 6k, deci 5 ak ak+1 6k 5(k + 1) = k 5, asadar
k 10.
In concluzie, an 5n, n 11; n particular, a11 55. Intrucat an este multiplu
al lui n care nu depaseste num
arul an+1 + 5, obtinem recursiv: a10 60, a9 63,
a8 64, a7 63, a6 66, a5 70, a4 72, a3 75, a2 80, a1 85. Sirul denit
prin a1 = 85, a2 = 80, a3 = 75, a4 = 72, a5 = 70, a6 = 66, a7 = 63, a8 = 64, a9 = 63,
a10 = 60, an = 5n, n 11, are proprietatile din enunt, prin urmare cea mai mare
valoare posibila a lui a1 este 85.

1
ex g(x)dx =
2

119

C
ateva solutii la problema L222 din
Recreatii Matematice, nr. 1/2012
Titu ZVONARU

Abstract. In this Note, the way to establish the inequality (1) is used as a pretext for pointing
out several procedures and working tools that are useful in approaching inequalities.
Keywords: AM-GM inequality, Chebyshev s inequality, Bergstr
om s inequality.
MSC 2010: 97H30, 52A40.

Vom prezenta opt solutii (printre care si solutia autorului) pentru problema urmatoare:
L222. Pentru a, b, c numere reale pozitive, demonstrati inegalitatea

(1)

1
1
+ 2
b2
c

+b

1
1
+ 2
c2
a

+c

1
1
+ 2
a2
b

18
.
a+b+c

Florin St
anescu, G
aesti
Cu acest prilej vor puse n evidenta diverse procedee de lucru si tehnici de calcul.
Este util sa scriem inegalitatea (1) n forma
b+c c+a a+b
18
+ 2 + 2
, a, b, c R+ .
2
a
b
c
a+b+c

(2)
Solutia 1.
obtinem:

Aplicand inegalitatea dintre media aritmetica si cea geometrica,


r

b+c c+a a+b


3 (a + b)(b + c)(c + a)
+ 2 + 2 3

a2
b
c
a2 b2 c2
r
18
8abc
6
3

3
=
,
3
a2 b2 c2
a+b+c
abc
de unde rezulta (2). Avem egalitate daca si numai daca a = b = c.
Solutia 2. Putem sa presupunem ca a b c fara a restrange generalitatea. Ca
1
1
1
urmare, 2 2 2 si b + c c + a a + b. Aplicand inegalitatea lui Cebsev,
a
b
c
obtinem

1
1
1
1
b+a c+a a+b
+ 2 + 2 (b + c + c + a + a + b) 2 + 2 + 2
a2
b
c
3
a
b
c

si ram
ane de aratat ca

2
1
1
1
(a + b + c) 2 + 2 + 2
3
a
b
c
1 Profesor,

18
(a + b + c)2

a+b+c

Com
anesti, tzvonaru@yahoo.com

120

1
1
1
+ 2+ 2
a2
b
c

27.

Cu inegalitatea dintre mediile aritmetica si geometrica, avem

(a + b + c)2

1
1
1
+ 2+ 2
2
a
b
c

3
3
= 27,
(3 abc)2
3
2
a b2 c2

ceea ce ncheie demonstratia.


x31 x32 x33
(x1 + x2 + x3 )3
+
+

,
y12 y22 y32
(y1 + y2 + y3 )2
1
3
xi , yi R+ (Bergstr
om) si

, a, b, c R+ (revine la
ab + bc + ca
(a + b + c)2
a2 + b2 + c2 ab + bc + ca). Avem:
Solutia 3 (a autorului). Vom folosi inegalitatile

1
1
a 2+ 2
b
c

Xa+b

X (a + b)3

(a + b)]3
P

=
c2
[c(a + b)]2
[ c(a + b)]2
8(a + b + c)3
2 9 (a + b + c)3
18
=

=
.
2
2
4(ab + bc + ca)
(a + b + c)
a+b+c
=

Solutia 4 (Daniel V
acaru, Pitesti). Punem (1) sub forma
(3)

a3 + b3
b3 + c3
c3 + a3
18
+ 2 2 + 2 2
.
2
2
a b
b c
c a
a+b+c

Pentru primul termen din membrul stang avem:

a3 + b3
1
1
1
= (a + b) 2
+ 2
2
2
a b
a
ab b

= (a + b)

1 1
+
a b

= (a + b)

1 1
+
a b

(s-a tinut seama de faptul ca (a + b)

1 1
+
a b

1 1
+
a b

1 1
+
a b

=
ab

1 1
+
a b

4, a, b R+ ), adica

a3 + b3
1 1
+ .
2
2
a +b
a b
Sumand aceasta inegalitate cu analoagele ei, deducem

a3 + b3
b3 + c3
c3 + a3
1 1 1
+
+
2
+ +
a2 b2
b2 c2
c2 a2
a b
c

si tinand seama de relatia (a + b + c)

1 1 1
+ +
a b
c

9, obtinem (3).

Solutia 5. Putem presupune, far


a a resrange generalitatea, ca a + b + c = 1. Vom
utliza inegalitatea
1x
45x + 21, x 0
x2
121

(ntr-adev
ar, 1x 45x3 +21x2 45x3 21x2 x+1 0 (3x1)2 (5x+1) 0).
Obtinem
Xa+b

c2

X1c

c2

(45c + 21) 45(a + b + c) + 63 = 18.

Solutia 6. Sa arat
am mai ntai ca
b+c
24a + 21b + 21c

.
2
a
(a + b + c)2
Intr-adev
ar, aceasta inegalitate este echivalenta cu (b + c)3 + 2a(b + c)2 + a2 (b + c)
3
24a +21a2 (b+c)(b+c)3 2a(b+c)2 +4a(b+c)2 8a2 (b+c)12a2 (b+c)+24a3 0
(b + c 2a)[(b + c)2 + 4a(b + c) 12a2 ] 0 (b + c 2a)2 (b + c + 6a) 0, care
este adevarat
a. Avem
Xb+c

a2

X 24a + 21b + 21c

(a + b + c)2

1
18
[24(a + b + c) + 21(a + b + c) + 21(a + b + c)] =
.
(a + b + c)
a+b+c

Solutia 7. Avem succesiv:


b+c
6
(b + c)2 + a(b + c) 6a2
(b + c + 3a)(b + c 2a)

=
=
,
2
2
a
a+b+c
a (a + b + c)
a2 (a + b + c)
b+c
6
(b + c + 3a)(b a) (b + c + 3a)(c a)

=
+
.
2
a
a+b+c
a2 (a + b + c)
a2 (a + b + c)
Ca urmare, obtinem:
X b + c

a2

a+b+c

X (b + c + 3a)(b a)

(b + c + 3a)(c a)
=
=
+
a2 (a + b + c)
a2 (a + b + c)

X (b + c + 3a)(b a)
(c + a + 3b)(a b)
=
=
+
a2 (a + b + c)
b2 (a + b + c)
X (a b)(a2 c + a3 + 3a2 b b3 b2 c 3ab2 )
=
,
a2 b2 (a + b + c)

de unde
(4)

Xb+c

a2

X (a b)2 (a2 + b2 + 4ab + bc + ac)


18
1
=
,
a+b+c
a+b+c
a2 b2

din care rezulta inegalitatea din enunt.


Solutia 8. Vom demonstra inegalitatea

(5)

1
1
1
b+c c+a a+b
+ 2 + 2 4
+
+
.
a2
b
c
b+c c+a a+b
122

Apoi, aplicand inegalitatea dintre media aritmetica si cea armonica, vom avea

(6)

1
1
1
+
+
b+c c+a a+b

49
18
=
(b + c) + (c + a) + (a + b)
a+b+c

si, combin
and cu (5), vom obtine inegalitatea (2).
Revenind la (5), procedam ca n solutia precedent
a. Prin calcul stabilim ca
b+c
4
(b + c + 2a)(b a) (b + c + 2a)(c a)

=
+
2
a
b+c
a2 (b + c)
a2 (b + c)
si apoi obtinem ca
(7)

Xb+c

a2

X
1
(a b)2
= [(a + b)(b + c)(c + a) + 2abc]
2
2
b+c
a b (b + c)(c + a)

si, deci, inegalitatea (5) este adevarat


a.
Inegalitatea (5) se mai poate demonstra utilizand inegalitatea dintre media aritmetica si cea geometrica si inegalitatea xy + yz + zx x2 + y 2 + z 2 , dupa cum
urmeaza:

4
4
4
1
1
1
1 1 1
=
+
+
2 + +
2
+ +
b+c c+a a+b
a b
c
ca
bc
ab
ab + bc + ca
a2 + b2 + c2
2
2
2
=2
2
a +b +c
abc
abc
bc
ca
ab

b+c c+a a+b


1
1
1
1
1
1
a 2 + 2 +b 2 + 2 +c 2 + 2 = 2 + 2 + 2 .
b
c
c
a
a
b
a
b
c
Observatii. 1) Inegalitatile (4) si (5) sunt mai tari decat cea din problema.
2) Deoarece a2 + b2 + 4ab + bc + ac > (a + b)(a + b + c) si (a + b)(b + c)(c + a) 8abc,
din (4) si (6) si (7) obtinem urmatoarele ranari ale inegalitatii (1):
(8)

Xb+c

a2
Xb+c

a2
(9)

>

X (a b)2 (a + b)
18
+
,
a+b+c
a2 b2

X
1
(a b)2
+ 10abc

2
2
b+c
a b (b + c)(c + a)
X
18
(a b)2

+ 10abc
.
2
2
a+b+c
a b (b + c)(c + a)

Nota Redactiei. La data de 30 iunie a.c. redactia revistei a primit de la dl.


Ioan Viorel Codreanu trei solutii la problema L222. Sunt utilizate inegalitatea
mediilor, inegalitatea lui Cebsev si inegalitatea lui Bergstrom ntr-un mod diferit de
cel prezent n Solutiile 1-3 de mai sus. Regretam ca nu mai putem include si aceste
variante de rezolvare a inegalitatii (1), deoarece nr. 2/2012 era denitivat la data
primirii materialului.
123

Cum se construieste un contraexemplu


1

Temistocle BIRSAN
Abstract. In this paper we explain some aspects concerning the construction of a counterexample.
Keywords: similar triangles, circle, circumradius, inradius.
MSC 2010: 51M04.

Atunci cand avem n fata un subiect si dorim sa facem un studiu asupra lui, vom
atinge scopul punnandu-se mereu ntrebari si gasind raspunsuri pana cand rezultatele
obtinute se constutuie ntr-un sistem/teorie ce epuizeaza (de dorit!) subiectul. Pe
parcus, vom mereu n situatia de a vedea daca o armatie pe care o formulam este
adevarat
a sau falsa. Pentru a demonstra ca o armatie este adev
arat
a, deseori folosim
diverse metode si procedee cunoscute, iar uneori trebuie sa ne descurcam cu ,,forte
proprii. Pe de alta parte, pentru a dovedi ca o armatie este fals
a, adesea se indica
un contraexemplu, far
a ca acesta sa e singurul mod de a proceda.
Scopul propus este de a vedea cum se construieste un contraexemplu. Faptul
nu este lipsit de interes, caci obtinerea unui contraexemplu poate extrem de dicila.
Trebuie stiut cum sa e desprinse din contextul problemei acele indicii care conduc
n nal la contraexemplul dorit.
Vom ilustra cele spuse pe un exemplu simplu, accesibil unui elev de clasa a IX-a.
Se stie: dou
a triunghiuri sunt asemenea dac
a laturile lor corespunz
atoare sunt
proportionale.

Problem
a (formulare general
a). Inlocuind
n acest enunt laturile cu alte trei
elemente liniare: o latur
a si dou
a n
altimi, o n
altime si dou
a mediane, dou
a bisectoare
si raza cercului circumscris etc., armatia rezultat
a va adev
arat
a?
N.B. Sugeram elevilor sa ncerce un astfel de studiu si sa-l nalizeze ca o Nota
proprie pe care s-o trimita spre publicare la redactia revistei Recreatii Matematice.
Nu ns
a nainte de a citi atent randurile care urmeaza!
Problem
a (caz particular). Dou
a triunghiuri, ABC si A B C , care ndeplinesc conditiile
(1)

a
R
r
= =
a
R
r

sunt asemenea? (Notatiile sunt uzuale si se subnteleg; elementele triunghiului A B C


sunt marcate cu accente.)
R
2R sin A
R
a
= , de unde
Mai nt
ai, avem: =

a
R
2R sin A
R
(2)
1 Prof.

sin A = sin A .
dr., Univ. Tehnic
a ,,Gh. Asachi, Iasi

124

Relatia (2) impune considerarea a doua cazuri: A = A si A + A = .


Cazul I: A = A . Fie valoarea comuna rapoartelor din (1). Fie E punctul de
tangent
a cu latura AC a cercului nscris n ABC, iar E punctul cu semnicatie
similara relativ la A B C .
A
A
Se stie ca AE = p a = r ctg ; analog, A E = p a = r ctg . Cum A = A ,
2
2
AE
pa
r
pa
a
obtinem =
=

s
i,

t
in
a
nd
seama
de
(1),
deducem
c
a
= . Ca
AE
p a
r
p a
a
p
a
urmare, = din care rezulta ca
p
a
p
=
p

(3)
si

b+c
= .
b + c

(4)
Din (1) si (3) avem ca

S
pr
= = 2 , deci
S
pr
S
= 2 .
S

(5)
Cu aceste pregatiri vom arata ca

ha
la
=
= .
ha
la

(6)

ha
2S a
1
Intr-adev
ar,
=

= 2 = . Lungimile bisectoarelor sunt date de la =

ha
a 2S

2bc
A
2b c
A
la
bc b + c
S b + c
1
cos si la =
cos
,
deci
=

= 2 = .

b+c
2
b +c
2
la
bc b+c
S b+c

Relatiile (6) implica asemanarea triunghiurilor dreptunghice AHL si A H L ,


unde H si L sunt picioarele
A
inaltimii si respectiv bisectoarei
A
duse din varful A, iar H si

L au aceleasi semnicatii n

A B C . Rezulta ca HLA

H L A si apoi, ind n cazul


A = A , deducem ca ABL
A B L . In consecint
a, B = B H
C
C B H L
L
B si, cu A = A , obtinem ca ABC A B C .
Din cele demonstrate n randurile precedente, desprindem urmatoarele armatii:
Propozitia 1. Dac
a triunghiurile ABC si A B C au A = A si
ele sunt asemenea.
125

ha
la
=
, atunci
ha
la

Propozitia 2. Dac
a triunghiurile ABC si A B C sunt e ascutiunghice e obtuzunghice si ndeplinesc conditiile (1), atunci ele sunt asemenea.
Cazul al II-lea: A + A = . Am ajuns la situatia n care va nevoie de un
contraexemplu pentru a duce la capat studiul propus.
Intr-adev
ar, dupa mai multe tentative esuate de a dovedi ca triunghiurile sunt
asemena si n acest caz, apare de la sine ndoiala: nu cumva n acest caz triunghiurile
pot s
a nu e asemenea? Pentru ca ndoiala sa devina certitudine va trebuie sa
gasim un contraexemplu: s
a g
asim dou
a triunghiuri, unul ascutitunghic si unul
obtuzunghic cu A + A = si conditiile (1) ndeplinite. Vom face acest lucru, dar nu
ncerc
and sa ,,ghicim (n-am avea nicio sansa!).
In primul pas, vom face unele simplicari, pentru ca gasirea celor doua triunghiuri
sa e mai comoda. In acest sens, vom lua 1 valoarea comuna rapoartelor din (1),
adica ne propunem sa gasim doua triunghiuri, unul ascutiunghic si altul obtuzunghic,
cu A + A = , a = a , R = R , r = r (adica, sa aratam ca pot sa nu e congruente
doua triunghiuri care au congruente cate o latura, razele R si R , precum si razele r si
r ). Mai mult, nu ne multumim cu simplicarea adusa de trecerea de la ,,asemanare
la ,,congruent
a si n plus presupunem ca R = R = 1, iar cercurile circumscrise celor
doua triunghiuri coincid. Asadar, acum c
aut
am dou
a triunghiuri nscrise ntr-un cerc
de raz
a egal
a cu 1, unul ascutiunghic si altul obtuzunghic si care au razele cercurilor
nscrise lor egale.
In al doilea pas, alegem ca triunghi obtuzunghic cel mai simplu triunghi
nscris n cercul de centru O si raza egala cu 1: zicem noi ca
acesta este ABC isoscel si cu A = 120 . Suntem acum n
A
fata unei probleme relativ usoare: s
a se g
aseasc
a un punct

(arcul ce nu contine A) astfel nc


A BC
at A BC s
a
O I
ndeplineasc
a conditia r = r. Intuitia ne spune ca exista
d
un astfel de triunghi: un cerc de raza r si tangent la BC,
r

cu centrul I pe dreapta d aata la distanta r de BC (n B


C
r
I
semiplanul determinat de BC ce nu contine punctul A)
poate rostogolit pan
a cand si latura A C va tangenta
A
acestuia.
In pasul al treilea determinam riguros, printr-un calcul elementar, triunghiul A BC
gasit intuitiv n pasul precedent.

In ce priveste ABC avem b = c = 1 (raza cercului), a = 3 (latura triunghiului


s

3
3
3
3 3
3
echilateral nscris), R = 1, p = 1 +
,S=
1+
1
=
,
2
2
2
2 2
4
1
S
r = = (2 3 3).
p
2

1
Despre A BC stim ca A = 60 , a = 3, R = 1, r = (2 3 3) si urmeaza
2
A

sa determinam b si c . Utilizam formula p a = r ctg , obtinem p a =


2

3
3
1

(2 3 3) ctg 30 = (2 3) si apoi p = 3
; ca urmare, b + c = 2p a =
2
2
2

.. .
..

126

2(3 3), adica (*)


b + c = 2(3 3). Utilizand formula a b c = 4R r p , obtinem
(**) b c = 15 8 3. Relatiile (*) si (**) arata ca b , c sunt solutiile
ecuatiei

2
de gradul doi t 2(3 3)t + 15 8 3 = 0 si deci b = 3 3 + 2 3 3,

c = 3 3 2 3 3.
In concluzie, dac
a triunghiurile ABC si A B C ndeplinesc conditiile (1) si A +

A = , nu rezult
a c
a sunt asemenea, dupa cum arata perechea urmatoare
de triun


ghiuri: ABC cu a = 3, b = c = 1 si A B C cu a = 3, b = 3 3+ 2 3 3,

c = 3 3 2 3 3 (cititorul va verica cu usurinta ndeplinirea conditiilor (1)


cu = 1).
Observatie. La contraexemplul cautat s-a ajuns prin particularizari succesive si
convenabile, care au condus n nal la o problema simpla si usor de rezolvat.
Comentariu. In [1], p.16, este dat rezultatul urmator:
O conditie necesar
a si sucient
a de existent
a a unui triunghi cu elementele a, R
si r este
(E)

p
4a2 r
4R2 a2 , pentru a > 2r si 2R a.

2R
+
a2 4r2

(Se specica ca acest rezultat este luat din [2].)


Aratam ca relatiile (E) nu sunt necesare si nici suciente cu urmatoarele contraexemple:

Triunghiurile cu laturile a = 3, b = c = 1, adica ABC de mai sus, are R = 1,


1
r = (2 3 3), veric
a a > 2r si 2R a, dar nu verica prima inegalitate din (E).
2
(Si A BC poate folosit n acelasi scop au aceleasi a, R si r). Deci conditia (E)
nu este necesara.

Numerele a = 3, R = 1 si r = 3 1 verica (E), dar nu formeaza un triunghi


(caci inegalitatea Euler R 2r nu are loc); deci conditia (E) nu-i sucienta pentru
existenta unui triunghi.
Bibliografie
1.

D.S. Mitrinovi
c, J.E. Pe
cari
c, V. Volenec Recent Advances in geometric
Inequalities, Kluwer Academic Publishers, 1989.

2. G. Petrov O podmnk
ach konstrukce troj
uhelnku, Casopis
pro pestovani matematiky, 77 (1952), 77-92.

Vizitati pagina web a revistei Recreatii Matematice:

http://www.recreatiimatematice.ro
127

Institutele Unite veche scoal


a particular
a model

In Iasul celei de-a doua jumatati a sec. al XIX-lea, exista, n paralel cu nvatamantul de stat, o retea numeroasa si importanta de scoli particulare. N. Iorga n a sa
Istorie a nv
at
am
antului rom
anesc spune: Dup
a 1864, n urma ,,Legii instructiunii
publice, scoala particular
a ia un alt caracter. Ea intr
a, cu program rom
anesc si
limb
a de propunere rom
aneasc
a, pe m
ana Rom
anilor si a celor mai distinsi dintre
profesorii rom
ani. Importante date privind locul si rolul scolilor particulare din Iasi
se gasesc n monumentala carte Orasul Iasi - monograe istoric
a si social
a, ilustrat
aa
istoricului N.A. Bogdan cat si n alte surse. Dintre scolile de atunci amintim: Institutul de Domnisoare (Institutul Humpel ), Institutul Pedagogic, Sacre-Coeur, Reuniunea
Femeilor Rom
ane s.a.
Una dintre aceste scoli cu un loc de frunte n ierarhia si elita asezamintelor de
nv
at
am
ant a fost Institutele Unite, care timp de 41 de ani, ncepand cu 1866 si
pan
a n 1907, a jucat un rol important n peisajul cultural al Iasului.
In 1866 s-a nintat primul liceu particular din Iasi, Institutul Academic, care a
functionat la nceput ntr-un local din str. Sf. Haralambie. Aceasta scoala particulara
de baieti a fost nintat
a de un grup de tineri profesori de la Universitate si de la
scolile secundare din Iasi. Printre fondatorii si profesorii scolii amintim pe Grigore
Cob
alcescu, Petru Poni, Neculai Culianu, Ioan Melic, S
tefan V
argolici, la care s-au
alaturat ulterior si alti profesori de prestigiu.
In 1869, N. Culianu, P. Poni, I. Melic, P. Paicu si I. Ciurea cumpara proprietatea
logofatului Alecu Mavrocordat, mare demnitar n T
ara Moldovei, de la un descendent
al sau, Dimitrie Mavrocordat; terenul cu casele si toate constructiile aferente este
situat ntre strazile (actuale) M. Kogalniceanu, N. Balcescu si Sararie. Cladirea veche
128

a familiei Mavrocordat s-a identicat cu istoria si framantarile tarii si era cunoscuta


n epoca sub numele de ,,Curtea domneasca din Sararie , ntrucat a fost resedinta a
doi domnitori - este drept efemeri -, Alexandru Moruzi si Mihail Sutu. Tot n aceasta
casa s-a semnat la 9 ianuarie 1792 pacea de la Iasi, dupa razboiul ruso-austro-turc
din 1787-1792, prin care Rusia si extinde hotarele pana la Nistru.
Aceasta cladire veche, cu un bogat trecut istoric, va din 1869 noul local al
Institutului Academic si va ram
ane fara ntrerupere pana n zilele noastre un spatiu
dedicat nv
at
am
antului si educatiei.
In anul 1879 Institutul Academic fuzioneaza cu o alta scoala particulara similara, Liceul Nou, fondat
a n 1870, luand o noua denumire - Institutele Unite.
Acest asez
am
ant scolar de baieti cuprindea toate ciclurile de nvatamant: un curs
primar de patru ani si altul secundar (curs inferior sau gimnazial si curs superior liceul). Cursul liceal dispunea de doua sectiuni, una pentru nvatamantul clasic si alta
pentru cel modern, la care se mai adaugau clasele preparatoare destinate pregatirii
bacalaureatului si prezent
arii la examenul de admitere la Scoala Militara, ce functiona
n actuala cladire a Spitalului Militar din Iasi. Scopul acestei scoli ind buna pregatire
n vederea absolvirii examenului de bacalaureat, cat si admiterii la Liceul Militar.
Institutele Unite au avut epoca lor de glorie datorita corpului profesoral de elita
dar si foarte bunei organizari. Marturiile vremii vorbesc despre ordinea, disciplina,
spiritul de munc
a si de emulatie din randul elevilor, aceasta atmosfera prielnica studiului datorandu-se autoritatii si naltului profesionalism al dascalilor sai, remarcabile
personalitati ale vietii culturale romanesti. Dintre ilustrii profesori ai Institutelor
Unite amintim cateva nume: Titu Maiorescu - profesor de literatura si losoe, Petru
Poni - profesor de chimie si zica, Grigore Cob
alcescu - profesor de stiintele naturii,
A.D. Xenopol - profesor de istorie, S
tefan V
argolici - profesor de limba latina, Aron
Densusianu si Alexandru Philippide - profesori de limba si literatura romana, Neculai
Culianu - profesor de matematica, State Dragomir - profesor de retorica si oratorie, Grigore Mezincescu si Eduard Caudella - profesori de muzica, precum si Mihai
Eminescu - profesor de limba germana si logica. Toata aceasta pleiada de profesori au
format mintea si suetul a numeroase generatii de tineri care ulterior s-au armat n
viata sociala si culturala a t
arii. In spatiul acestui articol, ne vom referi la activitatea
a numai trei dintre ei.
Ioan M. Melik (sau Melic) este unul dintre membrii fondatori ai Institutului Academic si director al acestuia si al Institutelor Unite. A fost considerat de
catre fostii sai elevi ,,parintele intelectual si moral al multor generatii de tineri. A
dat dovada unui deosebit spirit de organizare si administrare a ntregului proces de
nvatam
ant. Preciziunea masurilor luate, punctualitatea, perseverenta si constiinciozitatea au fost principii de baza n ntreaga sa activitate de director. A fost profesor de
calcul diferential si integral la Facultatea de Stiinte a Universitatii din Iasi si a facut
parte din Junimea.
Neculai Culianu, membru fondator al Institutului Academic, a fost o personalitate proeminent
a a Iasului cultural de odinioara. Ca profesor si director la Institutele
Unite s-a impus prin seriozitate, nalta tinuta intelectuala, modestie si generozitate.
A fost decan al Facult
atii de Stiinte si rector al Universitatii din Iasi si membru corespondent al Academiei Romane. A facut parte din societatea Junimea. Este fondator
129

al revistei Recreatii S
tiintice, prima revista stiintica din tara adresata tineretului.
Mihai Eminescu a fost profesor de limba germana doar un semestru, n anul
scolar 1874-1875, dar prin prestigiul pe care-l avea se bucura de stima si repectul
tuturor. In acelasi timp preda si orele de logica tinute de A.D. Xenopol, alcatuind
totodata si cursul pe care-l tinea n lipsa manualului scolar.
Ca profesor revarsa un farmec deosebit. Forta lui de patrundere se mbina cu
bogatia de idei si cu modul cum stia sa le aduca la puterea de ntelegere a auditorilor. Marturiile vremii spun ca era deosebit de sever cu elevii, n special cu notarea
acestora, ceea ce a generat un incident cu acestia n mai 1875 - povestit cu mult haz
de memorialistul Rudolf Sutu n atractiva sa monograe Iasii de odinioar
a. Printre
autorii acestui episod se numara si elevii Constantin Mille si Gheorghe Mortun, care
peste ani vor avea un rol important n publicistica ieseana de stanga.
Multi dintre cei care au urmat cursurile acestei scoli model au devenit ulterior,
datorita culturii solide cap
atate, personalitati importante n viata culturala si sociala
a t
arii. Mention
am doar pe cativa dintre ei: A.D. Xenopol - istoric, State Dragomir
- actor si profesor la Institutul de Arte Dramatice din Iasi, Constantin Buzdugan
- magistrat (a fost unul din cei trei regenti din Romania n perioada 1927-1930),
Gheorghe si Grigore Butureanu - profesori, Mihai Carp - profesor de limba romana
la Liceul Internat (azi Colegiul National ,,Costache Negruzzi) si multi alti profesori
de mare prestigiu la diferite licee din Iasi.
D
and Iasului si t
arii asemenea absolventi timp de 41 de ani, aceasta scoala a fost
un centru luminos de cultura, servind drept pilda tuturor scolilor similare nintate
dupa 1866 n Iasi si n ntreaga tara. Un astfel de exemplu ni-l ofera Institutul de
Domnisoare (Institutul Humpel ), pension de fete patronat de ,,Societatea ortodoxa
a femeilor romane, nintat n 1872 de profesorii asociati ai Institutului Academic.
Aici se faceau studii primare si liceale, directoare ind Emilia Humpel, sora lui Titu
Maiorescu, deosebit de competenta si autoritara - profesor de limba si literatura
roman
a.
Dob
andind n cei 41 de ani de existenta statutul de adevarat centru al nvatamantului preuniversitar iesean de tip privat, Institutele Unite, luminand mintea si
cugetul a numeroase generatii, au contribuit la prestigiul cultural al Iasului.
,,Curtea domneasca din Sararie, devenita local de scoala pentru Institutele Unite
prin actul caritabil al unor inimosi carturari ai Iasului, este nchiriata n 1909 de
S
coala Secundar
a de Fete ,,Oltea Doamna, al carei local nou din incinta Manastirii
Golia fusese distrus de un incendiu chiar n acel an. Se deschide astfel o noua pagina
a istoriei acestui asez
am
ant. In 1920 cladirea devine proprietatea noii institutii prin
straduinta vrednicei sale directoare, Tereza Str
atilescu. La vechea cladire, consolidata
si restaurata pe parcursul anilor, s-au adaugat noi spatii de scolarizare si camine.
Insusi numele acestei institutii scolare trece prin mai multe schimbari, potrivit cu
vremurile. Astazi, pe aceste vechi locuri se ridica Colegiul National ,,Mihai Eminescu
care, mandru de trecutul devenirii sale, este ancorat si implicat n prezent cu un corp
profesoral competent, garantie sigura a succeselor care vor urma.

Prof. Olga DIACONU


Colegiul National ,,Mihai Eminescu, Iasi
130

Concursul de matematic
a Florica T. C
ampan
Editia a XII-a, Iasi, 2012
Clasa I
I.1. Vom scrie anul 2012 folosind cifrele urmatoare:

Cate bete de chibrit vom utiliza?


I.2. Care este cel mai mic num
ar impar de doua cifre care are suma cifrelor 10?
I.3. C
ate scanduri lipsesc din gard?

II.1. Martinica cel istet intr


a n camara si mananca toata dulceata. Rezolvand
cerintele urmatoare veti aa ce fel de dulceata a mancat Martinica.
a) Aati numerele notate cu litere.
b) Asezati rezultatele crescator.
c) Asezati literele sub numerele astfel ordonate si veti aa ce fel de dulceata a
mancat Martinica.
1+r
3+u
6+e
11
11-z
a-4

14-m
II.2. Ana are niste oua de ciocolata. Mananca doua, ofera celor doua prietene
cate doua si i mai ram
an 13. Cate oua de ciocolata a avut Ana?
III. Pink primeste un joc de lego ce contine 18 rotite. El construieste biciclete
si masinute folosind toate rotitele din joc. Cate biciclete si cate masinute poate
construi acesta stiind ca el a construit cel putin o bicicleta si o masinuta? Gasiti
toate variantele.

Clasa a II-a
I.1. Descopera regula, apoi completeaza sirurile cu nca 4 numere:
a) 0; 2; 6; 12; . . . ; . . . ; . . . ; . . . .
131

b) 90; 81; 72; 63; . . . ; . . . ; . . . ; . . . .


I.2. A
a num
arul care micsorat cu 15 este mai mic cu 36 decat suma numerelor
48 si 25.
II.1. C
ate car
amizi lipsesc din zidul alaturat?

II.2. Florica are un sifonier cu 4 usi. Ea a ncurcat cele 4 chei cu ajutorul carora
le-a ncuiat. Care este num
arul maxim de ncercari pe care trebuie sa le faca Florica
pentru a descuia toate usile sifonierului?
III.1. Bianca si Andrei calatoresc cu trenul. Andrei are loc n al saptelea vagon
din fat
a, iar Bianca n al noualea vagon din spate. Ei au observat ca vor calatori n
acelasi vagon. Cate vagoane are trenul?
III.2. Un magician are n jobenul sau 4 jetoane pe care sunt scrise numerele
10, 11, 12 si 13. Cu ajutorul baghetei sale face urmatoarea magie: nlocuieste numerele
pare cu predecesoarele lor, iar pe cele impare cu succesoarele lor. Repeta magia de
mai multe ori asupra numerelor obtinute de ecare data.
Aati de cate ori apare scrierea initiala a numerelor ntre magiile 101 si 130.

Clasa a III-a
I.1. La ora de matematica copiii au masurat cu pasul lungimea terenului de
baschet. Paula a obtinut 25 de pasi, Ana 23, Ionut 20, iar Diana 26. Care dintre copii
are pasul cel mai mare?
I.2. Ilinca s-a nascut n anul 2000. Sora ei are acum 7 ani. Care este diferenta de
varst
a dintre ele?
II.1. Fat-Frumos se lupta cu un balaur cu noua capete. De ecare data cand
Fat-Frumos taie un cap al balaurului, acestuia i cresc n loc alte trei capete. Cate
capete are balaurul, dupa ce Fat-Frumos i-a taiat sase capete?
II.2. Fie a si b dou
a numere consecutive. Suma dintre aceste numere si vecinii
lor mariti cu 12 ecare este 939. Aati numerele a si b.
III.1. Cu ajutorul a doua drepte mpartiti cadranul unui ceas n trei parti n asa
fel nc
at adunand numerele din ecare parte sa obtineti aceeasi suma.
III.2. O carte are rupte mai multe foi consecutive. Prima pagina, de pe prima
foaie rupta, are num
arul 163, iar ultima pagina are numarul format din aceleasi cifre.
Pot mp
artite foile rupte n grupe de cate trei?

Clasa a IV-a
I.1. Daca 11 iepurasi de ciocolata costa cat 12 oua de ciocolata, iar 12 iepurasi
costa cu 46 lei mai mult decat 11 oua, se cere:
132

Cat costa 23 de iepurasi si 23 oua de ciocolata? Cu cat costa mai mult un iepuras
decat un ou de ciocolata?
I.2. Un baschetbalist la antrenament face 4 pasi nainte si apoi 3 pasi napoi. Daca
ecare pas masoar
a un metru, dupa cati pasi el a parcurs 200 metri de la punctul de
plecare?
II. Primii 15 termeni ai unui sir sunt: 1, 2, 0, 3, 4, 1, 5, 6, 2, 7, 8, 0, 9, 10, 1, . . .. Scrieti
urmatoarele trei numere din sir, apoi calculati suma primilor 111 termeni ai sirului.
III. Suma dintre catul si restul unei mpartiri a doua numere naturale este 53
iar restul depaseste cu 1 triplul catului. Aati demp
artitul stiind ca este un numar
natural cuprins ntre 740 si 750.

Clasa a V-a
I. Consideram multimea A = {x|x = 2m 3n , m, n N}.
a) Gasiti doua elemente din multimea A astfel ncat produsul lor sa e patrat
perfect.
b) Gasiti doua elemente din multimea A astfel ncat produsul lor sa nu e patrat
perfect.
c) Aratati ca din oricare cinci elemente ale multimii A putem alege doua al caror
produs este patrat perfect.
II. Pe tabla sunt scrise numerele 1, 2, 3, 6, 7, 9, 10, 23, 48. Dan si Ana au sters
ecare cate patru numere si au observat ca suma numerelor sterse de Dan este de
patru ori mai mare decat suma numerelor sterse de Ana.
a) Ce num
ar a ramas pe tabla?
b) Ce numere a sters ecare copil?
III. Ionel si Vasile au cumparat tricouri identice din doua magazine. Fiecare a
protat de oferta promotional
a a magazinului, ncercand sa obtina cel mai bun pret
mediu pentru un tricou. Magazinul de unde a cumparat Ionel avea oferta 2 + 1
gratis (la doua tricouri cumparate primesti nca unul gratis). Magazinul de unde a
cumparat Vasile avea oferta 3 + 1 gratis. Cei doi copii au constatat ca au cheltuit
aceeasi suma totala de bani si au n nal acelasi numar de tricouri.
a) Aratati ca num
arul de tricouri pe care le are n nal ecare copil este divizibil
prin 12.
b) Aati ce preturi au asat cele doua magazine pentru un tricou, stiind ca la
magazinul de unde a cumparat Ionel pretul era cu 2 lei mai mare decat la magazinul
de unde a cumparat Vasile.
Ciprian Baghiu

Clasa a VI-a
I. a) Exista numere rationale si nentregi, notate a, b, c, d, cu proprietatea ca ab
c
Z si Z?
d
b) Dati exemple de numere x, y, z, t Q\Z astfel ncat x + y Z si z t Z.
c) Exista a, b Q\Z astfel nc
at a + b Z si a b Z?
Cristian Laz
ar
133

II. Ioana pleaca din punctul A spre punctul B, apoi, si continua drumul catre
C si, la nal, decide sa se ntoarca n A. Punctele A, B, C sunt necoliniare. Se stie
ca drumurile ntre doua puncte sunt n linie dreapta si ca AB, BC, respectiv CA, au
lungimi (n kilometri) exprimate prin numere naturale nenule.
a) Daca AB + BC + CA = 5 km, demonstrati ca triunghiul ABC este isoscel.
b) Daca AB + BC + CA = 6 km, demonstrati ca triunghiul ABC este echilateral.
c) Determinati cel mai mic numar natural nenul n pentru care AB +BC +CA = n
km si este posibil ca Ioana sa parcurga un drum care nu reprezinta un triunghi isoscel.
Cristian Laz
ar
III. Se stie ca A = aabbcccc este patrat perfect si a = b = c = a.
a) Aratati ca 121|A.
b) Determinati toate numerele A cu proprietatea de mai sus.
Cristian Laz
ar

Clasa a VII-a
I. Se considera 40 de carti de joc: patru cu valoarea 1, patru cu valoarea 2, . . .,
patru cu valoarea 10. Toate cartile se mpart aleator, n mod egal, ntre doi jucatori.
Far
a a se uita la carti, pe rand, ecare asaza cate o carte pe masa, cu fata n sus.
Daca la un moment dat unul dintre jucatori observa pe masa cateva carti cu suma
15, el poate elimina din joc acel grup de carti. Castiga cel care a eliminat mai multe
astfel de grupe.
Bianca si Ioana joaca acest joc. Spre nal, pe masa ramane o singura carte, cu
valoarea 9. Bianca mai are n mana doua carti avand valorile 3 si 5, iar Ioana are n
man
a o singura carte. Ce valoare are cartea din mana Ioanei?
Dorel Luchian
2x + 1
y+2
II. Fie numerele rationale f1 = 2
si f2 =
, unde x Z, yZ\{2}.
2y + 5y + 2
3x 2
a) Demonstrati ca, daca y este numar par, atunci f1 nu este numar ntreg.
b) Determinati numerele ntregi x si y pentru care fractiile f1 si f2 sunt, simultan,
numere ntregi.
Claudiu-S
tefan Popa
III. Se considera trapezul ABCD cu baza mare (AB). Punctul E apartine laturii
(AD) astfel nc
at (AE) (CD). Paralela la dreapta AD prin punctul de intersectie a
diagonalelor trapezului intersecteaza dreapta BE n punctul F . Demonstrati ca (AF
este bisectoarea interioar
a a unghiului BAD.
Claudiu-S
tefan Popa

Clasa a VIII-a
I. Un raliu se desfasoara pe un traseu cu lungimea de 1000 km. Studiind harta,
un pilot decide sa parcurga distanta n patru etape, egale ca timp. In ecare etapa,
ncep
and cu a doua, viteza automobilului creste cu 20 km/h fata de etapa anterioara.
Pilotul reuseste sa-si respecte planul, terminand raliul n 8 ore.
a) Care a fost viteza masinii n prima etapa?
b) Aati distantele parcurse n ecare etapa.
134

c) La nal, seful de echip


a prezinta un grac n care este schematizata distanta
parcursa n functie de timp. Cum arata acest grac? (Luati ca unitati de masura
1 cm pentru o ora pe axa timpului, respectiv 1 cm pentru 100 km parcursi pe axa
distantei.)
Marian Pantiruc
II. Se dau patru cuburi, av
and muchiile de 1 cm, 2 cm, 3 cm, respectiv 5 cm.
Denisa lipeste cele patru cuburi unul de altul (daca doua cuburi sunt lipite, o fata a
unuia este inclusa ntr-o fat
a a celuilalt) si calculeaza aria corpului astfel obtinut.
a) Care este aria maxima pe care o poate obtine Denisa?
b) Care este aria minima pe care o poate obtine Denisa?
Cristian Vntur si Gabriel Popa
III. Se coloreaza ecare patr
atel 11 dintr-un careu 44 cu una din culorile rosu,
verde, galben, albastru, astfel nc
at daca doua patratele au macar un punct comun,
atunci ele au culori diferite.
a) Determinati o astfel de colorare.
b) Aati cate modalitati de colorare exista.
Silviu Boga

Decalogul scolarilor
In Anuarul S
colii Normale ,,Vasile Lupu din Iasi pe anul scolar 1929-1930 (nr.5,
1931) este publicat decalogul scolarilor - un proiect semnat de Radu Manoliu.
In acea perioada, nv
at
am
antul romanesc se aa n momentele sale cele mai bune,
iar S
coala Normal
a ,,Vasile Lupu era scoala model pentru scolile de acelasi tip din
tara. Acest proiect si propune sa dea ,,cuvenita atentie poruncilor morale, religioase,
sociale, igenice si estetice ntr-o forma concisa care sa permita tiparirea lui cu litere
mari, ncadrarea si asarea pe peretii claselor.
Proiectul si mentine interesul si actualitatea si n epoca noastra a globaliz
arii
dar si a p
astr
arii si arm
arii identit
atii nationale.
M
a leg:
1. S
a-mi tin cuv
antul dat.
2. S
a ascult cu bun
avoint
a si cuviint
a de cei ce m
a-ngrijesc si se str
aduiesc s
a-mi
dea nv
at
atur
a si bun
a crestere.
3. S
a nu t
ainuiesc adev
arul.
4. S
a fiu drept si cu mine si cu altii, m
asurndu-mi vorbele si faptele.
5. S
a crut pe cei ce sufer
a si s
a ajut, pe c
at pot, pe cei c
azuti prin nedreptatea
soartei.
a fiu curat la port si la vorb
a.
6. S
7. S
a fiu ordonat si st
aruitor la munc
a.
8. S
a cinstesc obiceiurile str
amosesti si legile t
arii mele.
a-mi iubesc T
ara si Neamul meu ca pe mine nsumi.
9. S
10. S
a-mi amintesc c
a viata nu ncepe si nu se sf
arseste cu mine si c
a este
o r
asplat
a pentru toti si toate.
135

Solutiile problemelor propuse n nr. 1/2012


Clasele primare
P.226. Scrieti vecinii num
arului care rezult
a din compunerea al
a- 1
2
turat
a.
(Clasa I )
Mihaela Cucoranu, elev
a, Iasi 2
5
Solutie. Prin compunerea numerelor 1 si 2 obtinem numarul 3, iar
prin compunerea numerelor 2, 3 si 5 obtinem numarul 10. Vecinii numarului 10 sunt
9 si 11.
P.227. La plecare n vacant
a, trei elevi au convenit s
a-si trimit
a felicit
ari: ecare
s
a trimit
a o singur
a felicitare la unul dintre ceilalti doi. Este posibil ca unul dintre
elevi s
a primeasc
a felicitare de la elevul c
aruia el nsusi i-a trimis felicitare?
(Clasa I )
Lavinia Dasc
alu, elev
a, Iasi
Solutie. Este posibil. De exemplu, elevii E1 si E2 si trimit felicitari reciproc,
iar elevul E3 poate trimite felicitare elevului E1 sau elevului E2.
P.228. Priviti cu atentie exercitiul de mai jos:

3 + 7+
20

= 100

60

Calculati: a)
+
b) 100 .
(Clasa I )
S
tefania Gavril, elev
a, Iasi
Solutie. 3 +
= 20
= 20 3 = 10 + 10 3 = 10 + 7 = 17; 7 +
=
60
= 60 7 = 50 + 10 7 = 50 + 3 = 53; 20 + 60 + = 100 80 + =
100 = 100 80 = 20.
a)
+
= 53 + 17 = 50 + 10 + 3 + 7 = 60 + 10 = 70;
b) 100 = 100 20 = 80.
P.229.
Intr-o cutie sunt 17 bile albe si 19 bile negre. Sorin ia la nt
amplare 5
bile. C
ate bile de ecare culoare r
am
an n cutie?
(Clasa a II-a)
Inst. Maria Racu, Iasi
bile albe ramase 17 16 15 14 13 12
Solutie.
bile negre ramase 14 15 16 17 18 19
P.230. Ar
atati c
a oricum am lua sase numere din sirul 11, 12, 13, . . . , 20, exist
a
dou
a care au suma 31.
(Clasa a II-a)
Mihaela G
alc
a, elev
a, Iasi
Solutie. Avem 11+20 = 31, 12+19 = 31, 13+18 = 31, 14+17 = 31, 15+16 = 31.
In cel mai nefavorabil caz, luam cate un termen din ecare adunare. Al saselea numar
va al doilea termen al unei adunari si existenta este justicata.
136

P.231. Asupra numerelor 10, 11, 12 si 13 se efectueaz


a operatia urm
atoare: numerele pare se nlocuiesc cu predecesorii lor si cele impare se nlocuiesc cu succesorii
lor.
In al doilea pas se repet
a aceast
a operatie asupra rezultatului obtinut; se continu
a n acelasi fel n pasii urm
atori. Aati de c
ate ori se repet
a scrierea initial
aa
numerelor ntre pasii 101 si 230 ai sirului de operatii.
(Clasa a II-a)
Paula Zaharia, elev
a, Iasi
Solutie. Se observa ca n pasii de ordin par numerele se scriu exact ca n forma
initiala. Intre pasii 101 si 230, pasii de ordin par sunt: 102, 104, 106, . . . , 228 care sunt
n numar de 64.
P.232. Cei sase membri ai unei echipe care particip
a la un concurs de matematic
a
au v
arste diferite, de cel putin 10 ani si cel mult 15 ani.
In timpul concursului membrii
echipei s-au asezat la mas
a n ordinea v
arstelor. S
a se ae ce v
arst
a are ecare stiind
c
a Ioana este cea mai mic
a, Anca este cea mai mare, Bogdan se a
a l
ang
a Ioana si
nu se a
a l
ang
a Bianca si Andrei, iar Alexandra se a
a ntre doi b
aieti.
(Clasa a III-a)
Constanta Tudorache si Nelu Tudorache, Iasi
Solutie. Ordinea asez
arii, n ordinea crescatoare a varstelor, este: Ioana, Bogdan,
Alexandru, Andrei, Bianca si Anca. Cum varstele sunt diferite, iar de la 10 la 15 ani
sunt 6 numere, urmeaza ca cel mai mic are 10 ani, iar cel mai mare 15 ani. Ceilalti
membri au 11, 12, 13 si respectiv 14 ani.
P.233. Dac
a a b = 441 si b se mparte exact la a, determinati a si b.
(Clasa a III-a)
Tatiana Ignat, elev
a, Iasi
Solutie. Daca b se mparte exact la a, atunci 441 se mparte exact la a a. Cum
441 = 1 3 3 7 7, avem cazurile: 1) a = 1, b = 441; 2) a = 3, b = 147; 3) a = 7,
b = 63 si 4) a = 21, b = 21.
P.234. Aati numerele naturale a si b astfel nc
at (a + 2) : (b + 1) = a.
(Clasa a III-a)
Codruta Filip, elev
a, Iasi
Solutie. Nu putem avea a = 0 sau b = 0. Atunci a + 2 = (b + 1)a si avem cazurile:
b = 1, a = 2; b = 2, a = 1. Pentru b 3 nu avem solutii.
P.235. O carte are rupte mai multe foi consecutive. Prima pagin
a de pe prima
foaie rupt
a are num
arul 163, iar ultima pagin
a are num
arul format din aceleasi cifre.
Pot mp
artite foile rupte n grupe de c
ate 3?
(Clasa a III-a)
Andreea Bzdg
a, elev
a, Iasi
Solutie. Pe ultima pagina trebuie sa e un numar par, mai mare ca 163, deci 316.
Numarul total de pagini este 316 163 + 1 = 154, iar numarul foilor este 154 : 2 = 77.
Cum 77 = 3 25 + 2, deducem ca foile rupte nu pot mpartite n grupe de cate 3.
dou
P.236. In
a cutii sunt mingi de tenis, n prima ind de dou
a ori mai multe
dec
at n a doua. Dac
a din prima cutie se scot 30 de mingi si din a doua 20, atunci
n prima cutie r
am
an de trei ori mai multe mingi dec
at n a doua. C
ate mingi sunt
n ecare cutie?

(Clasa a IV-a)
Inv. Petru Miron, Pascani
Solutie. Fie 2a, respectiv a num
arul mingilor din cele doua cutii. Din 2a 30 =
3(a 20) obtinem ca a = 30, prin urmare n cele doua cutii se aa 60, respectiv 30
de mingi.
137


P.237. Intr-o
camer
a sunt scaune cu 3 picioare si cu 4 picioare. C
and toate
scaunele sunt ocupate, num
arul picioarelor din camer
a este 39. C
ate scaune cu 4
picioare sunt n camer
a?
(Clasa a IV-a)
Inst. Laura Chiril
a, Iasi
Solutie. Dac
a avem a scaune cu 3 picioare si b scaune cu 4 picioare si toate
scaunele sunt ocupate, numarul total de picioare este 5a + 6b. Cautam a si b astfel
nc
at 5a + 6b = 39. Deoarece 39 si 6b se mpart exact la 3 atunci si 5a se mparte
exact la 3, deci a se mparte exact la 3. Unica solutie este: a = 3 si b = 4.
P.238. Aati numerele care se m
aresc de 11 ori prin ad
augarea unei cifre diferite
de zero la sf
arsitul lor.
(Clasa a IV-a)
Nicoleta Cump
at
a, elev
a, Iasi
Solutie. Daca num
arul are cel putin doua cifre, atunci 10 ab + c = 11 ab, de
unde ab = c, c ind cifra adaugata. Rezulta ca numerele cautate au o singura cifra.
Vom avea 10 a + c = 11a, ceea ce implica a = c, a = 0. Numerele care ndeplinesc
conditia problemei sunt 1, 2, 3, . . . , 9.
P.239. Dreptunghiul al
aturat reprezint
a o gr
adin
a care este format
a din dou
a alei si patru straturi dreptunghiulare egale. Aati
dimensiunile gr
adinii stiind c
a l
atimea ec
arei alei este de 2m,
diferenta dintre dimensiunile unui strat este de 1m, iar lungimea
2
unui strat reprezint
a din lungimea gr
adinii.
5
(Clasa a IV-a)
Petru Asaftei, Iasi
4
Solutie. Suma lungimilor a doua straturi reprezinta din lungimea gradinii, deci
5
1
2m reprezint
a din lungimea gradinii, de unde deducem ca lungimea gradinii este de
5
10m, iar lungimea ecarui strat este de 4m. Latimea gradinii este 2(41)+2 = 8m.

Clasa a V-a
V.144. Aati numerele naturale abc cu proprietatea c
a abc = a + 19b + 10c.
Nicolae Iv
aschescu, Craiova
Solutie. Cum abc = 100a + 10b + c, reducand termenii si mpartind prin 9, din
ipoteza deducem ca 11a = b + c. Vom avea obligatoriu ca a = 1, b + c = 11, deci
abc {129, 138, 147, 156, 165, 174, 183, 192}.
V.145. Demonstrati c
a num
arul A = 21 + 22 + 23 + . . . + 22012 nu este p
atrat
perfect.
Anca Chiritescu, T
ig
anasi (Iasi)
Solutie. Cum A se divide cu 2, dar nu se divide cu 4, rezulta ca nu este patrat
perfect.
V.146. Se consider
a multimea A = {abc | a, c cifre pare, b cifr
a impar
a}. Determinati cardinalul lui A si suma elementelor lui A.
Bogdan Chiriac, student, Iasi

Solutie. Intruc
at a {2, 4, 6, 8}, b {1, 3, 5, 7, 9}, c {0, 2, 4, 5, 8}, rezulta ca
|A| = 4 5 5 = 100. Pe pozitia sutelor, ecare dintre cifrele 2, 4, 6 si 8 apare de cate
25 de ori; pe pozitia zecilor, ecare cifra impara apare de cate 20 de ori, iar pe pozitia
138

unitatilor ecare cifra para apare de cate 20 de ori. Suma elementelor lui A va
100 25(2 + 4 + 6 + 8) + 20 20(1 + 3 + 5 + 4 + 9) + 20(0 + 2 + 4 + 6 + 8) = 55 400.
V.147. Putem aseza pe un cerc numerele 1, 2, 3, . . . , 2012 astfel nc
at suma oric
aror
patru numere consecutive s
a se divid
a cu 5?
Gheorghe Iurea, Iasi
Solutie. Daca, prin absurd, ar exista o astfel de asezare, e aceasta a1 , a2 , . . . , a2012 ,
.
.
.
atunci din a1 + a2 + a3 + a4 ..5, a2 + a3 + a4 + a5 ..5, . . . , a2012 + a1 + a2 + a3 ..5 ar rezulta,
.
.
prin sumare, ca 4(a1 + a2 + . . . + a2012 )..5, adica 4(1 + 2 + . . . + 2012)..5, ceea ce este
fals.
2371 + 26 3237
3371 + 124 5247
V.148. Ar
atati c
a
<
.
3240
5250
Petru Asaftei, Iasi
Solutie. Vom arata ca prima fractie este subunitara, iar a doua este supraunitara.
Avem: 2371 + 26 3237 = (211 )33 28 +26 3237 < (37 )33 36 + 26 3237 = 3237 + 26 3237 =
27 3237 = 3240 , iar 3371 + 124 5247 = (39 )41 32 + 124 5247 > (56 )41 5 + 124 5247 =
5247 + 124 5247 = 125 5247 = 5250 si astfel solutia problemei este completa.
V.149. Demonstrati c
a 6n (n N) nu poate scris ca sum
a de dou
a p
atrate
perfecte nenule.
Elena Iurea, Iasi
Solutie. Pentru n 2, sa presupunem ca 6n = a2 + b2 , cu a, b N . Se arata
usor ca a, b sunt pare si divizibile cu 3, deci a = 6x, b = 6y, cu x, y N . Rezulta
ca 6n2 = x2 + y 2 , cu x, y N . Continuand rationamentul obtinem ca 6n4 , 6n6
etc. se pot scrie ca suma de doua patrate perfecte nenule. In nal, 60 sau 61 vor avea
aceasta proprietate, ceea ce este evident fals.
V.150. Despre un num
ar natural a1 a2 . . . an spunem c
a este num
ar bun dac
a
exist
a o innitate de p
atrate perfecte care au suma cifrelor egal
a cu an1 an . Ar
atati
c
a 2012 nu este num
ar bun, ns
a 2013 este num
ar bun.
Cristian Laz
ar, Iasi
Solutie. Un patrat perfect nu poate avea suma cifrelor 12, deoarece 12 = M9 + 3
si un numar de forma M9 + 3 contine n descompunerea sa n factori primi factorul 31 ,
deci 2012 nu este num
ar bun. Deoarece numerele de forma 49 |00 {z
. . . 0} sunt patrate
2n zerouri

perfecte si au suma cifrelor 13, rezulta ca 2012 este numar bun.

Clasa a VI-a
VI.144. Fie p un num
ar prim impar. Ar
atati c
a exist
a un singur num
ar natural
nenul k pentru care p2 + k 2 este p
atrat perfect.
Marian Pantiruc, Iasi
Solutie. Dac
a p2 + k 2 = n2 , cu n N , atunci (n k)(n + k) = p2 . Cum p este
p2 1
prim, rezulta ca n k = 1, n + k = p2 , deci k =
N este singurul numar cu
2
proprietatile din enunt.
VI.145. Fie a N, a 2 si multimea A = {x Z|a a2 + 1 x a + a2 + 1}.
Determinati cardinalul lui A si suma elementelor din A.
Ionel Nechifor, Iasi
139

Solutie. Multimea A contine a2 a 1 elemente negative, a2 + a + 1 elemente


pozitive si l contine si pe 0, prin urmare |A| = (a2 a 1) + (a2 + a + 1) + 1 = 2a2 + 1.
Cand calculam suma elementelor lui A, cele a2 a 1 numere negative se reduc cu
primele a2 a 1 numere pozitive, deci ramane de aat suma S = (a2 a) + (a2
a + 1) + . . . + (a2 + a + 1). Excluzandu-l pe a2 + a + 1, cei 2a + 1 termeni ramasi
sunt simetrici fat
a de a2 , asadar suma lor este (2a + 1) a2 . In nal, obtinem ca
2
S = (2a + 1)a + (a2 + a + 1) = (a + 1)(2a2 + 1).
VI.146. Se pot mp
arti numerele 1, 2, 3, . . . , 2012 n c
ateva submultimi disjuncte
astfel nc
at cel mai mare element al ec
arei submultimi s
a e egal cu produsul celorlalte
elemente ale respectivei submultimi?
Mihai Cr
aciun, Pascani
Solutie. Daca o astfel de mpartire ar posibila, produsul elementelor ecarei
submutimi ar egal cu patratul celui mai mare element al ei, deci 2012! ar patrat
perfect. Insa 2012! se divide cu numarul prim 2011 si nu se divide cu 20112 , asadar
nu este patrat perfect. Rezulta ca mpartirea ceruta n problema nu este posibila.
VI.147. G
asiti dou
a numere rationale mai mari dec
at 40 al c
aror produs s
a e
2012, ecare dintre ele av
and c
ate o innitate de zecimale nenule.
Cristian Laz
ar, Iasi
4 72 503 7
Solutie. De exemplu, 2012 =

= 41, (142857) 48, 902(7).


7
72
a
VI.148. Determinati fractiile ireductibile
care se scriu sub form
a zecimal
a ca
b
fractii periodice, cu zecimala de pe pozitia b egal
a cu b.
Gabriel Popa, Iasi
a
Solutie. Cum b este cifra si
fractie periodica, rezulta ca b {3, 6, 7, 9}. Obb
a0
servam ca este sucient sa determinam fractiile subunitare ireductibile
, solutia
b
a0 + nb
, cu n N. Daca b = 3, atunci a0 {1, 2}; singura
generala a problemei ind
b
a0
1
solutie convenabil
a este
=
= 0, 3333 . . .. Daca b = 6, atunci a0 {1, 5};
b
3
a0
1
singura solutie convenabila este
=
= 0, 1666666 . . .. Daca b = 7, atunci
b
6
a0
5
a0 {1, 2, 3, 4, 5, 6}; singura solutie convenabila este
= = 0, 7142857142857 . . ..
b
7
a0
= 0, (a0 ), cu a0 = 9, nu obtinem
Daca b = 9, atunci a0 {1, 2, 4, 5, 7, 8} si, cum
9
a
3n + 1 6n + 1 7n + 5
noi solutii. In concluzie, {
,
,
n N}.
b
3
6
7
= 3m(C).
Fie M, N AC astfel
VI.149. Se consider
a triunghiul ABC cu m(B)

nc
at ABM M BN N BC si AP BN, cu P BN ; not
am {I} = BM AP.

Demonstrati c
a N I este bisectoarea unghiului AN
B
Nicolae Iv
aschescu, Craiova

Solutie.

Cum AN
B este unghi exterior triunghiului isoscel N BC, rezulta

140

= m(ABN
), deci
M
ca m(AN
B) = 2 m(C)
ABN este isoscel. Inaltimea bazei, AP , va
N
I
si bisectare, prin urmare I este intersectia a
doua bisectoare din ABN . Urmeaza de aici
P

ca N I este bisectoarea unghiului AN


B.
C
VI.150. Fie ABC un triunghi. Not
am cu B
D punctul de intersectie dintre perpendiculara n B pe BC si mediatoarea laturii AB
si cu E punctul de intersectie dintre perpendiculara n C pe BC si mediatoarea laturii
caculati m(DAE)
n functie de .
AC. Dac
a = m(BAC),
Adrian Zanoschi, Iasi

este
Solutie. I. Daca < 90 , deosebim situatiile: (i) B, C sunt ascutite; (ii) B

obtuz; (iii) B este drept. Fie AM n


altimea din A a triunghiului. In cazul (i), avem ca

A
BAM ABD DAB si M AC
A
E

E
ACE EAC, deci m(DAE) = 2
) + 2m(CAM
) = 2.
m(BAM
In cazul
D
D

DAB

(ii), avem ca M
AB ABD
si

EAC,
deci m(DAE)
=
M
AC ACE
m(BAD)
= + m(CAE)
B
m(BAE)
C
C
M
M B

m(BAD) = + (90 C) (B 90 ) =
C)
= 2.
= 2. La fel se
+ (180 B
In cazul (iii), este evident ca m(DAE)


procedeaza n cazurile: (ii ) C este obtuz; (iii ) C este drept.
= 180 .
II. Daca = 90 , atunci m(DAE)
= 360 2.
III. Daca > 90 , atunci m(DAE)

Clasa a VII-a
VII.144. Fie ABCD un trapez cu ABCD si AB=3 CD. Dac
a E si F sunt
simetricele punctelor B si A fat
a de C, respectiv D, ar
atati c
a CDEF este paralelogram.
Eugeniu Bl
ajut, Bac
au
Solutie. Fie {O} = AD BC; din asemanari imediate, obtinem ca AD = 2DO,
E
iar BC = 2CO. Intruc
at AD = DF si BC = CE, rezulta
F
ca OD = OF si OC = OE, adica CDEF este paraleloO
gram.
D
C
VII.145.
In triunghiul ABC, se consider
a mediana
AD si bisectoarea CE. Not
am {P } = AD CE si {F } =
B
P B AC. Demonstrati c
a triunghiul EF C este isoscel. A
Valentina Blendea si Gheorghe Blendea, Iasi
AF DC BE
AF
EA
Solutie. Din teorema lui Ceva

= 1, obtinem ca
=
,
F C BD EA
FC
EB
ECB
(alterne interne); ns
ECF
,
prin urmare EF BC. Rezulta ca CEF
a ECB
ECF
, adic
asadar CEF
a CEF este isoscel cu baza CE.
141

VII.146. Fie H ortocentrul triunghiului ascutiunghic ABC si D, E, F intersectiile


dreptelor AH, BH respectiv CH cu cercul circumscris triunghiului. S
tiind c
a patrulaterele HBDC, HCEA si HAF B au ariile egale, ar
atati c
a ABC este echilateral.
Adriana Dragomir si Lucian Dragomir, Otelu-Rosu
DAC
(au acela


Solutie. Observ
am ca HBC
si complement C)
si ca DBC
DBC.
Analog obtinem c

(subntind acela
DAC
si arc), asadar HBC
a HCB

DCB
si atunci HBC DBC (U.L.U.), prin urmare AHBDC = 2 AHBC .

Procedam la fel pentru celelalte patrulatere si astfel ipoteza AHBDC = AHCEA =


AHAF B revine la AHBC = AHCA = AHAB . Se stie nsa ca singurul punct din interiorul unui triunghi care, unit cu varfurile, da nastere la trei triunghiuri echivalente,
este centrul de greutate G. Deducem ca n triunghiul nostru H = G, adica ABC
este echilateral.
VII.147. Trapezul dreptunghic ABCD (ABCD, AD AB) este circumscris
1
cercului de centru O. Ar
atati c
a AABCD < (OB + OC)2 .
2
Daniela Munteanu, Iasi
1

= 90 , avem c
Solutie. Deoarece m(OBC) + m(OCB) = [m(B) + m(C)]
a
2
= 90 , deci OBC este dreptunghic n O. Not
C
m(BOC)
am D
T
OB = x, OC = y; atunci raza cercului este egala cu naltimea
xy
OT a triunghiului OBC, adica r = p 2
.
Fie
M
N
N
M
x + y2
O
linia mijlocie a trapezului. Avem ca M N = OM + ON =
1p 2
(x + y)2
1
xy
2 = p
+
x
+
y
. Astfel, A
r + BC = p 2
B
2
2
x + y2
2 x2 + y 2
inegalitatea de demonstrat se rescrie succesiv:
1
(OB + OC)2 2M N AD < (OB + OC)2
2
2(x + y)2
2xy
p 2
p 2
< (x + y)2 2xy < x2 + y 2 (x y)2 > 0.
2
2 x +y
x + y2

AABCD <

Evident ca (x y)2 0 si, cum ABCD este trapez, atunci x = y. Rezulta ca


(x y)2 > 0 si solutia problemei este completa.
VII.148. Rezolvati n numere ntregi ecuatia x(x + 4) = 5(3y 1).
Neculai Stanciu, Buz
au

ie. Intruc
at x Z si (3, 5) = 1, rezulta ca y N. Daca y N , obtinem ca
Solut
3 x(x + 4) + 5, adic
a 3 (x + 2)2 + 1; acest fapt nu este posibil, deoarece (x + 2)2 nu
poate de forma M3 + 2. Ramane ca y = 0, deci x2 + 4x = 0, cu solutiile x1 = 0 si
x2 = 4. In nal, (x, y) {(0, 0); (4, 0)}.
VII.149. Fie n N, n 2 si a = 2n n+2 , b = 58n 2n+2 . Ar
atati c
a produsul
a b se poate scrie ca sum
a de patru cuburi perfecte nenule.
Constantin Dragomir, Pitesti
3

142

Solutie. Avem ca a b = 102 2(n1)n(n+1) 5(2n1)2n(2n+1) . Cum (n 1)n(n + 1) =


3p, (2n 1)2n(2n + 1) = 3q, cu p, q N , iar 102 = 13 + 23 + 33 + 43 , rezulta ca
a b = (2p 5q )3 + (2p+1 5q )3 + (3 2p 5q )3 + (2p+2 5q )3 .
a x2 +y 2
VII.150. Fie x, y numere reale strict pozitive cu x > y. Demonstrati c

2 xy (x2 y 2 ) si interpretati geometric rezultatul.


Cosmin Manea si Dragos Petric
a, Pitesti
Solutie. Inegalitatea din enunt este echivalenta cu (x2 xy y 2 )2 0, care este
2xy(x2 y 2 )
x2 + y 2

evident adevarat
a pentru x, y (0, ). Scrisa sub forma
,
2
x2 + y 2
relatia din enunt arata ca mediana ipotenuzei triunghiului dreptunghic de laturi x2
y 2 , 2xy si x2 + y 2 este
cel putin egala cu naltimea ipotenuzei. Egalitatea se atinge
atunci cand x = (1 + 2)y.

Clasa a VIII-a

VIII.144. Fie ABCDA B C D un cub si V A B C D o piramid


a patrulater
a regulat
a cu toate muchiile egale si v
arful V n exteriorul cubului. Aati sinusul unghiului
dintre dreapta A C si planul (V A B ).
Mirela Marin, Iasi
Solutie. Notam cu O centrul patratului A B C D si e T proiectia lui O pe
apotema V M , unde M este mijlocul lui A B . Ducem O P A C, cu P V A ; atunci
](A C, (V A B )) = ](O P, (V A B )) = ](O P, P r(V A B ) O P ) = ](O P, T P ) =

P T . Dac

O
a Qeste centrul cubului, din V P O V A Q obtinem ca O P =

a 2 a 3
2
V O A Q
a 6
, unde a este muchia cubului. Apoi, O T este
=
= 2
a
a 2
VQ
2(1
+
2)
2 + 2

V O O M
a 6

naltime n triunghiul dreptunghic V O M, prin urmare O T =


=
.
VM
6

OT
1+ 2
P T ) =
In concluzie, sin(O

=
.
O P
3
VIII.145. Fie a (1, ) si x1 , x2 , . . . , xn numere reale astfel nc
at x1 + x2 +
. . . + xn = a + n 1 si x21 + x22 + . . . + x2n = a2 + n 1. Determinati cea mai mare
valoare posibil
a a lui xn .
Lucian Tutescu, Craiova si Dumitru S
avulescu, Bucuresti
Solutie. Observam ca (x1 1)2 + (x2 1)2 + . . . + (xn 1)2 = (x21 + x22 + . . . +
x2n ) 2(x1 + x2 + . . . + xn ) + n = a2 + n 1 2(a + n 1) + n = (a 1)2 . Atunci
(xn 1)2 (a 1)2 , de unde xn a. Cea mai mare valoare cautata este xn = a,
atinsa cand x1 = . . . = xn1 = 1.

x2 xy = 3
.
48x2 + 4xy(x + 1)2 = (x + 1)4
Vasile Chiriac, Bac
au
Solutie. Inmultim prima ecuatie a sistemului cu 4(x + 1)2 si o adunam la cea
de-a doua; efectuand calculele,
ajungem
la (3x + 1)(x 1)(x2 + 2x + 13) = 0. Solutiile

1 26
.
sistemului sunt (1, 2) si ,
3 3
VIII.146. Rezolvati n R sistemul
2

143

VIII.147. Determinati bazele de numeratie x N\{0, 1} pentru care num


arul
N = 11111(x) este p
atrat perfect.
C
at
alin Calistru, Iasi

2
1
1
Solutie. Observ
am ca x2 + x < N = x4 +x3 +x2 +x+1 < x2 + x + 1 .
2
2

2
1
1 2
2
2
Daca x ar num
ar par, atunci a = x + x si b = x + x + 1 ar patrate
2
2
perfecte consecutive, prin urmare N nu va putea patrat perfect. Daca x este impar,

1
1 2
2
atunci N este patrat perfect daca si numai daca N = x + x +
. Efectuand
2
2
calculele, obtinem unica solutie acceptabila x = 3.
VIII.148. Stabiliti c
ate submultimi {a, b} ale multimii A = {1, 2, 3, . . . , 100} au
proprietatea c
a a3 + b3 se divide cu 12.
Dorel Luchian, Iasi
Solutie. Dac
a a r (mod 12) si a3 r (mod 12), ntre resturile r si r exista
urmatoarea corespondent
a:
r
r

0
0

1
1

2
8

3
3

4
4

5
5

6
0

7
7

8
8

9
9

10
4

11
11

.
Fie r1 si r2 resturile modulo 12 ale numerelor a, b cu proprietatea ca a3 + b3 ..12; presupunand ca r1 r2 , vom avea ca (r1 , r2 ) {(0, 0); (0; 6); (6, 6); (1, 11); (2; 4); (2, 10);
87
(4, 8); (8, 10); (3, 9); (5, 7)}. Cum A contine 8 multipli de 12, exista
= 28 de
2
submultimi {a, b} corespunzatoare situatiei (r1 , r2 ) = (0, 0). Apoi, vom avea 88 = 64
87
submultimi {a, b} pentru (r1 , r2 ) = (0, 6);
= 28 submultimi cand (r1 , r2 ) = (6, 6);
2
9 8 = 72 submultimi daca (r1 , r2 ) = (1, 11) etc. In total, obtinem 28 + 64 + 28 + 72 +
81 + 72 + 72 + 64 + 72 + 64 = 617 submultimi cu proprietatea din enunt.
VIII.149. Demonstrati c
a abc(a + b + c)2 3(a5 + b5 + c5 ), a, b, c R+ .
Gheorghe Strutu si Adrian Stan, Buz
au
Solutie. Folosind inegalitatea (a + b + c)2 3(a2 + b2 + c2 ), a, b, c R+ , ramane
sa arat
am ca a3 bc + ab3 c + abc3 a5 + b5 + c5 , fapt care rezulta din inegalitatea
rearanjarilor.


x
VIII.150. Determinati multimea A = x R 4

Z
.
x x3 + 1
Elena Iurea, Iasi
Solutie. Evident ca 0 A. Fie x A (0, ). Cum x4 x3 + 1 > 0 (x (0, 1)
x4 x3 + 1 = x4 + (1 x3 ) > 0; x [1, ) x4 x3 + 1 = x3 (x 1) + 1 > 0)
x
x
N , deci 4
1. Atunci x4 x3 x + 1 0, adica
rezulta ca 4
3
x x +1
x x3 + 1
(x 1)2 (x2 + x + 1) 0, prin urmare x = 1; se verica faptul ca 1 A.
t
Z, cu t (0, ).
Fie acum x A (, 0); pentru t = x, avem ca 4
t + t3 + 1
t
t
Deoarece 4
> 0, rezulta ca 4
1, deci t4 + t3 t + 1 0. Acest
3
t +t +1
t + t3 + 1
144

lucru este ns
a imposibil: daca t (0, 1), atunci t4 + t3 t + 1 = t4 + t3 + (1 t) > 0,
iar daca t [1, ), atunci t4 + t3 t + 1 = t4 + 1 + t(t2 1) > 0. In concluzie,
A = {0, 1}.

Clasa a IX-a

IX.126.
In triunghiul ABC not
am cu O centrul cercului circumscris si cu O

centrul cercului circumscris triunghiului median M N P . Atr


atati c
a O O = O A +

O B + O C.
Ion P
atrascu, Craiova
Solutie. Punctul O este, de fapt, centrul cercului celor noua puncte asociat
ABC, adica O este mijlocul segmentului OH (unde H este ortocentrul triunghiu

lui). T
inand seama de relatia lui Sylvester, obtinem ca O A + O B + O C = (O O +



OA)+(O O+OB)+(O O+OC) = O O+2O O+(OA+OB+OC) = O O+HO+OH =

O O.
IX.127. Fie ABC un triunghi cu AB = AC, I centrul cercului nscris, G centrul
de greutate, {D} = AI BC si {T } = IG BC. Demonstrati c
a GDAT dac
a si
numai dac
a 3a = b + c. (A se vedea si articolul din RecMat2/2011, pag. 132-133.)
Titu Zvonaru, Com
anesti
Solutie.
Fie M mijlocul laturii BC.
Folosind teorema bisectoarei si
AI
b+c
A
teorema lui Van Aubel, obtinem ca
=
. Aplicand
ID
a
acum teorema lui Menelaus n ADM cu transversala T I
T D GM IA
TD 1 b + c
G, deducem ca

= 1, deci

= 1, prin
T M GA ID
TM 2 a
TD
2a
TD
urmare
=
. In aceste conditii, GDAT
=
TM
b+c
TM
AG
2a
2

=
b + c = 3a, adica are loc echivalenta
G
AM
b+c
3
dorita.
I
IX.128. Fie n N si x1 , x2 , . . . , xn numere reale pozitive
n
P

x2i (xi + n) n2 + n. B T
C
D M
Ion Nedelcu, Ploiesti si Lucian Tutescu, Craiova
Solutie. Far
a a restrange generalitatea, putem presupune ca x1 x2 . . . xn ;
atunci x21 x22 . . . x2n si x1 + n xn + n . . . xn + n. Conform inegalitatii
n
n
n
n
P
P
P
1 P
lui Cebsev, deducem ca
x2i (xi + n) ( x2i )( (xi + n)) = (n + 1)( x2i ) si
n i=1
i=1
i=1
i=1
cu suma egal
a cu n. Demonstrati c
a

i=1

ar sucient sa mai demonstam ca


C B S: (

n
P
i=1

n
P

x2i n. Aceasta ultima inegalitate rezulta din

i=1

x2i )(

n
n
n
n
P
P
P
P
12 ) ( xi )2 n( x2i ) n2
x2i n.

i=1

i=1

i=1

i=1

IX.129. Fie a, b, c numere reale pozitive cu a b 24000 si a + 2012 +

a+b
.
b + 2012 = 2 c + 2012. Determinati partea ntreag
a a num
arului
c
Ionel Tudor, C
alug
areni (Giurgiu)
145

Solutie. Vom demonstra ca 2

a+b
a+b
< 3, deci
= 2. Avem:
c
c

2c a + b 4c + 4 2012 2(a + b) + 4 2012

( a + 2012 + b + 2012)2 2(a + b) + 4 2012

2 (a + 2012)(b + 2012) a + b + 2 2012,


fapt care rezulta din inegalitatea mediilor. Apoi, folosindu-ne de egalitatea din enunt
pentru a-l nlocui pe c, obtinem:
a + b < 3c

3
(a + b 2 2012 + 2 (a + 2012)(b + 2012)) > a + b
4

(a + 2012)(b + 2012) > a + b + 6 2012

36(p + 2012s + 20122 ) > s2 + 12 2012s + 36 20122


s2 36p < 24 2012s,
unde s = a + b, p = a b. Aceasta ultima inegalitate este nsa evident adevarata,
ntruc
at s 48000 < 24 2012, de unde s2 36p < s2 < 24 2012 s.
IX.130. Rezolvati n numere naturale ecuatia 2a + 1 = 3b2 .
Adrian Zanoschi, Iasi
Solutie. Daca a = 2n, n N, atunci 2a + 1 = 4n + 1 = M3 + 2 si 3b2 = M3 ,
deci egalitatea din enunt nu poate avea loc. Daca a = 2n + 1, n N , atunci
2a + 1 = 22n+1 + 1 = (2 + 1)(22n 22n1 + . . . + 22 2 + 1) = 3(M4 1) si deducem
ca b2 = M4 1, fals. In sfarsit, daca a = 1, obtinem ca b = 1, deci unica solutie a
ecuatiei date este (1, 1).

Clasa a X-a
X.126. Fie a, b,
c R astfel nc
at ab + bc + ca 0. Demonstrati c
a |a + bi| +
|b + ci| + |c + ai| 6(ab + bc + ca).
Ovidiu Pop, Satu Mare
Solutie. Din inegalitatea modulului obtinem c
a
|a
+
bi| + |b + ci| + |c + ai|

|(a + bi) + (b + ci) + (c + ai)|


=
|a
+
b
+
c|

|1
+
i|
=
2|a
+
b + c|. Se verica imediat

inegalitatea |a + b + c| 3(ab + bc + ca) si de aici rezulta cerinta problemei.


X.127. Dac
a a, b, c (1, ), demonstrati c
a are loc inegalitatea
(loga ab )2
(loga ac )2
(loga bc)2
+
+
1.
2
loga ab
loga ac
loga abc
D.M. B
atinetu-Giurgiu, Bucuresti
Solutie. Cu notatiile loga b = x, loga c = y, cu x, y > 0, inegalitatea de demon(1 x)2 (1 y)2
(x + y)2
strat devine
+
+
1. Aceasta relatie rezulta din inegali1+x
1+y
2xy
tatea lui Bergstrom:
(1 y)2
(x + y)2
(1 x + 1 y + x + y)2
4
(1 x)2
+
+

= = 1.
1+x
1+y
2xy
1+x+1+y+2xy
4
146

X.128.
In raport cu reperul cartezian xOy se consider
a punctul A(a, b), 0 < a < b.
a) Ar
atati c
a exist
a o innitate de puncte B, cu ambele coordonate strict pozitive,
pentru care minM Oy (M A + M B) = minN Ox (N A + N B).
b) Expuneti un procedeu de obtinere a punctelor B folosind doar rigla si compasul.
Cecilia Deaconescu, Pitesti
Solutie. Consideram B(c, d) cu proprietatea ca min (M A+M B) = min (N A+
M Oy

N Ox

N B). Fie A (a, b) simetricul lui A fata de Oy si B (c, d) simetricul lui B fata de
Ox. Notam {P } = A B Oy, {Q} = AB Ox; ca n problema biliardului se arata
ca min (M A + M B) = P A + P B = A B, iar min (N A + N B) = QA + QB =
M Oy

N Ox

AB si astfel conditia din problema revine la A B = AB . Insa A B = AB

d
a
=
(c + a)2 + (d b)2 = (c a)2 + (d b)2 ac = bd
=
tg BOx
c
b

BOx
=
semidreptele (OA
ctg AOx
AOx
si (OB sunt simetrice fata de
2
Astfel, rezult
bisectoarea unghiului xOy.
a ca exista o innitate de puncte B cu proprietatea dorita.
determin
b) Tras
am bisectoarea unghiului xOy,
am simetricul A1 al lui A fata de
aceasta bisectoare si apoi desenam semidreapta (OA1 ; toate aceste constructii pot
realizate folosind doar rigla si compasul.
X.129. Fie z1 , z2 , z3 numere complexe distincte de modul 1. Ar
atati c
a
(z1 + z2 )4
(z2 + z3 )4
(z3 + z1 )4 )
+
+
3.
2
2
(z1 z2 )
(z2 z3 )
(z3 z1 )2
Florin St
anescu, G
aesti
Solutie. Vom folosi metode din nota Aplicatii ale numerelor complexe n geometria triunghiului, publicata de autorul problemei n RecMat 1/2012. Acolo s-a
(z2 + z3 )2
(z2 + z3 )4
= cos4 A; rezulta ca inegalitatea din
aratat ca
= cos2 A, deci
4z2 z3
16z22 z32
3
enunt se poate scrie sub forma cos4 A + cos4 B + cos4 C
. Aceasta inegalitate se
16
3
demonstreaz
a folosind C B S si cunoscuta cos2 A + cos2 B + cos2 C .
4
X.130. Fie R\{k|k Z}; ar
atati c
a ecuatia x4 + 4x3 + (4 4 sin
2 sin2 )x2 (8 sin + 4 sin2 )x + (4 sin2 + 4 sin3 + sin4 ) = 0 are toate solutiile
reale.
Ionel Tudor, C
alug
areni (Giurgiu)
Solutie. Termenul liber ind nenul pentru = k, k Z, rezulta ca x =
0 nu este solutie a ecuat
iei; putem atuncimparti ecuatia prin x2 , obtinand x2 +

2
2
2 sin + sin2
(2 sin + sin )
+
4
x

+ (4 4 sin 2 sin2 ) = 0. Cu notatia


x2
x
2 sin + sin2
x
= y, g
asim ecuatia rezolventa y 2 + 4y + 4 = 0, cu solutia dubla
x
y1 = y2 = 2. Apoi, x2 + 2x (2 sin + sin2 ) = 0, adica (x + 1)2 = (1 + sin )2 ,
prin urmare solutiile ecuatiei din enunt sunt x1 = x2 = sin , x3 = x4 = 2 sin .
147

Clasa a XI-a
Not
a. Dl. Moubinool Omarjee (Paris) observa ca exemplul prezentat n solutia
Problemei XI.122 din RecMat 1/2012, pag. 59, nu este corect. Se poate construi un
exemplu bun folosind blocuri patratice de ordin 2 sau 3 pe diagonala principala.
XI.126. Fie A Mn (R) o matrice cu proprietatea c
a A4 + (A + In )4 = On .
2
Demonstrati c
a matricea A + A + In este inversabil
a.
Dan Nedeianu, Drobeta Tr. Severin
Solutie. Efectuand calculele, relatia din enunt revine la 2(A4 + 2A3 + 3A2 + 2A +
In ) = In , adica 2(A2 + A + In )2 = In . Rezulta ca A2 + A + In este matrice inversabila,
cu inversa 2(A2 + A + In ).
XI.127. Fie (xn )nN un sir de numere din intervalul (0, 1) si (Fn )nN sirul lui
Fibonacci (F1 = F2 = 1, Fn+2 = Fn+1 + Fn , n N ). Ar
atati c
a

n
X
3 3
Fk

(Fn+2 1), n N .
xk (1 x2k )
2
k=1

D.M. B
atinetu-Giurgiu, Bucuresti
1
Solutie. Studiind variatia functiei f : (0, 1) R, f (x) =
, constatam
2
x(1 x )
1
3 3
1

, x (0, 1). Atunci


ca x0 = este punct de minim; rezulta ca
2)
x(1

x
2
3

n
n
P
3 3 P
3 3
f (xk )Fk
(Fk+2 Fk+1 ) =
(Fn+2 1).
2 k=1
2
k=1

xn
XI.128. Studiati convergenta sirului (xn )nN denit prin xn+1 =
, n
xn + a
N, unde a, x0 (0, ) sunt date.
Adrian Corduneanu, Iasi
Solutie. Se observa ca xn (0, 1), n N , deci sirul (xn ) este marginit. Intrucat
xn+1 xn =

a(xn xn1 )

(xn + a)(xn1 + a)(

xn
xn+a

x
n1

xn1 +a )

rezulta ca (xn ) este strict crescator daca x1 > x0 , constant daca x1 = x0 si strict
descrescator daca x1 < x0 . In concluzie, sirul dat este convergent. Se constata ca, n
1
ecare dintre cazurile de mai sus, lim xn = ( a2 + 4 a).
u
2
XI.129.
Determinat

i
cel
mai
mic
num
a
r
real pentru care tg x 4 sin x ,
h

. (In leg
atur
a cu problema IX.109. din RecMat1/2010.)
x 0,
2
Gabriel Popa, Iasi
h

R, f (x) = tg x 4 sin x + .
Solutie. Studiem variatia functiei f : 0,
2
3
1 4 cos x
1
Avem ca f (x) =
, care se anuleaza doar n x0 = arccos
, este negativa
3
cos2 x
4

pe [0, x0 ) si pozitiva pe (x0 , ). Rezulta ca f are un minim n x0 , valoarea minima


2
148

1
4) 1
. Astfel, valoarea minima a lui pentru care
234 r
h


f (x) 0, x 0,
este = (4 3 4) 1
.
2
234
Cu ajutorul unui calculator, obtinem 1, 99681, valoarea extrem de apropiata
de = 2, care apare n enuntul problemei IX.109.
ind f (x0 ) = (4

XI.130. Se consider
a numerele reale 1 < x1 < x2 < . . . < xn < e2 astfel nc
at
n
Q

2
2
n
xi = n e si
xi = e . Ar
atati c
a exn ln x1 < x1 xn < ex1 ln xn .

n
P
i=1

i=1

Mihai Haivas, Iasi


ln x
Solutie. Consider
am functia f : (1, e ) R, f (x) = , care este strict
x
ln x1
ln x2
ln xn
crescatoare; rezulta ca
<
< . . . < . Conform unei inegalitati uzuale,
x1
x2
xn
ln x1
ln x1 x2 . . . xn
ln xn
deducem ca
<
< . Folosind ipotezele problemei,

x1
x1 + x2 + . . . + xn
xn
ln x1
n
ln xn
obtinem ca
< < , de unde inegalitatea ceruta.
x1
xn
n e
2

Clasa a XII-a

x2 1

dx, x [1, 2].


x 5x2 (x2 x + 1)2
Constantin Dragomir, Pitesti
Solutie. Pentru nceput, observam ca expresia de sub radical este strict pozitiva pentru x [1, 2]. Avem ca 5x2 (x2 x + 1)2 = x4 + 2x3 + 2x2 + 2x

1
1
1 2
1
1 = x2 x2 + 2 + 2 x +
+ 2 = x2 x +
+2 x+
+ 4 . Cu
x
x
x
x
R
1
1
substitutia x +
= t, calculul integralei din enunt revine la
dt =
x
t2 + 2t + 4
R
1
t1
x2 x + 1

dt = arcsin . In concluzie, I = arcsin


+ C, x [1, 2].
5
x 5
5 (t 1)2

XII.126. Calculati I =

Z 2

XII.127. Calculati I =
1/2

1 x 1
1+x+
e x dx.
x

Adrian Corduneanu, Iasi

2
x
+
1
1 x 1
1
x
x , rezult
) =
a
c
a
1
+
x
+
e x =
Solutie. Intruc
at (e

e
x2
x


x2 + 1 x 1
1
1
1
1+x
e x = ex x + x(ex x ) = (x ex x ) . In aceste conditii, I =
x2
2
1 3
1
3
x x

xe
= 2e 2 e 2 .

2
1/2
1
x x

XII.128. Fie f : R R o functie de dou


a ori derivabil
a, cu f (x) < 0, x
149

[a, a + c]. Demonstrati c


a
Z 2c

f (x)dx + 2cf (2a) 4

Z a+c

f (x)dx.
a

Mihai Haivas, Iasi si I.V. Maftei,


Bucuresti

x1 + x2 
Solutie. Pe intervalul [a, a+c] functia f este concava, prin urmare f

2
f (x1 ) + f (x2 )
, x1 , x2 [a, a + c]. Atunci, cum f este integrabila, avem:
2
Z a+c
a

cX
ck
f (x)dx = lim
f a+
n n
n
n

k=1
n
X

c
n 2n

lim

f (2a) + f

k=1

cX
= lim
f
n n
n

2ck
n

k=1

2a + 2ck
n
2

c
1
f (2a) +
2
4

Z 2c

f (x)dx,
0

de unde concluzia problemei.


XII.129. Fie m, p N, cu m 2 si p num
ar prim. Demonstrati c
a exist
a un
2
grup nit G care are pm elemente, n care ecare element diferit de elementul neutru
are ordinul p.
Constantin S
cheau, Ploiesti
Solutie. Se veric
a usor ca (Mn (Zp ), +) este un grup care are proprietatile dorite.
XII.130. Determinati perechile de polinoame de gradul doi, cu coecientii reali
si unitare, ce veric
a conditia c
a r
ad
acinile unuia sunt coecientii celuilalt (se au n
vedere coecientii lui X si X 0 ). Indicati polinoamele de acest fel care intr
a n pereche
cu ele nsele.
Temistocle Brsan, Iasi
Solutie. Conform relatiilor lui Vi`ete, P (X) = X 2 + p1 X + p2 si Q(X) = X 2 +
q1 X + q2 formeaz
a o pereche ce ndeplineste conditiile din enuntul problemei daca
(1)

p1 + p2 = q1 , p1 p2 = q2 , q1 + q2 = p1 , q1 q2 = p2 .

Combin
and prima si a treia ecuatie ale sistemului (1), avem (p1 + p2 )(q1 + q2 ) =
p1 q1 p2 q1 + q2 (p1 + p2 ) = 0 p2 q1 q1 q2 = 0 q1 (p2 q2 ) = 0.
Cazul q1 = 0. Sistemul (1) are solutia p1 = p2 = q1 = q2 = 0, adica obtinem
perechea de polinoame P (X) = X 2 si Q(X) = X 2 .
Cazul p2 = q2 . A doua ecuatie a sistemului (1) se va scrie p2 (p1 1) = 0 si vom
distinge doua subcazuri. Daca p2 = 0, deci si q2 = 0, (1) revine la ecuatia p1 = q1 ,
ceea ce conduce la perechile de polinoame P (X) = X 2 + pX si Q(X) = X 2 pX,
p R. Daca p1 = 1, sistemul (1) revine la 1 + p2 = q1 , q1 + p2 = 1, q1 p2 = p2 .
Cum p2 = 0, rezulta q1 = 1. Singura ecuatie ce ramane, 1 + p2 = 1, ne da p2 = 2.
Ca urmare, avem o singura solutie p1 = q1 = 1, p2 = q2 = 2, adica o pereche de
polinoame, date de P (X) = Q(X) = X 2 + X 2.
In concluzie, raspund la cerintele problemei perechile de polinoame: P (X) = X 2 +
pX si Q(X) = X 2 pX, p R, precum si P (X) = Q(X) = X 2 + X 2. Exista numai
doua polinoame ce intr
a n pereche cu ele nsele: P (X) = X 2 si P (X) = X 2 + X 2.
150

Solutiile problemelor pentru preg


atirea
concursurilor propuse n nr. 1/2012
A. Nivel gimnazial
G216.
Intr-un p
atrat 3 3 se asaz
a numerele de la 1 la 9 astfel
4 1 9
nc
at produsul numerelor de pe linia k sau produsul numerelor de pe
coloana k s
a e p
atrat perfect, pentru ecare k {1, 2, 3}. Este posibil 6 3 2
ca n centrul p
atratului s
a se ae un num
ar impar?
Marius M
ainea, G
aesti * * 8
Solutie. R
aspunsul este armativ: patratul alaturat are produsele elementelor de
pe L1 , L2 , C3 egale cu 36, 36 respectiv 144, iar n centru se aa numarul impar 3.
G217. Pe tabl
a sunt desenate p p
atr
atele, p N . Ionut coloreaz
a un p
atr
atel,
Ana coloreaz
a trei p
atr
atele, apoi Ionut coloreaz
a cinci, Ana sapte s.a.m.d. Pierde
copilul care nu mai are pe tabl
a suciente p
atr
atele de colorat atunci c
and i vine
r
andul. Determinati numerele p pentru care c
astig
atorul jocului este Ionut si stabiliti
c
ate p
atr
atele i-ar r
am
ane de colorat Anei (n functie de p).
Gheorghe Iurea, Iasi
Solutie. Cum Ionut coloreaza 4k + 1 patratele, k = 0, 1, 2, . . . , Ionut va castiga
jocul daca si numai daca 1 + 3 + . . . + (4n + 1) p < 1 + 3 + . . . + (4n + 3),

n N (2n + 1)2 p < (2n + 2)2 [ p] = 2n + 1, n N. Anei i-ar ramane de


2
colorat p [ p] patr
atele.
G218. Se consider
a numerele reale a1 , a2 , . . . , an (n N, n 2). Demonstrati c
a
n
X
1X
exist
a o submultime A {1, 2, . . . , n} cu proprietatea c
a|
ai |
|ai |.
4 i=1
iA
Radu Miron, elev, Iasi
Solutie. Denim multimile A1 = {ai |ai 0, i par}; A2 = {ai |ai < 0,
i par};
P
A3 = {ai |ai 0, i impar} si A4 = {ai |ai < 0, i impar}. Se observa ca
|x| =
xAi

P
xAi

x|, i = 1, 4 si atunci

4
P

i=1 xAi

x| =

n
P

|aj |. Ca urmare, cel putin una dintre

j=1

submultimile Ai , i = 1, 4, are valoarea absoluta a sumei elementelor cel putin egala


n
1 P
cu
|aj |.
4 j=1
G219. Fie a, b, c numere nenule, a impar, b>c astfel nc
at a=

2bc
si (a, b, c) = 1.
bc

Ar
atati c
a abc este p
atrat perfect.
Neculai Stanciu, Buz
au si Titu Zvonaru, Com
anesti
Solutie (Gheorghe Iurea). Fie d = (b, c) N ; atunci b = dx, c = dy cu
2dxy
x, y N , (x, y) = 1, iar a =
. Cum (x, x y) = (y, x y) = (x, y) = 1, rezulta
xy
2d
ca n =
este num
ar natural. Intrucat a = nxy este numar impar, numerele
xy
151

xy
xy
x, c = n
y si (a, b, c) = 1;
2
2




xy
xy
x y 2
x
y = xy
.
deducem ca n = 1 si atunci abc = (xy)
2
2
2
G220. Determinati cifrele a cu proprietatea c
a exist
a p
atrate perfecte de forma
2 aa
.
.
.
a
6.
| {z }
n, x, y vor impare. Avem ca a = nxy, b = n

n ori

Adriana Dragomir si Lucian Dragomir, Otelu-Rosu


Solutie. Cum nu exista patrate perfecte de forma M3 + 2, rezulta ca a nu poate
lua valorile 0, 3, 6 sau 9. Intrucat nu exista patrate perfecte de forma M4 +2, eliminam
si cazurile a {2, 4, 8}. Aratam n continuare ca nu putem avea nici a = 7. Se verica
imediat ca 276, 2776, 27776 si 277776 nu sunt parate perfecte. Daca n 5, atunci
2 77
. . . 7 6 = 16 1736 11
. . . 1 ; 16 este patrat perfect, n timp ce al doilea factor este
| {z }
| {z }
n de 7

n3 de 1

de forma M4 + 3, deci nu este patrat perfect.


Dac
a a = 1, num
arul 2116 = 462 este patrat perfect. Pentru a = 5, numarul
2
256 = 16 este patrat perfect. In concluzie, numerele care satisfac cerintele problemei
sunt a = 1 si a = 5.
G221. Determinati numerele naturale n pentru care


A = 168

1
1

[ n2 + n + 15] [ n2 + n + 16]

N.

Mircea Fianu, Bucuresti


Solutie. Observ
am ca
p

< 1, n N ,
+ n + 16 + n2 + n + 15

prin urmare numerele [ n2 + n + 15] si [ n2 + n + 16] sunt e egale, e consecutive.


In primul caz, cerinta problemei este ndeplinita (0 N); ramane sa studiem situatia

[ n2 + n + 16] = [ n2 + n + 15] + 1. Aceasta egalitate are loc daca si numai daca


n2 +n+16 = k 2 , k N, adica 4k 2 (4n2 +4n+1) = 63 (2k 2n1)(2k +2n+1) =
63. Avem posibilita
tile 63 = 1 63 = 3 21 = 7 9 si obtinem ca n {15, 4, 0}.
7 28
Corespunzator, A
, , 14 , prin urmare A N n N\{4, 15}.
30 5
n2 + n + 16

n2 + n + 15 =

n2

G222. Rezolvati ecuatia (x + 2)3 = x(x2 2)5 , x (0, ).


Dan Nedeianu, Drobeta Tr. Severin
Solutie. Se vede ca x = 2 este
solutie; vom arata ca ecuatia nu mai are alte solutii.
Evident, nu putem avea x (0, 2): primul membru ar pozitiv, n timp ce al doilea
ar negativ. Daca x (2, ), atunci x(x2 2) > x + 2 (inegalitate echivalenta cu
(x 2)(x + 1)2 0), iar (x2 2)4 > (x + 2)2 (echivalent cu (x 2)(x3 + 2x2 1) > 0,
ceea ce este adevarat pentru x 2). Prin nmultire, obtinem ca x(x2 2)5 > (x + 2)3 ,
deci ecuatia nu
are solutii x (2, ). Reluand rationamentul,
gasim ca x(x2 2)5 <

(x + 2)3 , x [ 2, 2), deci ecuatia nu are solutii nici n [ 2, 2).


152

G223. Pentru x, y, z 0, demonstrati c


a are loc inegalitatea
xy(x2 y 2 )2 + xz(x2 z 2 )2 + yz(y 2 z 2 )2 4(x y)2 (x z)2 (y z)2 .
Marian Tetiva, B
arlad
Solutie. Inegalitatea ind simetrica, putem presupune fara a restrange generalitatea ca x y z 0. Daca z = 0, avem de demonstrat ca xy(x2 y 2 )2
4x2 y 2 (x y)2 sau xy(x y)4 0, ceea ce este evident adevarat (cu egalitate daca
y = 0 sau daca x = y). Daca z > 0, atunci xy(x2 y 2 )2 = xy(x + y)2 (x y)2
4(x y)2 (y z)2 (z x)2 (rezulta prin nmultirea inegalitatilor x > x z 0,
y > y z 0 si (x + y)2 > (x z + y z)2 4(x z)(y z)). Adaugand n stanga
nca doi termeni nenegativi, obtinem concluzia problemei. Egalitatea se realizeaza
pentru x = y = z.
G224. Trapezul isoscel ABCD are baza mare AB si diagonalele perpendiculare
n O. Paralela prin O la baze taie laturile neparalele BC si AD n P , respectiv R.
Punctul Q este simetricul lui P fat
a de mijlocul lui BC. Dreapta RQ intersecteaz
a
AC si BD n punctele E, respectiv F . Demonstrati c
a:
a) RQ AD si RQ = AD;
b) RE = F Q = CP si P Q = EF .
Claudiu-S
tefan Popa, Iasi
Solutie. a) Fie S simetricul lui Q fata de axa de simetrie M N a trapezului, iar
{T } = SQ M N ; atunci T S = T Q si T M = ON . In triunghiurile dreptunghice
N
isoscele OCD si OAB avem ca DN = ON = T M si
D
C
AM = OM = T N . Rezulta ca AM T T N D

(C.C.), de unde AT = DT si T
AM DT
N . Cum
R
P

T AM si AT M sunt complementare, obtinem ca DT


N
O
E
U

si AT
M sunt complementare, asadar AT
D este unghi
F
drept. Astfel, AT D este dreptunghic isoscel. Daca U S
Q
T
1
este mijlocul lui AD, atunci T U AD si T U = AD.
2
Pe de alta parte, T U este linie mijlocie n SRQ, asadar
A
M
B
1
T U RQ si T U = RQ, prin urmare RQ AD si RQ = AD.
2
b
drepte, deci este inscriptibil; deducem
b) Patrulaterul DOER are unghiurile R
si O

ca m(DER) = m(DOR) = 45 . Atunci RDE este dreptunghic isoscel, cu RD =


= m(RDT
) = 45 , punctele D, E
RE. Apoi, cum m(RDE)
si T vor coliniare si

analog se arata ca punctele A, T si F sunt coliniare. Avem ca m(QF


B) = m(DF
R) =

90 m(ADB) = m(DAO) = m(DBC), deci QBF este isoscel, cu QF = QB =


SA. Deoarece RE = DR = SA, obtinem ca RE = F Q = CP. Acum, EF =
QR (ER + F Q) = AD (AS + DR) = RS = P Q si cu aceasta solutia este
completa.
G225. Lucian-Georges are o plac
a triunghiular
a omogen
a ABC de mas
a 40 si o
balant
a cu dou
a talere. El doreste s
a taie placa ABC dup
a m drepte paralele cu BC
astfel nc
at, folosind aceste pl
aci ca greut
ati dispuse pe talerele dorite ale balantei, s
a
153

poat
a c
ant
ari orice obiect cu masa num
ar natural n, cu 1 n 40. Cum l sf
atuiti
s
a procedeze, astfel nc
at m s
a e minim posibil?
Dan Br
anzei , Iasi
Solutie. Valoarea minima a lui m este 3: daca obtinem, dupa taiere, patru
A
placi cu masele 1, 3, 9, si 27, putem cantari orice obiect
cu masa num
ar natural n, cu 1 n 40. De exemplu,
B1 D C1
pentru a cant
ari un obiect cu masa 17, asez
am obiectul
1
B2 D
C2
si placile de mase 1 si 9 pe un taler si placa de masa 27
2
pe celalalt taler. (Rezultatul este cunoscut ca problema B
C3
3
D3
lui Bachet).
Fie B1 C1 , B2 C2 si B3 C3 cele trei drepte paralele
cu BC si D1 , D2 , D3 intersectiile lor cu naltimea AD.

AAB1 C1
1
AD1 2
1
Avem ca
=
, deci
=
si atunci B
C
D
AABC
40
AD
40

2
1
1+3
AD2
AAB2 C2
1
AD1 = AD. Apoi,
=
, prin urmare
, de unde
=
AABC
40
AD
10
2 10

1
AAB3 C3
1+3+9
AD3 2
13
AD2 = AD. In sfarsit,
=
, asadar
=
, si astfel
AABC
40
AD
40
10
13
AD3 = AD. In concluzie, cele trei drepte paralele cu BC mpart naltimea
2 10

AD
artile AD1 , D1 D2 , D2 D3 si D3 D proportionale cu numerele 1, 1, 13 2 si
n p

2 10 13.

B. Nivel liceal
L216. Tangentele unghiurilor unui triunghi ABC sunt numere rationale. Ar
atati
c
a numerele En = sinn A sinn B sinn C + cosn A cosn B cosn C sunt rationale,
oricare ar n N.
C
at
alin Calistru, Iasi
Solutie (Titu Zvonaru, Comanesti). Mai general, vom arata ca ecare dintre
2 tg A
termenii sumei En este numar rational. Deoarece sin 2A =
, rezulta ca nu1 + tg2 A
merele sin 2A, sin 2B, sin 2C sunt rationale. Din identitatea sin 2A + sin 2B + sin 2C =
4 sin A sin B sin C deducem ca produsul sin A sin B sin C este rational. Analog, din
1 tg2 A
cos 2A =
si cos 2A + cos 2B + cos 2C = 1 4 cos A cos B cos C obtinem ca
1 + tg2 A
si num
arul cos A cos B cos C este rational.
Not
a. S-au primit solutii corecte de la D. V
acaru si I.V. Codreanu.
L217. M
asurile unghiurilor B si C ale triunghiului ABC sunt de 70 , respectiv

ECF
F

30 . Pe latura AB se consider
a punctele E si F astfel nc
at ACE
CB.
Fie AD n
altimea din A, D BC, iar {M } = AD CF. Demonstrati c
a M B este

bisectoarea unghiului DM
F.
Eugeniu Bl
ajut, Bac
au

154

Fie {T } = BM AC. Cu teorema lui Menelaus n ADC


BD CT
A
si transversala B M T obtinem ca

BC T A
AM
CT
M D BC
CM
= 1, de unde
=

T
MD
TA
M A BD
MA
E
M D BC
M D BC
SABC

. Insa

= sin 10
=
CM BD
M C BD
SABD

AB AC sin 80
1
sin 80
F
sin 10
= sin 10

20
M
AB

AD

sin
20
sin
30
sin
20
CT
CM
2 sin 10 cos 10
=
1.
Rezult
a
c
a
=
,
deci
T
M
B
C
D
sin 20
TA
MA

este bisectoarea unghiului AM C si de aici urmeaza concluzia problemei.


Not
a. Au rezolvat problema si Gh. Iurea, T. Zvonaru, D. V
acaru si S
t.
Dominte. Solutia autorului se bazeaza pe o constructie auxiliara: considera punctul

N (CF ) astfel nc
at m(N
AC) = 20 , demonstreaza ca ABN este echilateral etc.
Solutie.

L218. Fie ABC un triunghi isoscel cu AB = AC si D un punct pe latura BC.


Consider
am punctele E si F pe laturile AB, respectiv AC astfel nc
at BD = DE si
CD = CF. Not
am {T } = BF CE. Ar
atati c
a patrulaterul BDT E este inscriptibil
dac
a si numai dac
a patrulaterul DCF T este inscriptibil.
Titu Zvonaru, Com
anesti
Solutie (Gabriel Popa, Iasi). Conditia BD = DE este sucienta pentru a obtine
concluzia problemei; vom da o solutie care nu tine seama de ipoteza CD = CF . Din
B
C. Atunci: patrulaterul
triunghiurile isoscele DBE si ABC obtinem ca DEB

DCF T este inscriptibil BT D C BT D BED patrulateral BDT E este


inscriptibil.
Not
a. Au rezolvat problema Daniel V
acaru si S
tefan Dominte.
Intre enuntul si solutia problemei primita din partea d-lui T. Zvonaru era o
neconcordant
a: enuntul este cel publicat n revista, nsa solutia se bazeaza pe ipoteza
BD = BE (n loc de BD = DE). Noua problema este mult mai dicila si expunem
n continuare solutia autorului:
Notam b = BD, c = CD, t = AB = AC. Din relatia lui Stewart
A
BC 2 AE + AC 2 BE = CE 2 AB + AB AE EB deducem ca t CE 2 = (b + c)2 (t b) + tb2 . Cu teorema lui
Menelaus n ACE cu transversala B T F obtinem
F
BE F A T C
TC
tc
ca

= 1, deci
=
, de unde
BA F C T E
TE
bt bt
CT
tc
E T
=
. Atunci: BDT E este patrulater inCE
t(b + c) bc
c
tCE 2 = B
scriptibil CT CE = CDCB
C
t(b + c) bc
D
2
2
2
2
2
c(b+c) (b+c) tb(b+c) +tb = (b+c) tbc(b+c) tb = (b+c) c(b+c) t =
b+c ABC este echilateral. Analog obtinem ca DCF T este patrulater inscriptibil
ABC este echilateral, de unde cerinta problemei.
L219. Fie date un triunghi ABC si numerele naturale m n 1. Construiti cu
rigla si compasul punctele A din planul triunghiului pentru care triunghiul A BC are
155

perimetrul si aria de m, respectiv n ori mai mare dec


at cele ale triunghiului ABC.
Temistocle Brsan, Iasi
Solutie. Cu notatiile uzuale, conditiile din enunt revin la b + c = m(a + b + c) a
si ha = n ha . Prima spune ca punctul A se aa pe elipsa E cu focarele B si C si
1
av
and semiaxa mare = [m(a + b + c) a]. A doua conditie arata ca A se aa
2
pe una din dreptele d paralele cu BC si situate la distanta nha de BC. Asadar,
A E d.
1
a2
Pentru semiaxa mica a elipsei E avem: 2 = [m(a + b + c) a]2
=
4
4
1
m(a + b + c)[m(a + b + c) 2a]. Cu rigla si compasul pot construite segmentele
4
si . Construim apoi cercurile C1 si C2 cu centrele n O (mijlocul segmentului
BC) si av
and razele de lungimi , respectiv . In conditiile problemei, dreapta d
intersecteaz
a C2 ; ntr-adev
ar,
4n2 S 2
1
m(a + b + c)[m(a + b + c) 2a]
a2
4
n2 (a + b + c)(a b + c)(a + b c) ma2 [m(a + b + c) 2a]

nha

n2 (b + c a)[a2 (b c)2 ] ma2 [m(a + b + c) 2a]


n2 (b + c a)(b c)2 a2 [n2 (b + c a) + m2 (a + b + c) 2am] 0
n2 (b + c a)(b c)2 a2 {(b + c)(m2 n2 ) + a[(m 1)2 + n2 1]} 0.
Are loc inegalitatea stricta nha < , daca excludem cazul banal m = n = 1 si b = c;
atunci avem doua puncte P1 si P2 de intersectie ntre d si C2 .
Acum, constructia punctelor A este imediata (sugerata de constructia elipsei prin
puncte): e {Q1 } = C1 (OP1 , {Q2 } = C1 (OP2 , A1 = P rd Q1 si A2 = P rd Q2 .
Alte doua puncte, A3 si A4 , se obtin n cazul n care d se aa n semiplanul opus
determinat de dreapta BC. Se justica usor faptul ca punctele Ai , i = 1, 4, astfel
construite se aa pe elipsa E, prin urmare problema are patru solutii (construibile cu
rigla si compasul).
L220. a) Fie n N, n 10. Ar
atati c
a exist
a o innitate de n-uple (x1 , x2 , . . . , xn ),
n
P
n
cu xi (0, 1), i = 1, n si
xi = .
2
i=1
x2
x1
+
+ ... +
b) Pentru un n-uplu ca la a), not
am En =
1 x1
(1 x1 )(1 x2 )
xn
1
. Ar
atati c
a
se exprim
a ca num
ar zecimal n care cel
(1 x1 ) . . . (1 xhn ) i
En + 1
n
zecimale sunt zerouri.
putin primele 3
10
Cecilia Deaconescu, Pitesti
1
1
1
Solutie. a) Daca n = 2p, luam x1 = 1 , . . . , xp = p , xp+1 = +
2
2
2

1
1
p , . . . , x2p = + 1 , unde i 0,
sunt astfel ncat 1 > 2 > . . . > p . Daca
2
2
1
1
1
1
n = 2p+1, luam x1 = 1 , . . . , xp = p , xp+1 = , xp+2 = +p , . . . , x2p+1 =
2
2
2
2
156

1
+ 1 , cu o alegere similara a numerelor i , i = 1, p.
2
n
P
n
b) Notam yi = 1 xi , i = 1, n; atunci yi (0, 1), i = 1, n si
yi = ,
2
i=1

1 y1
1 y2
1 yn
1
1
1
iar En =
+
+ ... +
=
1 +

+ ... +
y1
y1 y2
y1 y2 . . . yn
y1
y1 y2
y1

1
1
1
1

1. Astfel,
= y1 y2 . . . yn
=
y1 . . . yn
y1 . . . yn1
y1 . . . yn
En + 1

n
h i
y1 + y2 + . . . + yn
1
1
n
1
1
= n . Notand k =
, avem ca n 10k =
<
n
2
10
2
2
1024k
1
1
1
are cel putin primele 3k zecimale egale cu 0.
= 3k , deci num
arul
1000k
10
En + 1
Not
a. Am primit solutii corecte de la D. V
acaru, Pitesti, si T. Zvonaru,
Comanesti.
L221. Fie n un num
ar natural impar. Stabiliti c
ate numere naturale nenule p
au proprietatea c
a p2 + n2 este p
atrat perfect si determinati cel mai mare asemenea
num
ar.
Marian Pantiruc, Iasi
Q
i
Solutie. Fie n = n
descompunere

n
factori
primi
a
lui n, cu n1 <
n < ...,
i
Q 12
si p N cu proprietatea ca n2 + p2 = k 2 ; atunci (k p)(k + p) =
ni i , iar
2
2
k p < k + p (deoarece p = 0). Numarul de numere p pentru care n + p este patrat
perfect va , deci, cel mult egal cu numarul de divizori ai lui n2 strict mici decat n,
1Q
numar dat de N = [ (2i + 1) 1]. Vom arata ca acest maxim N este efectiv atins.
2
Q a
Exista numerele naturale 0 ai 2i astfel ncat k p =
ni i si k + p =
Q 2 a
Q 2 a
Q a
1
ni i i , prin urmare p = ( ni i i ni i ). Pentru a demonstra ca exista N
2
numere p ca n enunt, ar sucient sa justiticam faptul ca un anumit p nu poate
scris n doua moduri distincte ca n relatia precedenta. Sa presupunem, prin absurd,
ca
()

p=

Y
1 Y 2i ai Y ai
1 Y
i bi
( ni

ni ) = ( n2

nbi i ).
i
2
2

Atunci se observa ca min(2i ai , 2i bi ) = min(ai , bi ), de unde 2


max(a
,b ) =
Qi b
Qi i
min(ai , bi ), adica 2i = ai + bi . Revenind n (*), obtinem ca
ni i nai i =
Q a
Q
Q a
Q b
ni i nbi i , adica
ni i =
ni i , fapt care este adevarat doar daca ai = bi ,
1Q
oricare ar i, asadar num
arul numerelor p ca n enunt este N = [ (2i + 1) 1].
2
1
Un num
ar p este cu atat mai mare cu cat ai sunt mai mici, deci pmax = (n2 1).
2
Not
a. A rezolvat problema D. V
acaru, Pitesti.
L222. Pentru a, b, c numere reale pozitive, demonstrati inegalitatea

1
1
+ 2
2
b
c

+b

1
1
+ 2
2
c
a

+c

1
1
+ 2
2
a
b

18
.
a+b+c

Florin St
anescu, G
aesti
157

Not
a. S-au primit solutii corecte, diferite de cea data de autor, de la Daniel
V
acaru (o solutie), Titu Zvonaru (6 solutii) si I.V. Codreanu (3 solutii). A se
vedea n articolul C
ateva solutii ale problemei L222, publicat n acest numar de revista
la pag. 120.
L223. Dac
a a, b, c sunt numere reale pozitive cu a + b + c = 3, ar
atati c
a
X

ab
1 X (a b)2
+
1.
ab + a + b 9
ab + a + b

Titu Zvonaru, Com


anesti
Solutie. Avem ca 3(ab+a+b) = 3ab+(a+b)(a+b+c) = a2 +b2 +5ab+bc+ac, de
(a b)2
1
9ab + (a2 + b2 2ab) (a2 + b2 5ab bc ac)
ab
unde
+
=
ab + a + b 9(ab + a + b) 3
9(ab + a + b)
b(a c)
a(b c)
=
+
. Atunci
9(ab + a + b) 9(ab + a + b)

X
ab
1 X (a b)2
b(a c)
a(b c)
+
1
+
=
ab + a + b 9
ab + a + 1
9(ab + a + b) 9(ab + a + b)



X
X b(a c)
b(a c)2 (b + 1)
b(c a)
=

=
+
0,
9(ab + a + b) 9(bc + b + c)
9(ab + a + b)(bc + b + c)

adica tocmai inegalitatea din enunt. Avem egalitate doar daca a = b = c = 1.


Not
a. Rezultatul problemei ntareste inegalitatea propusa de Cao Minh Quang
n Crux Mathematicorum 4/2009.
Not
a. S-a primit solutie corecta de la D. V
acaru, Pitesti.
L224. Fie n si k numere ntregi pozitive. Demonstrati identit
atile:
a)

n1
X
j=1

h i

n
j
k h n i h n i
=n

+1 ;
k
k
2 k
k

b)

n
X
n
k

j=1

k
n]

X
n

q=1

qk

Marian Tetiva, B
arlad
Solutie. a) Identitatea se scrie echivalent sub forma
n1
X
j=1

[n]

k
X
nj
=
(n kr).
k
r=1

Aceasta provine din exprimarea n doua moduri a numarului de progresii aritmetice


neconstante
care au k + 1 termeni din multimea {1, 2, . . . , n}: n partea stanga,

nj
reprezint
a num
arul progresiilor aritmetice de acest tip care ncep cu j, iar n
k
membrul drept, n kr este numarul acestor progresii care au ratia r.
b) Rezulta analog, num
arand n doua moduri progresiile geometrice neconstante
cu k+1 termeni n multimea {1, 2, . . . , n} (dupa primul termen si dupa ratie); obtinem
ca

k
n

n
X
X
n
n
k
1 =
,
j
qk
q=2
j=1
158

iar aceasta inegalitate se pune cu usurinta n forma din enunt.


Not
a. Daniel V
acaru, Pitesti, rezolva problema explicitand termenii celor doua
sume si urmarind contributia lor n rezultatul nal.
L225. Fie A, B Mn (R) si X un vector nenul din Rn astfel nc
at AX = O
si exist
a Y Rn pentru care AY = BX. Not
am cu Aj matricea obtinut
a nlocuind
coloana j a matricei A cu coloana j a matricei B. Ar
atati c
a

n
P

detAj = 0.

j=1

Adrian Reisner, Paris


Solutie. Notam cu ij complementii algebrici ai matricei A si cu bij elementele
lui B . Dezvolt
and dupa coloana j determinantul matricei Aj , obtinem ca detAj =
n
P

bij ij . Cum aceasta suma este elementul de indici (j, j) al matricei M = A B

i=1

(unde A este adjuncta lui A), rezulta ca

n
P

detAj = tr M. Intrucat AX = O, cu

j=1

X = O, vom avea ca A nu este inversabila, deci rangA n 1. In cazul n care


rangA < n 1, evident ca A este nula si concluzia este imediata.
Prespunem ca rangA = n 1; atunci KerA = {X| R}. Pe de alta parte,
A A = (det A)In = On , prin urmare ImA Ker A si astfel rang A 1 (n
realitate, rangA = 1, indc
a A are macar un minor de ordin n 1 nenul). Avem
deci rang M 1 si Im M Ker A. Daca M = On , rezultatul este trivial. Daca
M = On , e {e1 , . . . , en1 } o baza a subspatiului vectorial KerM , pe care o completam
cu vectorul en pan
a la o baza a lui Rn. Matricea endomorsmului
canonic asociat

On1,n1 Kn1,1
matricei M n baza {e1 , . . . , en } este
, unde d = tr M . Are
O1,n1
d
loc echivalenta: tr M = 0 Im M Ker M . In cazul nostru, BX Im A, deci
BX Ker A si atunci M X = O, de unde concluzia problemei.

Premiu pe anul 2012 acordat de


ASOCIAT
IA ,,RECREAT
II MATEMATICE
Se acorda un premiu n valoare de 100 lei elevului

R
CEUCA
azvan-Dumitru

Colegiul National, Iasi

pentru nota ,,O dubl


a inegalitate integral
a si c
ateva aplicatii aparuta n acest numar
al revistei Recreatii Matematice, pag. 111.

159

Probleme propuse1
Clasele primare
P.240. Alina are 14 baloane rosii si verzi. Baloane verzi are cel mult 5. Cate
baloane rosii poate avea Alina?
(Clasa I )
Inst. Maria Racu, Iasi
P.241. Completati casetele cu semnele + sau astfel ncat scrierea
1234 = 122134 sa e corecta. Dati cel putin doua solutii.
(Clasa I )
Paula Balan, elev
a, Iasi
P.242. C
ate puncte se aa n interiorul patratului si cercului, dar
nu si n interiorul dreptunghiului?
(Clasa I )
Ionut Airinei, elev, Iasi

... .. . ..
.

P.243. Un elev realizeaza urmatoarea structura:  . . ., n total


44 forme geometrice. Exista o portiune a structurii n care sa se gaseasca exact trei
triunghiuri, iar num
arul dreptunghiurilor sa e 20?
(Clasa a II-a)
Mariana Nastasia, elev
a, Iasi
P.244. Avem doua vase, unul de 5 litri si celalalt de 8 litri. Cum masuram 4 litri
de apa?
(Clasa a II-a)
Codruta Filip, elev
a, Iasi
P.245. Dac
a a este cel mai mare numar natural par mai mic decat 902, iar b este
cel mai mare num
ar de trei cifre diferite cu suma 10, sa se arate ca
(b a) + (b a) + . . . + (b a) = a.
|

{z

90 termeni

(Clasa a II-a)

Lidia Balica, elev


a, Iasi

P.246. Cele doua verisoare, Oana si Camelia, au primit mere de la bunica lor.
Oana spune:
Da-mi 2 mere, pentru a avea cat tine!
Camelia raspunde:
Da-mi tu 2 mere, pentru ca merele ramase tie sa reprezinte jumatate din cat voi
avea eu!
C
ate mere a primit ecare?
(Clasa a III-a)
Inst. Maria Racu, Iasi

P.247. Intr-o cutie sunt bile. Triplam numarul bilelor si scoatem din cutie 17
bile, apoi triplam num
arul bilelor ramase si iar scoatem 17 bile s.a.m.d. Putem goli
cutia prin repetarea acestei operatii?
(Clasa a III-a)
Iulia Sticea, elev
a, Iasi
P.248. O eleva a cules mere. Vazand ca poate forma un numar exact de grupe de
10 mere, eleva da cate trei mere din ecare grupa unui camin de batrani si constata
1 Se

primesc solutii p
an
a la data de 15 ianuarie 2013.

160

ca diferenta dintre num


arul merelor care i-au ramas si cel al merelor date este 36.
Cate mere a cules eleva?
(Clasa a III-a)
Ioana Gr
aunte, elev
a, Iasi
P.249. Aratati ca num
arul 343 343 343 . . . 3437 (2012 grupe de 343) se mparte
exact la 7.
(Clasa a III-a)
Tatiana Ignat, elev
a, Iasi
P.250. Patru frati au mpreun
a 45 de ani. Varstele lor ar deveni egale daca primul
ar avea cu doi ani mai mult, al doilea cu doi ani mai putin, al treilea de doua ori mai
mult decat are, iar al patrulea jumatate din varsta pe care o are. Cati ani are ecare?

(Clasa a IV-a)
Inv. Valeria Avas
alcei, Iasi
P.251. Se considera urmatorul sir de perechi de numere: (1, 100), (2, 99), (3, 98),
. . ., (100, 1). Cate perechi (x, y) din sir au proprietatea 3x < 5y?
(Clasa a IV-a)
Nicoleta Cump
at
a, elev
a, Iasi
P.252. Se considera sirul 3; 4; 6; 9; 13; 18; 24; . . ..
a) Scrieti al 100-lea termen al sirului.
b) Aratati ca termenul de pe locul 1113334 se mparte exact la 3.
(Clasa a IV-a)
Andreea Bzdg
a, elev
a, Iasi
P.253. Varstele a doi frati sunt doua numere impare consecutive, iar varsta tatalui
lor este de patru ori mai mare decat varsta ului cel mare.
a) Aratati ca suma varstelor tatalui si ilor se mparte exact la 4, dar nu si la 3.
b) Care din numerele 32, 44, 52 poate sa reprezinte suma varstelor lor?
(Clasa a IV-a)
Amalia Munteanu, elev
a, Iasi
P.254. Cinci elevi joaca urmatorul joc. Primul scrie pe tabla doua numere distincte. Al doilea le nlocuieste cu suma si diferenta lor; la fel si ceilalti trei elevi. In
nal, pe tabla sunt scrise doua numere a caror suma este 128 si a caror diferenta este
8. Ce numere a scris primul elev pe tabla?
(Clasa a IV-a)
Petru Asaftei, Iasi

Clasa a V-a
V.151. Determinati cifra x (n baza 10) pentru care x + x2 + x3 + x4 + x5 = x6x.
Nicolae Iv
aschescu, Craiova
V.152. Determinati restul mp
artirii numarului A = 1 + 2 + 22 + . . . + 22012 prin
60.
Anca Chiritescu, T
ig
anasi (Iasi)
225

V.153. Demonstrati ca num


arul 3

are cel putin 101 cifre.


Constantin Dragomir, Pitesti

V.154. Demonstrati ca exista o innitate de patrate perfecte cu ultimele doua


cifre ale scrierii zecimale egale cu 69.
Cristian Laz
ar, Iasi
V.155. Se considera num
arul natural A = 1 00
. . . 0 44. Demonstrati ca A nu
| {z }
2012 de 0

este nici patrat perfect nici cub perfect.


Ioana Maria Popa, elev
a, Iasi
161

V.156. Fie succesiunea de cifre 1 si 0 urmatoare: 10110111011110111110 . . .


a) A 2012-a cifra este 0 sau 1?
b) Cati de 0 sunt naintea de a 2012-a cifra? Dar de 1?
Temistocle Brsan, Iasi
V.157. Pe un cerc se aa 160 de bile, numerotate n ordine crescatoare de la 1
la 160. Incep
and cu bila 1, se elimina bilele, din doua n doua, pana ramane una
singura. Ce num
ar este scris pe bila ramasa?
Petre B
atr
anetu, Galati

Clasa a VI-a
VI.151. Fie numerele p1 , p2 , . . . , p6 prime si cel putin egale cu 5. Aratati ca suma
p21 + p22 + . . . + p26 se divide cu 6, dar nu se divide cu 12.
Camelia Dan
a, Craiova
a b
VI.152. Demonstrati ca nu exista a si b cifre nenule pentru care + = a, b+b, a.
b a
Bianca Petrescu, elev
a, Iasi
VI.153. Determinati numarul solutiilor ntregi ale ecuatiei x2 + y 2 + z 2 = |x|
|y| + z.
Elena Iurea, Iasi
VI.154. Determinati n N pentru care exista numerele naturale a1 , a2 , . . . , an
cu proprietatea ca a1 + a2 + . . . + an + n = a1 a2 . . . an .
Gheorghe Iurea, Iasi
VI.155. La mp
artirea a doua numere naturale nenule, dempartitul si mpartitorul
sunt direct proportionale cu restul, respectiv catul. Restul si catul sunt prime ntre
ele. Aratati ca demp
artitul este patrat perfect si aati cele mai mici trei valori
posibile ale acestuia.
Gheorghe Bumb
acea, Busteni
b = 30 .
VI.156. Se considera triunghiul isoscel ABC, cu AB = AC si m(A)
astfel nc
= 15
Punctul D se aa n interiorul unghiului ACB,
at m(DCB)
si

m(DBA) = 75 . Demonstrati ca AC = AD.


Petrisor Rocsoreanu, Craiova
= 10
= 20 .
VI.157. Se considera triunghiul ABC cu AC = 1, m(B)
si m(C)
Demonstrati ca perimetrul triunghiului este mai mic decat 6.
Petre B
atr
anetu, Galati

Clasa a VII-a

VII.151. Comparati numerele A = 4 8 + 3 + 15 + 10 2 5 si B =

( 5 1)( 6 2) + ( 5 + 1)( 3 + 1) 5 5.
Ionel Tudor, C
alug
areni (Giurgiu)

VII.152. Determinati numere prime p, q si r, stiind ca 2p = 2012 + q 2 r2 .


Adriana Dragomir si Lucian Dragomir, Otelu-Rosu
162

VII.153. Daca n este num


ar natural par, aratati ca putem alege p {3, 5, 6}
astfel nc
at num
arul 2n + 5n + p s
a e multiplu al lui 7.
Radu Gologan, Bucuresti
VII.154. Demonstrati ca nu exista numere ntregi x cu proprietatea ca
x5 + (x + 1)5 + . . . + (x + 10)5 = 13311332.
S
tefan Dominte, elev, Iasi
VII.155. Ar
atati ca exista o innitate de numere naturale nenule a, b, c, d astfel
ncat a3 = b2 + c4 + d8 .
Lucian Tutescu, Craiova
AB
VII.156. Fie ABCD un trapez cu bazele AB si CD, astfel ncat raportul k =
CD
este num
ar rational.
Notam {O} = r
AC BD.
r
AABCD
AABCD
a) Aratati ca
Q si
Q.
AAOB
rACOD
r

AABCD
AABCD
b) Daca, n plus, k Q, atunci
Q si
Q.
AAOD
ABOC
Claudiu-S
tefan Popa, Iasi
b < m(B)

VII.157. Se considera triunghiul isoscel ABC cu AB = AC si m(A)

b
si e M un punct pe latura AC. Demonstrati ca M BC A daca si numai daca
M B 2 = AC M C.
Mirela Marin, Iasi

Clasa a VIII-a
VIII.151. Daca a, b, c R, demonstrati identitatea
2(a2 + b2 + c2 ab ac bc)2 = (a b)4 + (b c)4 + (c a)4 .
Marian Tetiva, B
arlad
VIII.152. Determinati numerele reale x, y si z pentru care x2 y 2 = x y + 2z,
y z 2 = y + z + 2x si z 2 x2 = x z + 2y.
Vasile Chiriac, Bac
au

VIII.153. Fie numerele a, x, y, z reale, pozitive si astfel ncat x yz + y zx +

z xy a. Ar
atati ca x + y + z 3a.
Lucian Tutescu, Craiova si Marian Voinea, Bucuresti
2

VIII.154. Daca n Z, consideram An = n6 + (n + 3)6 + (3n + 4)6 + (3n + 5)6 .


Demonstrati ca An se divide cu 2n2 + 6n + 5.
Cosmin Manea si Dragos Petric
a, Pitesti
VIII.155. Daca x, y, z R si x2 + y 2 + z 2 = x3 + y 3 + z 3 = 1, calculati xyz.
Lenuta Andrei, Craiova
163

VIII.156. In cubul ABCDA B C D cu muchia de lungime a, notam cu N mijlocul muchiei A D . Aratati ca dreptele BN si DC sunt perpendiculare si calculati
distanta dintre ele.
Mirela Marin, Iasi
VIII.157. Ar
atati ca suma distantelor de la centrul de greutate al unui tetraedru
la fetele sale este cel putin egala cu suma distantelor de la centrul sferei nscrise n
tetraedru la fetele acestuia.
Aurel B
arsan, Brasov

Clasa a IX-a
IX.131. Rezolvati ecuatiile:
a) x[x] + x{x} + {x}[x] = x2 + [x]2 + {x}2 ;
b) x[x] + x{x} + {x}[x] = 54 .
2

IX.132. Demonstrati ca 4

1 +

Mariana M
arculescu, Craiova
3

1 + 4(n + 1)2
1 + 1 + 4n2
5=
, n N .
2
2

Ovidiu Pop, Satu Mare

1
, stiind
IX.133. Determinati sirul de numere reale (an )n1 din intervalul 0,
2
ca sirurile (sin an )n1 si (cos an )n1 sunt progresii geometrice.
Dumitru Cr
aciun, F
alticeni
IX.134. Fie (n )n1 un sir de triunghiuri dreptunghice coplanare, ecare avand
cate un unghi ascutit de 30 , astfel ncat ipotenuza lui n+1 este cateta opusa unghiului de 30 n triunghiul n , iar celelalte doua laturi ale lui n+1 sunt n interiorul lui
n , oricare ar n N . Demonstrati ca exista un unic punct M situat n interiorul
tuturor triunghiurilor n , iar distantele an = M An formeaza o progresie geometrica
1
de ratie q = .
2
Silviu Boga, Iasi
IX.135. Aratati ca doua triunghiuri care au egale perimetrele, razele cercurilor
nscrise si cate o n
altime sunt congruente.
Temistocle Brsan, Iasi

Clasa a X-a
X.131. Intr-o urna se aa 2 bile albe si 3 bile negre, iar n alta urna se aa 4 bile
albe si 1 bila neagra. Se alege aleator o urna, din care se extrage o bila. Consideram
evenimentul B: bila extrasa este alba, iar A este un eveniment compatibil cu B astfel
4
nc
at P (A B) = si PA (B) = P (B). Calculati P (A).
5
Laurentiu Modan, Bucuresti
1
1
1
X.132. Demonstrati ca
+
+
3.
log 12 sin 70
log 12 sin 50
log 12 sin 10
Bogdan Victor Grigoriu, F
alticeni
164

X.133. Determinati tripletele de numere complexe nenule (x, y, z) pentru care


1
1
1
3
+ 6
+ 6
=
.
2|xyz|2
x6 + x6
y + y6
z + z6
Florin St
anescu, G
aesti
X.134. Fie A si B dou
a puncte xate pe un cerc de centru O si raza R, astfel
= 2 < 90 . Determinati dou
ncat m(AOB)
a puncte C si D ale cercului astfel ncat
aria patrulaterului ABCD sa e maxima.
Adrian Corduneanu si Paul Georgescu, Iasi
X.135. Demonstrati ca n orice triunghi ABC are loc inegalitatea
cos2 B
cos2 C
cos2 A
+
+
1.
sin B sin C
sin A sin C
sin A sin B
I.V. Maftei, Bucuresti si Mihai Haivas, Iasi

Clasa a XI-a
XI.131. Se considera un triunghi

0 a

a 0
1
|V (a, b, c)|, unde V (a, b, c) =
4
b c
c b

ABC,
atati ca S =
cu notatii uzuale. Ar
b c
c b
.
0 a
a 0
Constantin Dragomir, Pitesti

XI.132. Determinati functiile continue f : R R care au proprietatea ca


f (x + y) = f (x) f (y) 20122xy , x, y R, iar f (1) = 20122 .
Carmen Liana Georgescu, Craiova
(n + 2)n+1

XI.133. Daca un =
, n N , calculati lim ( n+1 u1 u2 . . . un+1
n
n
(n
+
1)

n u u . . . u ).
1 2
n
D.M. B
atinetu-Giurgiu, Bucuresti si Neculai Stanciu, Buz
au
XI.134. Fie A R o multime nevida. Daca A2 = {x2 : x A}, determinati
intervalele marginite de numere reale I R care satisfac relatia I 2 = I.
Dan Popescu, Suceava
XI.135. Fie f : [a, b] R o functie de clasa C 2 cu proprietatea ca (b a)f (a) =
f (b) f (a). Demonstrati ca exista c (a, b) pentru care f (c) f (c) = f (a).
Dan Nedeianu, Drobeta Tr. Severin

Clasa a XII-a
XII.131. Fie n 2 un num
ar natural pentru care exista un element b
a al inelului
1
Zn cu proprietatea
c
a

b
a
=
b
a
.

a) Daca a < n, demonstrati ca n = a2 + 1.


b) Aratati ca exista n 2 si a astfel ncat n = a2 + 1.
Ovidiu Pop, Satu Mare
XII.132. Fie p 5 un num
ar prim. Demonstrati ca numarul 3p1 2p1 se
divide cu p, insa 3p 2p nu se divide cu p.
R
azvan Ceuc
a, elev, Iasi
165

XII.133. Functia derivabila g : R R are proprietatea ca g (x) = 0, x R, iar


f : Img R este o functie derivabila astfel ncat f (g(x)) = x, x R. Aratati ca g
este inversabil
a, iar f este o primitiva a lui g 1 .
Dan Popescu, Suceava
XII.134. Fie f : [a, b] [0, ) o functie integrabila. Aratati ca
n Z b
Y
k=0

f 2 (x)dx (b a)n

Z b

f2

n+1

(x)dx.

Romanta Ghit
a si Ioan Ghit
a, Blaj
XII.135. Fie f : R R o functie impara si continua; aratati ca
Z 2

xf (cos x + sin x)dx =

f (cos x + sin x)dx.


0

Adrian Corduneanu, Iasi

Probleme pentru preg


atirea concursurilor
A. Nivel gimnazial
G226. C
ate dintre numerele de trei cifre n baza 10 se pot scrie sub forma abc +
ab + a?
Andrei Eckstein, Timisoara

1
1 1 1
. Demonstrati ca pentru
G227. Se considera multimea M = 1, , , , . . . ,
2 3 4
100
ecare n {3, 4, . . . , 15}, exista o submultme A = {a1 , a2 , . . . , an } a lui M si o alegere
convenabil
a a semnelor astfel ncat a1 a2 . . . an = 0.
Gabriel Popa, Iasi
G228. Spunem ca num
arul natural m are proprietatea (P) daca exista a, b N
7m + 4
a
astfel nc
at a+b 3m si
= . Notam cu E(n) numarul elementelor multimii
11m + 2
b
{1, 2, . . . , n} care au proprietatea (P). Determinati n N pentru care E(n) = 2012.
Vlad Emanuel, Bucuresti
G229. Se considera numerele ntregi distincte a, b, c si d, cu proprietatea ca
ab + ac + ad + bc + bd + cd 500. Demonstrati ca a2 + b2 + c2 + d2 340.
Dan Nedeianu, Drobeta Tr. Severin
x
y
z
1
G230. Aratati ca 3
+ 3
+ 3
, x, y, z
y + z 3 10
z + x3 10
x + y 3 10
4
[1, 0).
Bogdan Chiriac, student, Iasi
G231. Aratati ca oricum am aseza 2012 puncte n interiorul unui triunghi echilateral de latura 2012, exista cel putin doua puncte astfel ncat distanta dintre ele sa
e mai mica decat 46.
Nicolae Iv
aschescu, Craiova
166

= 90
G232. Fie ABCD un trapez dreptunghic cu ABCD, m(B)
si AB =
BC = a. Not
am cu E mijlocul segmentului BC. Demonstrati ca AE este bisectoarea
a
dac
unghiului BAD
a si numai daca CD = .
4
Adrian Zanoschi, Iasi

G233. Determinati punctele M din planul triunghiului echilateral ABC cu proprietatea ca M B = 2M A si M C = 3M A.


Temistocle Brsan, Iasi
G234. Fie ABC un triunghi cu AB < AC, D un punct pe latura AC astfel ncat
AD = AB si M mijlocul laturii BC. Paralela la AC prin M intersecteaza pe BD n
punctul E, iar dreapta AE intersecteaza latura BC n punctul F . Daca paralela prin
M

F la AC intersecteaz
a pe BD n punctul T , demonstrati ca BAT
AC.
Titu Zvonaru, Com
anesti
G235. Fie ABC un triunghi ascutiunghic, M punctul n care bisectoarea din A
retaie cercul circumscris, iar I centrul cercului nscris. Fie D piciorul naltimii din
A, iar N si P sunt proiectiile punctului I pe BC, respectiv AD. Aratati ca punctele
M, N si P sunt coliniare.
Neculai Roman, Mircesti (Iasi)

B. Nivel liceal
L226. Fie C cercul circumscris triunghiului oarecare ABC, iar A1 centrul cercului
tangent interior cercului C si laturilor [AB], [AC]. In mod analog construim punctele
B1 si C1 . Fie A2 centrul cercului tangent exterior cercului C si semidreptelor [AB, [AC
si n mod similar construim punctele B2 si C2 . Aratati ca A1 B1 C1 A2 B2 C2 .
Neculai Roman, Mircesti (Iasi)
L227. Fie ABC un triunghi ascutitunghic, D un punct pe latura BC si M
BM 2
CM 2
simetricul lui A fat
a de D. Daca
+
= AB + AC, aratati ca AD este
AB
AC
bisectoare sau n
altime n ABC.
Titu Zvonaru, Com
anesti si Neculai Stanciu, Buz
au

L228. In triunghiul ABC consideram simedianele AD si BE, avand mijloacele


ABP
.
P , respectiv Q. Demonstrati ca BAQ

Titu Zvonaru, Com


anesti
L229. Consideram cercurile de ecuatii x +2Rx+y = 0, respectiv x2 2Rx+y 2 =
0 si e C1 si C2 centrele lor. Cercul cu centrul n C(0, y) cu y > 0 este tangent celor

doua cercuri date. Fie S = S(), 0 < <


aria suprafetei din semiplanul superior
2
marginit
a de cele 3 cercuri, unde este masura (n radiani) a unghiurilor de
 la baza

C1 C2 C. Demonstrati ca functia S = S() este strict crescatoare pe 0,
si
2
calculati lim S().
2

Adrian Corduneanu, Iasi


L230. Pentru x R, demonstrati inegalitatile:
p

2
a) 1 + 2 cos2 x sin2 x + 1 + 2 sin2 x cos2 x + | sin x| + | cos x|;
3 3
167

2
3 2 cos4 x cos4 x 1 .
3 3
Mih
aly Bencze, Brasov
L231. Un paralelipiped dreptunghic are dimensiunile x, y, z si diagonala d. Aratati
b)

3 2 sin4 x sin4 x +

ca

d4
d4
3a + 2b
d4
+
+

,
4
4
4
4
4
4
ad + bx
ad + by
ad + bz
a(a + b)

oricare ar a > 0 si b 0.
Marius Olteanu, Rm. V
alcea
L232. Daca a b c > 0, aratati ca are loc inegalitatea

a6 + b6 + c6 3a2 b2 c2 (a3 c3 ) (a3 b3 )(b3 c3 ).


Marian Tetiva, B
arlad
L233. Fie n 2 un numar natural si a1 , a2 , . . . , an numere reale pozitive cu
proprietatea ca a1 + a2 + . . . + an 1. Demonstrati inegalitatea
a31

a1
a2
an
n3
+ 3
+ ... + 3
3
.
2
2
2
+ a1 + 1 a2 + a2 + 1
an + an + 1
n +n+1

Titu Zvonaru, Com


anesti
L234. Determinati multimile A de numere reale cu proprietatea ,,x, y R, x2 +
y 2 A x3 + y 3 A.
Vlad Emanuel, Bucuresti
L235. Fie f : I R o functie de doua ori derivabila cu derivata a doua marginita
pe intervalul I. Demonstrati ca exista un numar k 0 (depinzand de functia f ) astfel
nc
at inegalitatea

x + y
f (x) + f (y) + 6f
+ k(x y)2 4 f
2

x + 3y
4

+f

3x + y
4

sa e adevarat
a pentru orice x, y I.
Marian Tetiva, B
arlad

Training problems for mathematical contests


A. Junior highschool level
G226. How many among the numbers of three digits in basis 10 can be written
under the form abc + ab + a ?
Andrei Eckstein, Timisoara

1 1 1
1
1, , , , . . . ,
. Prove that, for each
2 3 4
100
n {3, 4, . . . , 15}, there is a subset A = {a1 , a2 , . . . , an } of M and an appropriate
selection of the signs so that a1 a2 . . . an = 0.
Gabriel Popa, Iasi
G227. It is considered the set M =

168

G228. Let us say that the natural number m has the property (P) if there are
7m + 4
a
a, b N such that a + b 3m and
= . We say that E(n) is the number
11m + 2
b
of elements in the set {1, 2, . . . , n} that own property (P). Determine n N such
that E(n) = 2012.
Vlad Emanuel, Bucuresti
G229. There are considered the distinct integer numbers a, b, c and d, with the
property that ab + ac + ad + bc + bd + cd 500. Prove that a2 + b2 + c2 + d 2 340.
Dan Nedeianu, Drobeta Tr. Severin
x
y
z
1
G230. Show that 3
+ 3
+ 3
, x, y, z
y + z 3 10
z + x3 10
x + y 3 10
4
[1, 0).
Bogdan Chiriac, student, Iasi
G231. Show that, whichever 2012 points are arranged inside an equilateral triangle of side length 2012, there are at least two points so that the distance between
them is less than 46.
Nicolae Iv
aschescu, Craiova
= 90 and
G232. Let ABCD be a right-angled trapezium with ABCD, m(B)
AB = BC = a. Let E be the middle point of BC. Prove that AE is the angle-bisector
a
if and only if OD = .
of BAD
4
Adrian Zanoschi, Iasi

G233. Determine the points in the plane of the equilateral triangle ABC with
the property that M B = 2 M A and M C = 3 M A.
Temistocle Brsan, Iasi
G234. Let ABC be a triangle with AB < AC, D a point on the side AC such
that AD = AB and M = the midpoint of the side BC. The parallel to AC prin M
intersects BD at point E, and the line AE intersects the side BD at point F. If
M

the parallel through F to AC intersects BD at point T , show that BAT


AC.
Titu Zvonaru, Com
anesti
G235. Let ABC be an acute-angled triangle, with M = the point where the
angle-bisector from A cuts the circumcircle again, and let I be the centre of the
inscribed circle. Let D be the foot of the altitude from A, and let N and P be the
projections of point I on BC, respectively AD. Show that the points M, N and P
are collinear.
Neculai Roman, Mircesti (Iasi)

B. Highschool Level
L226. Let C be the circumcircle of an arbitrary (scalene) triangle ABC and let
A1 be the centre of the circle wich is tangent from inside to circle C and to the sides
[AB], [AC]. The points B1 and C1 are analogously built. Let A2 be the centre of the
exterior circle which is tangent to C and to the half-lines [AB, [AC . The next points
B2 si C2 are similarly built. Show that A1 B1 C1 A2 B2 C2 .
Neculai Roman, Mircesti (Iasi)
169

L227. Let ABC be an acute-angled triangle, D a point on the side BC and


BM 2
CM 2
M = the symmetric point of A with respect to D. If
+
= AB + AC,
AB
AC
show that AD is an angle-bisector or an altitude in ABC.
Titu Zvonaru, Com
anesti and Neculai Stanciu, Buz
au
L228. We consider, in the triangle ABC , the symmedians AD and BE with
ABP
.
their midpoints P and respectively Q. Show that BAQ
Titu Zvonaru, Com
anesti
L229. We consider the circles of equations x2 + 2Rx + y 2 = 0, respectively
x2 2Rx + y 2 = 0 and let C1 , C2 be their centres. The circle with its centre at

C(0, y), y > 0 is tangent to the two given circles. Let S = S(), 0<< be the area
2
in the superior (positive) half-plane of the zone bounded by the three circles, where
is the measure in radians of the angles at the base
1 C2 C. Prove that the function
 of C

and calculate lim S().
S = S() is strictly increasing on the interval 0,
2
2
Adrian Corduneanu, Iasi
L230. For x R, prove the inequalities :
p

2
a) 1 + 2 cos2 x sin2 x + 1 + 2 sin2 x cos2 x + | sin x| + | cos x|;
3 3
p

2
4
4
4
4
b) 3 2 sin x sin x + 3 2 cos x cos x 1 .
3 3
Mih
aly Bencze, Brasov
L231. A right-angled parallelepiped has the dimensions (edge lengths) x, y, z and
the diagonal d. Show that
d4
d4
d4
3a + 2b
+
+

ad 4 + bx4
ad 4 + by 4
ad 4 + bz 4
a (a + b)
for all a > 0 and b 0.
Marius Olteanu, Rm. V
alcea
L232. If a b c > 0, show that the following inequality holds :

a6 + b6 + c6 3a2 b2 c2 (a3 c3 ) (a3 b3 )(b3 c3 ).


Marian Tetiva, B
arlad
L233. Let n 2 be a natural number and let a1 , a2 , . . . , an be some real positive
numbers such that a1 + a2 + . . . + an 1. Prove the following inequality:
a1
n3
a2
an

.
+
+
.
.
.
+
a31 + a21 + 1 a32 + a22 + 1
a3n + a2n + 1
n3 + n + 1
Titu Zvonaru, Com
anesti
L234. Determine the sets A of real numbers with the property x, y R, x2 +
y 2 A x3 + y 3 A.
Vlad Emanuel, Bucuresti
170

L235. Let f : I R be a twice dierentiable function with its second derivative


bounded on the interval I. Show that there is a number k 0 (depending on
function f ) so that the inequality


f (x) + f (y) + 6 f

x + y
+ k (x y)2 4 f
2

x + 3y
4

+f

3x + y
4

hold true for any x, y I.


Marian Tetiva, B
arlad

Primul numar al Colectiei ,,Recreatii Matematice

1. D. Br
anzei, Al. Negrescu Probleme de pivotare,
Ed. ,,Recreatii Matematice, Iasi, 2011 (208 pag.)
poate fi procurat printr-o simpla cerere la adresa: t birsan@yahoo.com si
indicarea adresei postale proprii. Cartea va fi trimisa cu plata ramburs
la adresa indicata contra sumei de 25 lei (inclusiv taxe postale).

171

Pagina rezolvitorilor
CRAIOVA
Colegiul National ,,Fratii Buzesti. Clasa a X-a (prof. TUT
ESCU Lucian).
ENUCA Andreea Ramona: VIII(145,146), IX(126,128), X(127,129); GOLEA Monica:
VIII(145,146,150), IX(126,128,130), X(127,129).
GRAJDURI (IAS
I)
Ana-Lacramioara). AFRA
S
coala ,,Valea Satului. Clasa a III-a (nv. URMA
SINEI Sebastian: P(226-228,232,235); RUSU Alexandra: P(226-228,232,235); STOICA Mariana-Ionela: P(226-228,232,235).
IAS
I
S
coala nr. 3 ,,Al. Vlahut
a. Clasa a IV-a (prof.nv.primar MAXIM Gabriela).

DASCALU
Lorena: P(228-232); NICA Daniel: P(228-232); ROBU Carmen: P(228232). Clasa a V-a (prof. MARIN Mirela). ENEA Codrut: P(236-238), V(144,145);
Andreea: P(236-238),
POPOVICI Teodor-Andrei: P(236-238), V(144,145); ROMILA
V(144,145). Clasa a VI-a (prof. MARIN Mirela). MARIN Marius: V(144,145),
VI(145,149,150); VLAD Ioana: V(144,145), VI(145,149,150).
S
coala nr. 11 ,,Otilia Cazimir. Clasa IV-a (nv. MARDARE Carmen). OLENIUC Iulian: P(226,228-230,234,238), V.144.
S
coala nr. 26 ,,G. Cosbuc. Clasa a II-a (inst. RUSU Eugenia). AVRAM Elena:
P(226-230); Barghi Sara: P(226-230); CAZACU Costin: P(226-230); CIRDEI Dan:
P(226-230); CRISTEA Alexandru: P(226-230); FRONEA Bogdan: P(226-230); IA Alexandra: P(226-230); PARA
Gabriel: P(226-230).
COB Anca: P(226-230); LAZAR

Clasa a III-a (prof.nv.primar RACU Maria). CIOPEICA Sebastian-Andrei: P(226229, 236); GROSU Victor-Alessandru: P(227-229,232,234); LUCHIAN Maria-Clara:
P(226,228-230, 232-234,236); MANOLE Alexandra-Georgiana: P(226-230,233); PASNICU Cosmin-Constantin: P(226,228,232-234); Popescu Andrei-Eduard: P(228-230,

235,236); RAILEANU
Ana-Maria: P(226,228,232-234); RAILEANU
Razvan-Constantin: P(226-230); VASILE Raluca-Andreea: P(226,227,230,233,234).
S
coala nr. 33 ,,M. Kog
alniceanu. Clasa a II-a (nv. SIRBU Lenuta). CIOCOIU
Alexandru Boris: P(229-232,235-239).
Grupul S
colar ,,Virgil Madgearu. Clasa a X-a (prof. OLENIUC Claudia). MEIU Luminita-M
ad
alina: VIII.146, IX(126,127,130), XI.126; SIRGHI Nicoleta: VIII.146,
IX(126,127,130), XI.126; SOFRONEA Daniela: VIII.146, IX(126,127,130), XI.126.
Loredana). LULiceul Teoretic ,,Miron Costin. Clasa a III-a (nv. T
ARA
CHIAN Denisa: P(228,230,232-234,237), V.144.
Colegiul National. Clasa a V-a (prof. BENT
A Valerica). SAADE K. San
dra: P(236-239), V(144-147,150), VI.147, G.216. Clasa a V-a (prof. LAZAR
Cristian). POPA Ioana-Maria: P(236-239), V(144-150), VI(144-148), VII(148,149),
XI.130, G(216,217,220); PRIOTEASA Ioana-Cristiana: P(236-239), V(144-147,
149,150), VI.147, G.216. Clasa a VI-a (prof. POPA Gabriel). ASTEFANEI Cosmin:
Stefan: P(236-239), V(144,146,147,
P(236-239), V(144,145,147), VI(149,150); OBADA
149), VI(146,147,149,150). Clasa VII-a (prof. POPA Gabriel). DOMINTE Stefan:
172

VI(144-150), VII(144-150), VIII(145-150), IX(127,130), G(216,217,220,221,224), L(217,


Razvan-Dumitru: X(126,127,130),
218). Clasa a XII-a (prof. POPA Gabriel). CEUCA
XI(126,128), XII(126-128,130).
PAS
CANI

S
coala ,,Iordachi Cantacuzino. Clasa a IV-a (nv. MIRON Petru). CRACIUN
Stefana-Maria: P(226-238).

T
IGANAS
I (IAS
I)
S
coala cu clasele I-VIII ,,M. Kog
alniceanu. Clasa a II-a (nv. SAMSON DanielMihai). DUCA DARIA Cat
alina-Stela: P(229-239); PASANCU Andreea: P(229 A Aurica). DUCA Ema232, 234,236-239). Clasa a III-a (prof.nv.primar BADIT
Alexandra-Ionela: P(229-239); PASANCU EduStefania: P(229-239); GHIOANCA
ard: P(229-239); SANDU Marta: P(229-239); STEFANACHE Anamaria: P(229239); TICAN David-Petru: P(229-239).

Elevi rezolvitori premiati


S
coala nr. 3 ,,Al. Vlahuta
, Iasi
NICA Ioan-Daniel (cl. a IV-a): 2/2011(6pb), 1/2012(6pb), 2/2012(5pb).
ENEA Codrut (cl. A V-a): 1/2011(7pb), 1/2012(6pb),2/2012(5pb).
POPOVICI Teodor-Andrei (cl. a V-a): 2/2011(6pb), 1/2012(6pb), 2/2012(5pb).
Andreea (cl. a V-a): 2/2011(6pb), 1/2012(6pb), 2/2012(5pb).
ROMILA
VLAD Ioana (cl. a VI-a): 2/2011(5pb), 1/2012(6pb), 2/2012(5pb).
S
coala nr. 26 ,,G. Cosbuc, Iasi
POPESCU Andrei-Eduard (cl. a III-a): 1/2011(5pb); 1/2012(6pb); 2/2012(5pb).
S
coala ,,M. Kog
alniceanu, T
ig
anasi (Iasi)
DUCA Daria-C
at
alina-Stela (cl. a II-a): 2/2011(5pb); 1/2012(6pb); 2/2012(11pb).
DUCA Ema-S
tefania (cl. a III-a): 2/2011(5pb); 1/2012(6pb); 2/2012(11pb).
Colegiul National, Iasi
S
OBADA
tefan (cl. a VI-a): 2/2011(7pb), 1/2012(5pb), 2/2012(12pb).

173

IMPORTANT
In scopul unei legaturi rapide cu redactia revistei, pot fi utilizate urmatoarele
adrese e-mail: t birsan@yahoo.com si profgpopa@yahoo.co.uk . Pe
aceasta cale colaboratorii pot purta cu redactia un dialog privitor la materialele trimise acesteia, procurarea numerelor revistei etc. Sugeram colaboratorilor care trimit probleme originale pentru publicare sa le numeroteze
si sa-si retin
a o copie xerox a lor pentru a putea purta cu usurinta o discutie
prin e-mail asupra acceptarii/neacceptarii acestora de catre redactia revistei.
La problemele de tip L se primesc solutii de la orice iubitor de matematici
elementare (indiferent de preocupare profesional
a sau v
arst
a ). Fiecare dintre
solutiile acestor probleme - ce sunt publicate n revista dupa un an - va fi
urmata de numele tuturor celor care au rezolvat-o.
Adres
am cu insistent
a rug
amintea ca materialele trimise revistei
s
a nu fie (s
a nu fi fost) trimise si altor publicatii.
Rugam ca materialele tehnoredactate sa fie trimise pe adresa redactiei
nsotite de sierele lor (de preferinta n LATEX).
Pentru a facilita comunicarea redactiei cu colaboratorii ei, autorii materialelor sunt rugati sa indice adresa e-mail.

174

Revista semestrial RECREAII MATEMATICE este editat de


ASOCIAIA RECREAII MATEMATICE. Apare la datele de 1 martie i
1 septembrie i se adreseaz elevilor, profesorilor, studenilor i tuturor celor
pasionai de matematica elementar.
n atenia tuturor colaboratorilor
Materialele trimise redaciei spre publicare (note i articole, chestiuni de
metodic, probleme propuse etc.) trebuie prezentate ngrijit, clar i concis; ele
trebuie s prezinte interes pentru un cerc ct mai larg de cititori. Se recomand ca
textele s nu depeasc patru pagini. Evident, ele trebuie s fie originale i s
nu fi aprut sau s fi fost trimise spre publicare altor reviste. Rugm ca materialele tehnoredactate s fie nsoite de fiierele lor (trimise la adresa de mai jos).
Problemele destinate rubricilor: Probleme propuse i Probleme pentru
pregtirea concursurilor vor fi redactate pe foi separate cu enun i demonstraie/rezolvare (cte una pe fiecare foaie) i vor fi nsoite de numele autorului, coala i localitatea unde lucreaz/nva.
Redacia va decide asupra oportunitii publicrii materialelor primite.
n atenia elevilor
Numele elevilor ce vor trimite redaciei soluii corecte la problemele din
rubricile de Probleme propuse i Probleme pentru pregatirea concursurilor
vor fi menionate n Pagina rezolvitorilor. Elevii vor ine seama de regulile:
1. Pot trimite soluii la minimum cinci probleme propuse n ultimul numr aprut al revistei. Pe o foaie va fi redactat soluia unei singure probleme i
vor fi menionate datele: numele i prenumele, clasa, profesorul, coala i
localitatea.
2. Elevii din clasele VI-XII au dreptul s trimit soluii la problemele
propuse pentru clasa lor, pentru orice clas mai mare, din dou clase mai mici i
imediat anterioare. Elevii din clasa a V-a pot trimite soluii la problemele propuse
pentru clasele a IV-a, a V-a i orice clas mai mare, iar elevii claselor I-IV pot
trimite soluii la problemele propuse pentru oricare din clasele primare i orice clas mai mare. Orice elev poate trimite soluii la problemele de concurs (tip G i L).
3. Oricine poate trimite soluii la problemele de tip L; la publicare, soluiile
acestora vor fi urmate de numele tuturor rezolvitorilor.
4. Plicul cu probleme rezolvate se va trimite prin pot (sau va fi adus
direct) la adresa Redaciei:
Prof. dr. Temistocle Brsan
Str. Aurora, nr. 3, sc. D, ap. 6,
700 474, Iai
Jud. IAI
E-mail: t_birsan@yahoo.com

CUPRINS
DAN BRNZEI (1942 - 2012)................................................................................................85
Caius IACOB 100 de ani de la natere (A. Corduneanu).........................................................87
Henri POINCAR 100 de ani de la dispariia sa (V. Oproiu)..........................................89

ARTICOLE I NOTE
T. BRSAN Mulimi de puterea continuului n spaii euclidiene............................................92
M. TETIVA Cum putem folosi ntregii algebrici n matematica elementar ....................... 98
C. DRAGOMIR Conexiuni cu cercul celor ase puncte ................................................... 103
D.M. BTINEU-GIURGIU, N. STANCIU Cteva inegaliti geometrice
n poligoane convexe i nu numai..................................................................106

NOTA ELEVULUI
R.-D. CEUC O dubl inegalitate integral i cteva aplicaii..........................................111

CORESPONDENE
A. REISNER Cercles et inversion..................................................................................... 113
Concursul Internaional de Matematic Vladimir Andrunachievici, ed. II, Chiinu, 2012..........115

CHESTIUNI METODICE
T. ZVONARU Cteva soluii la problema L222 din nr. 1/2012 ....................................... 120

CUM CONCEPEM... CUM REZOLVM


T. BRSAN Cum se construiete un contraexemplu ......................................................... 124

COLI I DASCLI
O. DIACONU Institutele Unite veche coal particular model.................................. 128

CONCURSURI I EXAMENE
Concursul de matematic Florica T. Cmpan, ed. a XII-a, Iai, 2012 ................................ 131

PROBLEME I SOLUII
Soluiile problemelor propuse n nr. 1/2012........................................................................ 136
Soluiile problemelor pentru pregtirea concursurilor propuse n nr. 1/2012.................... 151
Probleme propuse................................................................................................................... 160
Probleme pentru pregtirea concursurilor ............................................................................ 166
Training problems for mathematical contests ..................................................................... 168
Pagina rezolvitorilor ............................................................................................................ 172
Elevi rezolvitori premiai...................................................................................................... 173
ISSN 1582 1765

8 lei

S-ar putea să vă placă și